You are on page 1of 171

ARELLANO UNIVERSITY SCHOOL OF LAW

A Compilation of case digest submitted to


ATTY. VICTOR CARLO ANTONIO V. CAYCO

In partial fulfillment of the requirements for Finals in


Criminal Procedure

Law Students from Section 23


Tuesday, 5:30-8:30 PM​

TABLE OF CONTENTS
GARCIA vs SANDIGANBAYAN ​5
ARNEL ESCOBAL vs. HON. FRANCIS
GARCHITORENA ​7
PEOPLE OF THE PHILIPPINES vs. HENRY T. GO ​10
RAMISCAL vs. SANDIGANBAYAN ​12
PEOPLE OF THE PHILIPPINES vs. ALFREDO L.
BENIPAYO ​14
LACSON vs EXECUTIVE SECRETARY ​16
SANCHEZ vs. DEMETRIOU ​18
SERANA vs. SANDIGANBAYAN ​22
DISINI vs SECRETARY OF JUSTICE ​24
JIMENEZ vs. SORONGON ​25
PEOPLE vs. VALDEZ ​27
MIGUEL vs. SANDIGANBAYAN ​29
PEOPLE vs SORIA ​31
UNION BANK vs PEOPLE ​33
DR. FERNANDO P. SOLIDUM vs. PEOPLE OF THE
PHILIPPINES ​34
CASTILLO vs. SALVADOR ​36
LIM vs KOU CO PING ​39
CASUPANAN vs LAROYA ​41
PEOPLE vs. ROMERO ​43
MAGISTRADO vs. PEOPLE ​45
PIMENTEL vs PIMENTEL ​47
JM DOMINGUEZ AGRONOMIC COMPANY, INC., et al vs.
CECILIA LICLICAN et al ​49
FENEQUITO vs. VERGARA, JR. ​51
BURGUNDY REALTY CORPORATION vs. JOSEFA "JING"
C. REYES ​53
CITY PROSECUTOR ARMANDO P. ABANADO vs. JUDGE
ABRAHAM A. BA YONA ​55
HEIRS OF THE LATE NESTOR TRIA vs. ATTY. EPIFANIA
OBIAS ​57
UY vs. JAVELLANA ​60
PEOPLE OF THE PHILIPPINES vs. SEGUNDINO
VALENCIA ​62
PCGG vs. NAVARRO-GUTIERREZ ​64
DE LIMA vs REYES ​66
AMADO I. SARAUM vs. PEOPLE OF THE
PHILIPPINES ​68
COMERCIANTE vs PEOPLE ​70
LUZ vs. PEOPLE OF THE PHILIPPINES ​73
GEORGE ANTIQUERA Y CODES vs. PEOPLE OF THE
PHILIPPINES ​76
PEOPLE OF THE PHILIPPINES vs. DONALD
VASQUEZ ​78
FLORESTA vs. UBIADAS ​79
ZUNO vs. CABEBE ​81
GOVERNMENT OF HONGKONG SPEC. ADM. REGION VS
OLALIA ​82
PEOPLE OF THE PHILIPPINES vs. SANDIGANBAYAN
(FIRST DIVISION), VICTORINO A. BASCO, ROMEO S.
DAVID, and ROGELIO L. LUIS ​84
TERESITA TANGHAL OKABE vs. HON. PEDRO DE LEON
GUTIERREZ ​87
LEVISTE vs CA ​90
JUAN PONCE ENRILE vs. SANDIGANBAYAN ​92
DEL CASTILLO vs. PEOPLE OF THE PHILIPPINES ​95
PEOPLE OF THE PHIPPINES vs. ARTURO LARA y
ORBISTA ​97
JOSE SANICO vs. PEOPLE OF THE PHILIPPINES ​99
THE PEOPLE OF THE PHILIPPINES vs. JUDGE RUBEN
AYSON ​101
VILLARUEL vs. PEOPLE OF THE PHILIPPINES ​103
PEOPLE OF THE PHILIPPINES vs. ESTOMACA ​105
PEOPLE OF THE PHILIPPINES vs. ALFREDO
PANGILINAN ​107
DAAN VS SANDIGANBAYAN ​108
PEOPLE vs. JANJALANI, ET. AL. ​110
ABS-CBN vs GOZON ​112
ENRILE vs PEOPLE ​114
PEOPLE vs LACSON ​116
PANAGUITON VS DOJ ​118
PEOPLE v DUMLAO ​121
SORIANO vs. PEOPLE ​124
CEREZO VS PEOPLE ​126
WILLIAM CO a.k.a. XU QUING HE vs. NEW PROSPERITY
PLASTIC PRODUCTS ​128
PEOPLE vs DE LEON ​130
SALVANERA vs. PEOPLE OF THE PHILIPPINES ​132
VDA. DE MANGUERRA vs RAUL RISOS ​134
MANUEL J. JIMENEZ, JR. vs. PEOPLE OF THE
PHILIPPINES ​136
PEOPLE vs DE GRANO ​138
JOCELYN ASISTIO vs. PEOPLE OF THE
PHILIPPINES ​141
ANTONIO CABADOR vs. PEOPLE OF THE
PHILIPPINES ​143
PEOPLE vs. TAN ​146
NELSON IMPERIAL, ET AL., vs. MARICEL M.
JOSON ​148
PEOPLE VS SANDIGANBAYAN ​150
SALLY GO-BANGAYAN vs. BENJAMIN BANGAYAN,
JR ​152
PEOPLE OF THE PHILIPPINES vs. JOSE C. GO ​154
PEOPLE OF THE PHILIPPINES vs. JERRY PEPINO AND
PRECIOSA GOMEZ ​158
LLAMAS vs CA ​160
NILO HIPOS vs. JUDGE TEODORO A. BAY ​162
PEOPLE VS. LORENZO ​164
PEOPLE OF THE PHILIPPINES vs. RENE BARON ​165
FELIXBERTO A. ABELLANA vs. PEOPLE OF THE
PHILIPPINES and Spouses SAAPIA B. ALONTO and DIAGA
ALONTO ​167
PEOPLE vs. ASIS ​169
RODOLFO BASILONIA vs. HON. DELANO F.
VILLARUZ ​172
ARMILYN MORILLO vs. PEOPLE OF THE PHILIPPINES
AND RICHARD NATIVIDAD ​174
MUNIB S. ESTINO and ERNESTO G. PESCADERA vs .
PEOPLE OF THE PHILIPPINES ​176
BRIONES vs. PEOPLE OF THE PHILIPPINES ​178
SALUDAGA VS SANDIGANBAYAN ​180
LUMANOG vs. PEOPLE OF THE PHILIPPINES ​182
Payumo v. Sandiganbayan ​183
MACAPAGAL vs. PEOPLE OF THE PHILIPPINES ​185
PEOPLE vs MORALES ​187
QUIDET vs PEOPLE ​188
BALABA VS. PEOPLE ​189
PEOPLE vs. OLIVO ​191
GUASCH vs DELA CRUZ ​193
PEOPLE VS. TARUC ​195
PEOPLE vs. TIU ​196
COLINARES vs. PEOPLE ​199
VILLAREAL vs PEOPLE ​200
DANDY L. DUNGO and GREGORIO A. SIBAL, JR vs.
PEOPLE OF THE PHILIPPINES ​202
MANANSALA vs PEOPLE ​204
ABRAHAM MICLAT, JR. y CERBO vs. PEOPLE OF THE
PHILIPPINES ​206
PEOPLE OF THE PHILIPPINES vs. BELEN
MARIACOS ​208
PEOPLE vs. TUAN ​210
ESQUILLO vs PEOPLE ​212
SPOUSES JOEL AND MARIETTA MARIMLA vs. PEOPLE OF
THE PHILIPPINES ​214
PEOPLE OF THE PHILIPPINES vs. JERRY PUNZALAN AND
PATRICIA PUNZALAN ​217

Case 1 – Cajurao
GARCIA vs SANDIGANBAYAN
603 SCRA

Doctrine:

The Sandiganbayan has jurisdiction over actions for forfeiture under Republic Act No. 1379, albeit the
proceeding thereunder is civil in nature—the civil liability for forfeiture cases does not arise from the
commission of a criminal offense.

Facts:

To recover unlawfully acquired funds and properties that the Garcias’ had allegedly amassed and
acquired, the Republic, through the OMB filed with the SB 2 petitions for the forfeiture of those
properties. The Garcias’ filed motion to dismiss on the ground of SB’s lack of jurisdiction for lack of
proper and valid service of summons: (1) Forfeiture I – the corresponding summons on the case were all
issued, and all served on Gen. Garcia at his place of detention; (2) Forfeiture II – the sheriff stated giving
the copies of the summons to the OIC/Custodian of the PNP Detention Center who in turn handed them
to Gen. Garcia. The general signed his receipt of the summons, but as to those pertaining to the other
respondents, Gen. Garcia acknowledged receiving the same, but with the following qualifying note: Im
receiving the copies of Clarita, Ian Carl, Juan Paolo & Timothy but these copies will not guarantee it
being served to the above-named.

Issues/s:

Whether the Sandiganbayan acquired jurisdiction on the person of Clarita Garcia and her three
sons, considering that summons was improperly served and that the plunder case (Crim. Case No.
28107) has already been filed and pending with another division of the SB, i.e., Second Division of the
SB.

Ruling:

Forfeiture Cases and the Plunder Case Have Separate Causes of Action; the Former Is Civil in
Nature while the latter Is Criminal a forfeiture case under RA 1379 arises out of a cause of action
separate and different from a plunder case, thus negating the notion that the crime of plunder charged in
Crim. Case No. 28107 absorbs the forfeiture cases. In a prosecution for plunder, what is sought to be
established is the commission of the criminal acts in furtherance of the acquisition of ill-gotten wealth.
In the language of Sec. 4 of RA 7080, for purposes of establishing the crime of plunder, it is sufficient to
establish beyond reasonable doubt a pattern of overt or criminal acts indicative of the overall unlawful
scheme or conspiracy to amass, accumulate or acquire ill-gotten wealth]. On the other hand, all that the
court needs to determine, by preponderance of evidence, under RA 1379 is the disproportion of
respondents properties to his legitimate income, it being unnecessary to prove how he acquired said
properties. As correctly formulated by the Solicitor General, the forfeitable nature of the properties
under the provisions of RA 1379 does not proceed from a determination of a specific overt act
committed by the respondent public officer leading to the acquisition of the illegal wealth. On the issue
of lack of jurisdiction, petitioner argues that the SB did not acquire jurisdiction over her person and that
of her children due to a defective substituted service of summons. There is merit in petitioners
contention. Sec. 7, Rule 14 of the 1997 Revised Rules of Civil Procedure clearly provides for the
requirements of a valid substituted service of summons, thus:

SEC. 7. Substituted service.If the defendant cannot be served within a reasonable time as
provided in the preceding section [personal service on defendant], service may be effected (a) by
leaving copies of the summons at the defendants residence with some person of suitable age and
discretion then residing therein, or (b) by leaving the copies at defendants office or regular place
of business with some competent person in charge thereof.

It is basic that a court must acquire jurisdiction over a party for the latter to be bound by its decision or
orders. Valid service of summons, by whatever mode authorized by and proper under the Rules, is the
means by which a court acquires jurisdiction over a person.

In the instant case, it is undisputed that summons for Forfeitures I and II were served personally on Maj.
Gen. Carlos Flores Garcia, who is detained at the PNP Detention Center, who acknowledged receipt
thereof by affixing his signature. It is also undisputed that substituted service of summons for both
Forfeitures I and II were made on petitioner and her children through Maj. Gen. Garcia at the PNP
Detention Center. However, such substituted services of summons were invalid for being irregular and
defective.

Requirements as laid down in Manotoc vs CA


1. Impossibility of prompt personal service, i.e., the party relying on substituted service or the sheriff
must show that defendant cannot be served promptly or there is impossibility of prompt service within a
reasonable time. Reasonable time being “so much time as is necessary under the circumstances for a
reasonably prudent and diligent man to do, conveniently, what the contract or duty requires that should
be done, having a regard for the rights and possibility of loss, if any [,] to the other party.” Moreover, we
indicated therein that the sheriff must show several attempts for personal service of at least three (3)
times on at least two (2) different dates.

2. Specific details in the return, i.e., the sheriff must describe in the Return of Summons the facts and
circumstances
surrounding the attempted personal service.

3. Substituted service effected on a person of suitable age and discretion residing at defendant’s house or
residence; or on a competent person in charge of defendant’s office or regular place of business.
Case 2 – Masungsong

ARNEL ESCOBAL vs. HON. FRANCIS GARCHITORENA


G.R. No. 124644 February 5, 2004
Doctrine:
Jurisdiction of the Sandiganbayan vis-à-vis of the RTC. Petition for certiorari with a prayer for the
issuance of a temporary restraining order and preliminary injunction

Facts:
Petitioner Escobal is a graduate of the PMA, a member of the AFP and the Philippine
Constabulary, as well as the Intelligence Group of the Philippine National Police. On March 16, 1990,
the petitioner was conducting surveillance operations on drug trafficking at a café bar and restaurant in
Naga City when he somehow got involved with a shooting incident that resulted to the death of Rodney
Nueca.
Escobal was preventively suspended from the service. When arraigned, he pleaded not guilty.
Thereafter, he filed a Motion to Quash the Information alleging that the court martial, not the RTC, had
jurisdiction over criminal cases involving PNP members and officers. RTC denied the motion.
Trial proceeded. The prosecution rested its case and petitioner presented his evidence. On July
20, 1994, the petitioner filed a Motion to Dismiss the case. Citing Republic of the Philippines v.
Asuncion, et al., he argued that since he committed the crime in the performance of his duties, the
Sandiganbayan had exclusive jurisdiction over the case. The RTC dismissed the motion but ordered the
conduct of a preliminary hearing to determine whether or not the crime charged was committed by the
petitioner in relation to his office as a member of the PNP.
On July 31, 1995, the trial court issued an Order declaring that the petitioner committed the
crime charged while not in the performance of his official function. The trial court added that
nonetheless, upon the enactment of R.A. No. 7975, the issue had become moot and academic since the
amendatory law transferred the jurisdiction over the offense charged from the Sandiganbayan to the
RTC. The petitioner did not have a salary grade of "27" as provided for in or by Section 4(a)(1), (3)
thereof.

The trial court nevertheless ordered the prosecution to amend the Information pursuant to the
ruling in Republic v. Asuncion and R.A. No. 7975, and to include therein an allegation that the offense
charged was not committed by the petitioner in the performance of his duties/functions, nor in relation to
his office.
The petitioner filed a MR of the said order, reiterating that based on his testimony and those of
his witnesses, the offense charged was committed by him in relation to his official functions. He asserted
that R.A. No. 7975, which was enacted on March 30, 1995, could not be applied retroactively.
The RTC ordered the public prosecutor to file a Re-Amended Information and to allege that the
offense charged was committed by the petitioner in the performance of his duties/functions or in relation
to his office; and, conformably to R.A. No. 7975, to thereafter transmit the same to the Sandiganbayan.
The Sandiganbayan returned the records of the case to the RTC, contending that the latter has
jurisdiction over the case.
Issue:
Whether the case falls in the jurisdiction of the Sandiganbayan or of the RTC
Ruling:
The case is within the jurisdiction of the RTC.
Under Section 4(a) of P.D. No. 1606 as amended by P.D. No. 1861, the Sandiganbayan had
exclusive jurisdiction in all cases involving the following:
(1) Violations of Republic Act No. 3019, as amended, otherwise known as the Anti-Graft and Corrupt
Practices Act, Republic Act No. 1379, and Chapter II, Section 2, Title VII of the Revised Penal Code;
(2) Other offenses or felonies committed by public officers and employees in relation to their office,
including those employed in government-owned or controlled corporations, whether simple or
complexed with other crimes, where the penalty prescribed by law is higher than prision correccional or
imprisonment for six (6) years, or a fine of P6,000.00 ….
For the Sandiganbayan to have exclusive jurisdiction under the said law over crimes committed
by public officers in relation to their office, it is essential that the facts showing the intimate relation
between the office of the offender and the discharge of official duties must be alleged in the Information.
It is not enough to merely allege in the Information that the crime charged was committed by the
offender in relation to his office because that would be a conclusion of law. The amended Information
filed with the RTC against the petitioner does not contain any allegation showing the intimate relation
between his office and the discharge of his duties. Hence, the RTC had jurisdiction over the offense
charged when on November 24, 1995, it ordered the re-amendment of the Information to include therein
an allegation that the petitioner committed the crime in relation to office. The trial court erred when it
ordered the elevation of the records to the Sandiganbayan. It bears stressing that R.A. No. 7975
amending P.D. No. 1606 was already in effect.
Under Sec. 2 of said law, even if the offender committed the crime charged in relation to his
office but occupies a position corresponding to a salary grade below "27," the proper Regional Trial
Court or Municipal Trial Court, as the case may be, shall have exclusive jurisdiction over the case. In
this case, the petitioner was a Police Senior Inspector, with salary grade "23." He was charged with
homicide punishable by reclusion temporal. Hence, the RTC had exclusive jurisdiction over the crime
charged conformably to Sections 20 and 32 of Batas Pambansa Blg. 129, as amended by Section 2 of
R.A. No. 7691.
The petitioner’s contention that R.A. No. 7975 should not be applied retroactively has no legal
basis. It bears stressing that R.A. No. 7975 is a substantive procedural law, which may be applied
retroactively.
Case 3 – Fetilo
PEOPLE OF THE PHILIPPINES vs. HENRY T. GO
G.R. No. 168539. March 25, 2014
Doctrine:

Jurisdiction. The rule is well-settled that the act of an accused in posting bail or in filing
motions seeking affirmative relief is tantamount to submission of his person to the jurisdiction of the
court. Thus, it has been held that: When a defendant in a criminal case is brought before a competent
court by virtue of a warrant of arrest or otherwise, in order to avoid the submission of his body to the
jurisdiction of the court, he must raise the question of the court’s jurisdiction over his person at the very
earliest opportunity. If he gives bail, demurs to the complaint or files any dilatory plea or pleads to the
merits, he thereby gives the court jurisdiction over his person. (State ex. rel. John Brown vs. Fitzgerald,
51 Minn., 534)

The Sandiganbayan is a special criminal court which has exclusive original jurisdiction in all
cases involving violations of Republic Act (R.A.) 3019 committed by certain public officers, as
enumerated in Presidential Decree (P.D.) 1606 as amended by R.A. 8249. This includes private
individuals who are charged as co-principals, accomplices or accessories with the said public officers.

Facts:
On or about July 12, 1997, or sometime prior or subsequent thereto, in Pasay City, Metro Manila,
Philippines and within the jurisdiction of this Honorable Court, the late ARTURO ENRILE, then
Secretary of the Department of Transportation and Communications (DOTC), committing the offense in
relation to his office and taking advantage of the same, in conspiracy with accused, HENRY T. GO,
Chairman and President of the Philippine International Air Terminals, Co., Inc. (PIATCO), did then and
there, willfully, unlawfully and criminally enter into a Concession Agreement, after the project for the

construction of the Ninoy Aquino International Airport International Passenger Terminal III (NAIA IPT
III) was awarded to Paircargo Consortium/PIATCO, which Concession Agreement substantially
amended the draft Concession Agreement covering the construction of the NAIA IPT III under Republic
Act 6957, as amended by Republic Act 7718 (BOT law), specifically the provision on Public Utility
Revenues, as well as the assumption by the government of the liabilities of PIATCO in the event of the
latter's default under Article IV, Section 4.04 (b) and (c) in relation to Article 1.06 of the Concession
Agreement, which terms are more beneficial to PIATCO while manifestly and grossly disadvantageous
to the government of the Republic of the Philippines.

Issue:
Whether the Sandiganbayan has jurisdiction over the person of Mr. Henry T. Go.

Ruling:

Yes. The Sandiganbayan has jurisdiction over the person of Mr. Henry T. Go. As alleged in the
Information filed against Mr. Go, which is deemed hypothetically admitted in his Motion to Quash, the
latter conspired with Secretary Enrile in violating Section 3 (g) of R.A. 3019 and that in conspiracy, the
act of one is the act of all. Hence, the criminal liability incurred by a co-conspirator is also incurred by
the other co-conspirators.
It is true that by reason of Secretary Enrile's death, there is no longer any public officer with whom
respondent can be charged for violation of R.A. 3019. It does not mean, however, that the allegation of
conspiracy between them can no longer be proved or that their alleged conspiracy is already expunged.
The only thing extinguished by the death of Secretary Enrile is his criminal liability. His death did not
extinguish the crime nor did it remove the basis of the charge of conspiracy between him and private
respondent. In fact, the Office of the Deputy Ombudsman for Luzon found probable cause to indict
Secretary Enrile for infringement of Sections 3 (e) and (g) of R.A. 3019. Were it not for his death, he
should have been charged. The death of one of two or more conspirators does not prevent the conviction
of the survivor or survivors.

Mr. Go’s act of posting bail and filing his Motion for Consolidation vests the SB with
jurisdiction over his person. The rule is well settled that the act of an accused in posting bail or in filing
motions seeking affirmative relief is tantamount to submission of his person to the jurisdiction of the
court. If he gives bail, demurs to the complaint or files any dilatory plea or pleads to the merits, he
thereby gives the court jurisdiction over his person. Verily, Mr. Go’s participation in the proceedings
before the Sandiganbayan was not confined to his opposition to the issuance of a warrant of arrest but
also covered other matters which called for respondent court’s exercise of its jurisdiction. Mr. Go may
not be heard now to deny said court’s jurisdiction over him. However, by reason of the death of
Secretary Enrile, this can no longer be done. Nonetheless, it does not follow that the Sandiganbayan is
already divested of its jurisdiction over the person of and the case involving herein respondent. To rule
otherwise would mean that the power of a court to decide a case would no longer be based on the law
defining its jurisdiction but on other factors, such as the death of one of the alleged offenders.
Case 4 – Garcia
RAMISCAL vs. SANDIGANBAYAN
630 SCRA
Doctrine:
The question of the number of criminal charges that must be instituted against a criminal
respondent (whether one count or multiple counts of the same offense) is one addressed to the sound
discretion of the prosecution service.
Facts:
In 1998, the Senate Committees on Accountability of Public Officers and Investigation(Blue
Ribbon) and on National Defense and Security (collectively, Senate Blue RibbonCommittee) carried out
an extensive joint inquiry into the "coup rumors and the alleged anomalies" in the Armed Forces of the
Philippines-Philippine Retirement Benefits Systems (AFP-RSBS). In its Report, the Senate Blue Ribbon
Committee outlined, among others, the anomalies in the acquisition of lots in Tanauan, Batangas,
Calamba, Laguna and Iloilo City by the AFP-RSBS, and described the modus operandi of
the perpetrators as follows: The modus operandi in the buying of the lots was to cover the same
transactions with two deeds of sale. One deed of sale would be signed only by the seller or sellers
(unilateral deed).Another deed of sale would be signed by the seller or sellers and the buyer, AFP-RSBS
(bilateral deed). These Unilateral Deeds of Sale recorded lower consideration paid by the System to the
buyer(s) than those stated in the Bilateral Deeds. The motivation was obviously to evade payment of the
correct taxes to the government and save money for the seller(s), broker(s) andwho knows, probably
even for the kickbacks going to certain officials of RSBS, the buyer. Pursuant to the recommendation of
the Senate Blue Ribbon Committee to "prosecute and/or cause the prosecution of Gen. Jose Ramiscal Jr.
(Ret), past AFP-RSBS President, who had signed the unregistered deeds of sale covering the acquisition
of certain parcels of land," Ombudsman Investigators conducted a fact-finding investigation. They
executed a Joint Affidavit-Complaint, stating that based on their findings, B/Gen. Jose Ramiscal, Jr.,
among others, may be charged with falsification of public documents and violation of Section 3(e)
and (g) of Republic Act (R.A.)No. 3019.
Issue:
Whether petitioner may be charged and prosecuted for all five (5) counts of estafa thru
falsification of public documents.

Ruling:
Yes. It is enough, as this Court has already ruled, that the informations filed in these cases
are based on facts establishing probable cause for the offenses charged. This Court will not compel the
Office of the Ombudsman to file only one information for Estafa through Falsification of Public
Documents when its preliminary investigation established the commission of several counts thereof
as such action on the part of this Court would constitute undue interference with the Office of the
Ombudsman's control over the prosecution of these cases. In the second place, this Court is not
persuaded that what is involved in these cases is a continuous crime, that is to say, a single crime
consisting of a series of acts arising from a single criminal resolution or intent not susceptible of
division, with each act in that series being merely the partial execution of a single delict.

Case 5- Lasam
PEOPLE OF THE PHILIPPINES vs. ALFREDO L. BENIPAYO
G.R. No. 154473 April 4, 2009

Doctrines:
1. The jurisdiction of the court to hear and decide a case is conferred by the law in force at the time
of the institution of the action, unless a latter statute provides for a retroactive application
thereof.

2. The grant to the Sandiganbayan of jurisdiction over offenses committed in relation to (public)
office, similar to the expansion of the jurisdiction of the Municipal Trial Court (MTCs), did not
divest the Regional Trial Court (RTC) of its exclusive and original jurisdiction to try written
defamation cases regardless of whether the offense is committed in relation to office.

Facts:
Respondent Alfredo L. Benipayo, then Chairman of the Commission on Elections (COMELEC),
was charged with libel before the Office of the City Prosecutor by Photokina Marketing Corporation,
which felt alluded to in a speech made by Alfredo before the Bahay Kalinaw, University of the
Philippines, and in an television interview before “Point Blank”, a show hosted by Ces Orena-Drilon at
ANC. The Office of the City Prosecutor, in both instances, filed Informations for libel before the
Regional Trial Court. In both instances, Alfredo moved for the dismissal of the case, considering that at
the time he made the alleged utterances, he was an impeachable officer and the same was made in
relation to his duties, therefore, even assuming that he can be charged with libel, the same should be
lodged with the Sandiganbayan. In both instances, the RTC ruled in his favor, thus Photokina elevated
the case to the Supreme Court.
Issue:
Whether or not the utterances were made in relation to his office; and that the RTC had no
jurisdiction over the case?
Ruling:
“Libel cases shall be tried by the regional trial courts having jurisdiction over them to the
exclusion of the metropolitan trial courts, municipal trial courts in cities, municipal trial courts and
municipal circuit trial courts.”
As held in Jalandoni, Bocobo, People v. Metropolitan Trial Court of Quezon City, Br. 32,
Manzano, and analogous cases, we must, in the same way, declare herein that the law, as it still stands at
present, dictates that criminal and civil actions for damages in cases of written defamations shall be filed
simultaneously or separately with the RTC to the exclusion of all other courts. A subsequent enactment
of a law defining the jurisdiction of other courts cannot simply override, in the absence of an express
repeal or modification, the specific provision in the RPC vesting in the RTC, as aforesaid, jurisdiction
over defamations in writing or by similar means. The grant to the Sandiganbayan of jurisdiction over
offenses committed in relation to (public) office, similar to the expansion of the jurisdiction of the
MTCs, did not divest the RTC of its exclusive and original jurisdiction to try written defamation cases
regardless of whether the offense is committed in relation to office. The broad and general phraseology
of Section 4, Presidential Decree No. 1606, as amended by Republic Act No. 8249, cannot be construed
to have impliedly repealed, or even simply modified, such exclusive and original jurisdiction of the
RTC.
Since jurisdiction over written defamations exclusively rests in the RTC without qualification, it
is unnecessary and futile for the parties to argue on whether the crime is committed in relation to office.
Thus, the conclusion reached by the trial court that the respondent committed the alleged libelous acts in
relation to his office as former COMELEC chair, and deprives it of jurisdiction to try the case, is,
following the above disquisition, gross error.
Case 6 – Roxas
LACSON vs EXECUTIVE SECRETARY
G.R. No. 128096 January 20, 1999

Doctrine:

The jurisdiction of a court is defined by the Constitution or statute. The elements of that definition must
appear in the complaint or information so as to ascertain which court has jurisdiction over a case.
Hence the elementary rule that the jurisdiction of a court is determined by the allegations in the
complaint or information, and not by the evidence presented by the parties at the trial. As stated earlier,
the multiple murder charge against petitioner and intervenors falls under Section 4 [paragraph b] of
R.A. 8249. Section 4 requires that the offense charged must be committed by the offender in relation to
his office in order for the Sandiganbayan to have jurisdiction over it. This jurisdictional requirement is
in accordance with Section 5, Article XIII of the 1973 Constitution which mandated that the
Sandiganbayan shall have jurisdiction over criminal cases committed by public officers and employees,
including those in government-owned or controlled corporations, "in relation to their office as may be
determined by law." This constitutional mandate was reiterated in the new [1987] Constitution when it
declared in Section 4 thereof that the Sandiganbayan "shall continue to function and exercise its
jurisdiction as now or hereafter may be provided by law."

Facts:
Petitioner Lacson was involved in a criminal case that started when eleven persons, believed to
be members of the Kuratong Baleleng Gang (KBG) were killed by the Anti-Bank Robbery and
Intelligence Task Group (ABRITG) where the petitioner was one of the heads. Then, in a media expose,
it was said that what happened was a rub-out and not a shoot-out.

The accused filed separate motions questioning the jurisdiction of the Sandiganbayan, asserting
that under the amended informations, the cases fall within the jurisdiction of the Regional Trial Court
pursuant to Section 2 of RA 7975. They contend that the said law limited the jurisdiction of the
Sandiganbayan to cases where one or more of the “principal accused” are governement officials with
Salary Grade 27 or higher, or PNP officials with rank of Chief Superintendent or higher. Thus, they did
not qualify under said requisites. However, pending resolution of their motions, RA 8249 was approved
amending the jurisdiction of the Sandiganbayan by deleting the word “principal” from the phrase
“principal accused” from the said statute.

Issue:
Whether or not the criminal cases against petitioners were committed in relation to the office of
the accused PNP officers which is essential to the determination whether the case falls within the
Sandiganbayan’s or Regional Trial Court’s juridiction.
Ruling:

Yes. The Regional Trial Court has jurisdiction over the case.

In the aforecited case of People vs. Montejo, it is noteworthy that the phrase committed in
relation to public office does not appear in the information, which only signifies that the said phrase is
not what determines the jurisdiction of the Sandiganbayan. What is controlling is the specific factual
allegations in the information that would indicate the close intimacy between the discharge of the
accused’s official duties and the commission of the offense charged, in order to qualify the crime as
having been committed in relation to public office.

Consequently, for failure to show in the amended informations that the charge of murder was
intimately connected with the discharge of official functions of the accused PNP officers, the offense
charged in the subject criminal cases is plain murder and, therefore, within the exclusive original
jurisdiction of the Regional Trial Court, not the Sandiganbayan.
Case 7 – Lukban
SANCHEZ vs. DEMETRIOU
227 SCRA 627
Doctrine:
Remedial Law; Criminal Procedure; Preliminary Investigation; the absence of preliminary investigation
does not impair the validity of the information or otherwise render the same defective and neither does it
affect the jurisdiction of the court over the case or constitute a ground for quashing the information.
The petitioner was present at that hearing and he never disowned Atty. Panelo as his counsel. During
the entire proceedings, he remained quiet and let this counsel speak and argue on his behalf. It was only
in his tardy Reply that he has suddenly bestirred himself and would now question his representation by
this lawyer as unauthorized and inofficious. Section 3, Paragraph (d), Rule 112 of the Rules of Court,
provides that if the respondent cannot be subpoenaed or, if subpoenaed, does not submit counter-
affidavits, the investigating officer shall base his resolution on the evidence presented by the
complainant. Just as the accused may renounce the right to be present at the preliminary investigation,
so may he waive the right to present counter-affidavits or any other evidence in his defense. At any rate,
it is settled that the absence of a preliminary investigation does not impair the validity of the information
or otherwise render the same defective and neither does it affect the jurisdiction of the court over the
case or constitute a ground for quashing the information.If no preliminary investigation has been held,
or if it is flawed, the trial court may, on motion of the accused, order an investigation or reinvestigation
and hold the proceedings in the criminal case in abeyance. In the case at bar, however, the respondent
judge saw no reason or need for such a step. Finding no arbitrariness in her factual conclusions, we
shall defer to her judgment. -
Same; Same; Same; Ombudsman; The Ombudsman’s power under Sec. 15 paragraph (1) of RA 6770 is
not an exclusive authority but rather a shared or concurrent authority in respect of the offense charged.
The ombudsman is indeed empowered under Section 15 paragraph (1) of RA 6770 to investigate and
prosecute any illegal act or omission of any public official. However as we held only two years ago in
the case of Aguinaldo v. Domagas, this authority "is not an exclusive authority but rather a shared or
concurrent authority in. respect of the offense charged." Petitioners finally assert that the information
and amended information filed in this case needed the approval of the Ombudsman. It is not disputed
that the information and amended information here did not have the approval of the Ombudsman.
However, we do not believe that such approval was necessary at all. In Deloso v. Domingo, 191 SCRA.
545 (1990), the Court held that the Ombudsman has authority to investigate charges of illegal or
omissions on the part of any public official, i.e., any crime imputed to a public official. It must, however,
be pointed out that the authority of the Ombudsman to investigate "any [illegal] act or omission of any
public official" (191 SCRA at 550) is not an exclusive authority but rather a shared or concurrent
authority in respect of the offense here charged, i.e., the crime of sedition. Thus, the non-involvement of
the office of the Ombudsman in the present case does not have any adverse legal consequence upon the
authority the panel of prosecutors to file and prosecute the information or amended information.
Same; Same; Same; Arrest; Application of actual force, manual touching of the body, physical restraint
or a formal declaration of arrest is not required. It is enough that there be an intent on the part of one of
the parties to arrest the other and an intent on the part of the other to submit, under the belief and
impression that submission is necessary.
- "Arrest" is defined under Section 1, Rule 113 of the Rules of Court as the taking of a person into
custody in order that he may be bound to answer for the commission of an offense. Under Section 2 of
the same Rule, an arrest is effected by an actual restraint of the person to be arrested or by his voluntary
submission to the custody of the person making the arrest. Application of actual force, manual touching
of the body, physical restraint or a formal declaration of arrest is not, required. It is enough that there be
an intent on the part of one of the parties to arrest the other and an intent on the part of the other to
submit, under the belief and impression that submission is necessary. The petitioner was taken to Camp
Vicente Lim, Canlubang, Laguna, by virtue of a letter-invitation issued by PNP Commander Rex Piad
requesting him to appear at the said camp for investigation. In Babst v. National Intelligence Board 13
this Court declared: Be that as it may, it is not idle to note that ordinarily, an invitation to attend a
hearing and answer some questions, which the person invited may heed or refuse at his pleasure, is not
illegal or constitutionally objectionable. Under certain circumstances, however, such an invitation can
easily assume a different appearance. Thus, where the invitation comes from a powerful group
composed predominantly of ranking military officers issued at a time when the country has just emerged
from martial rule and when the suspension of the privilege of the writ of habeas corpus has not entirely
been lifted, and the designated interrogation site is a military camp, the same can be easily taken, not as
a strictly voluntary invitation which it purports to be, but as an authoritative command which one can
only defy at his peril. . . . (Emphasis supplied) In the case at bar, the invitation came from a high-
ranking military official and the investigation of Sanchez was to be made at a military camp. Although
in the guise of a request, it was obviously a command or an order of arrest that the petitioner could
hardly he expected to defy. In fact, apparently cowed by the "invitation," he went without protest (and in
informal clothes and slippers only) with the officers who had come to fetch him. It may not be amiss to
observe that under R.A. No. 7438, the requisites of a "custodial investigation" are applicable even to a
person not formally arrested but merely "invited" for questioning. It should likewise be noted that at
Camp Vicente Lim, the petitioner was placed on "arrest status" after he was pointed to by Centeno and
Malabanan as the person who first raped Mary Eileen Sarmenta. Respondent Zuño himself
acknowledged during the August 13, 1993 hearing that, on the basis of the sworn statements of the two
state witnesses, petitioner had been "arrested."
Same; Same; Same; Jurisdiction over the person of the accused; Motion to quash; Case at bar; Where
the accused objects to the jurisdiction of the Court over his person, he may move to quash the
information but only on that ground. If he raises other grounds in the motion to quash, he is deemed to
have waived that the objection and to have submitted his person to the jurisdiction of the court.
The original warrantless arrest of the petitioner was doubtless illegal. Nevertheless, the Regional Trial
Court lawfully acquired jurisdiction over the person of the petitioner by virtue of the warrant of arrest it
issued on August 26, 1993 against him and the other accused in connection with the rape-slay cases. It
was belated, to be sure, but it was nonetheless legal. Even on the assumption that no warrant was issued
at all, we find that the trial court still lawfully acquired jurisdiction over the person of the petitioner. The
rule is that if the accused objects to the jurisdiction of the court over his person, he may move to quash
the information, but only on that ground. If, as in this case, the accused raises other grounds in the
motion to quash, he is deemed to have waived that objection and to have submitted his person to the
jurisdiction of that court. The Court notes that on August 13, 1993, after the petitioner was unlawfully
arrested, Judge Lanzanas issued a warrant of arrest against Antonio L. Sanchez in connection with
Criminal Cases Nos. 93-124634 to 93-124637 for violation of R.A No. 6713. Pending the issuance of the
warrant of arrest for the rape-slay cases, this first warrant served as the initial justification for his
detention. The Court also adverts to its uniform ruling that the filing of charges, and the issuance of the
corresponding warrant of arrest, against a person invalidly detained will cure the defect of that
detention or at least deny him the right to be released because of such defect. * Applicable by analogy to
the case at bar is Rule 102 Section 4 of the Rules of Court that: Sec, 4. When writ is not allowed or
discharge authorized. — If it appears that the person alleged to be restrained of his liberty is in the
custody of an officer under process issued by a court or judge or by virtue of a judgment or order of a
court of record, and that the court or judge had jurisdiction to issue the process, render the judgment, or
make the order, the writ shall not be allowed; or if the jurisdiction appears after the writ is allowed, the
person shall not be discharged by reason of any informality or defect in the process, judgment, or order.
Nor shall, anything in this rule be held to authorize the discharge of a person charged with or convicted
of an offense in the Philippines or of a person suffering imprisonment under lawful judgment.
Same; Same; Prosecutor; The prosecutor cannot be compelled to include in the information a person
against whom he believes no sufficient evidence of guilt exists.
While the prosecuting officer is required by law to charge all those who in his opinion, appear to be
guilty, he nevertheless cannot be compelled to include in the information a person against whom he
believes no sufficient evidence of guilt exists. The appreciation of the evidence involves the use of
discretion on the part of the prosecutor, and we do not find in the case at bar a clear showing by the
petitioner of a grave abuse of such discretion. The decision of the prosecutor may be reversed or
modified by the Secretary of Justice or in special cases by the President of the Philippines. But even this
Court cannot order the prosecution of a person against whom the prosecutor does not find sufficient
evidence to support at least a prima facie case. The courts try and absolve or convict the accused but as
a rule have no part in the initial decision to prosecute him. The possible exception is where there is an
unmistakable showing of a grave abuse of discretion that will justify judicial intrusion into the precincts
of the executive. But in such a case the proper remedy to call for such exception is a petition for
mandamus, not certiorari or prohibition. Moreover, before resorting to this relief, the party seeking the
inclusion of another person as a co-accused in the same case must first avail itself of other adequate
remedies such as the filing of a motion for such inclusion.
Facts:
A warrant of arrest was served to Mayor Antonio L. Sanchez of Calauan, Laguna in connection
with the rape-slay of Mary Eileen Sarmenta and the killing of Allan Gomez.
The respondent Secretary of Justice expressed his apprehension that the trial of the said cases
might result in a miscarriage of justice because of the tense and partisan atmosphere in Laguna in favor
of the petitioner and the relationship of an employee in the trial court with one of the accused. Thus, the
court ordered the transfer of the venue of the seven cases to Pasig, Metro Manila where they were raffled
to respondent Judge Harriet Demetriou.
Issue:
Whether or not as a public officer, he can be tried for the offense only by the Sandiganbayan.
Ruling:
No. Section 4, paragraph (a) of P.D. No, 1606, as amended by P.D. No.1861, provides:
The Sandiganbayan shall exercise:
a) Exclusive original jurisdiction in all cases involving:

1. (1) Violations of Republic Act No. 3019, as amended, otherwise known as the Anti-Graft and
Corrupt Practices Act, Republic Act No. 1379, and Chapter II, Section 2, Title VII of the Revised
Penal Code:

2. (2) Other offenses or felonies committed by public officers and employees in relation to their
office, including those employed in government-owned or controlled corporations, whether
simple or complexed with other crimes, where the penalty prescribed by law is higher
thanprision correccional or imprisonment for six (6) years, or a fine of P6,000.00. . . .
The crime of rape with homicide with which the petitioner stands charged obviously does not fall
under paragraph (1), which deals with graft and corruption cases. Neither is it covered by paragraph (2)
because it is not an offense committed in relation to the office of the petitioner.
In the case at bar, there is no allegation that the crime of rape with homicide imputed to the
petitioner was connected with the discharge of his functions as municipal mayor or that there is an
"intimate connection" between the offense and his office. It follows that the said crime, being an
ordinary offense, is triable by the regular courts and not the Sandiganbayan

Case 8 – Marcos
SERANA vs. SANDIGANBAYAN
G.R. No. 162059 January 22, 2008

Doctrine:

1. Section 4(8) of PD. No. 1606 which defines the jurisdiction of the Sandiganbayan reads.
"Other offenses or felonies whether simple or complexed with other crimes committed by the
public officials and employees mentioned in subsection (a) of this section in relation to their
office."

2. While the first part of Section 4(A) covers only officials with Salary Grade 27 and higher, its
​ second part specifically includes other executive offiCials whose positions may not be of
Salary
​ Grade 27 and higher but who are by express provision of law placed under the jurisdiction of
​ the said court.

Facts:
Hannah Serana was appointed by former President Estrada as a student regent of UP Cebu, to
serve a one-year term. President Estrada gave P15,000,000.00 to the Office of the Student Regent
Foundation, Inc as financial assistance for the proposed renovation. The renovation of Vinzons Hall
Annex failed to materialize. The Ombudsman filed estafa case against her before the Sandiganbayan.
She moved to quash the information. She claimed that the Sandiganbayan does not have any jurisdiction
over the offense charged or over her person, in her capacity as UP student regent because the
Sandiganbayan has no jurisdiction over estafa; the petitioner is not a public officer with Salary Grade
27; the offense charged was not committed in relation to her office; and the funds in question personally
came from President Estrada, not from the government. As to jurisdiction over her person, she contends
that as a UP student regent, she is not a public officer who held the position in an ex officio capacity.
The Sandiganbayan denied her motion for lack of merit.

Issue:
Whether or not the Sandiganbayan has no jurisdiction over Serana’s case.

Ruling:
No, Sandiganbayan has jurisdiction over this case. In Geduspan v. People, the SC held that while
the first part of Sec. 4(A) covers only officials with Salary grade 27 and higher but who are by express
provisions of law placed under the jurisdiction of the Sandiganbayan as she is placed there by express
provisions of law. Sec. 4(A)(1)(g) of PD No. 1606 explicitly vested the Sandiganbayan with jurisdiction
over Presidents, directors and trustees, or manager of government-owned or controlled corporations,
state universities, or educational foundations. Petitioner falls under this category. As the Sandiganbayan
pointed out, the Board of Regents performs functions similar to those of a board of trustee of a non-
stock corporation. By express mandate of law, petitioner is, indeed, a public officer as contemplated by
PD No. 1606. Thus, her position as a board of regent (UP student regent) is among those enumerated
and the Sandiganbayan has jurisdiction over her.
Case 9 – Remigio

DISINI vs SECRETARY OF JUSTICE


G.R. no. 203335 February 18, 2014
Doctrine:
Cybersecurity refers to the collection of tools, policies, risk management approaches, actions, training,
best practices, assurance and technologies that can be used to protect cyber environment and
organization and user’s assets. Any law that overreaching the terms provided are unconstitutional on the
basis of violation of rights.

Facts:
The petitioners filed a complaint for the constitutionality of certain provisions of RA10175. The
said statutes violates the constitution rights to privacy, freedom of information, and freedom of
communication for any information that is contained within the use in the web can be used as grounds to
detain any person. The practice can be used as a fishing expedition.

Issue:
Whether or not the certain provisions of RA10175 are constitutional.

Ruling:
No, not all provision are constitutional for it breaches cyber security. Cybersecurity refers to the
collection of tools, policies, risk management approaches, actions, training, best practices, assurance and
technologies that can be used to protect cyber environment and organization and user’s assets. The law
extended it's reach to collect private data from devices which is considered as a fishing expedition.

Case 10- Mortel


JIMENEZ vs. SORONGON
G.R. No. 178607 December 5, 2012

Doctrine:
In appeals of criminal cases before the CA or SC, the OSG is the appellate counsel of the People.
Facts:
The petitioner is the president of Unlad Shipping & Management Corp., a local manning agency,
Antzoulatos, Alamil, Gaza, and Avgoustis (respondents) are some of the listed incorporators of Tsakos
Maritime Services, Inc. (TMSI), another local manning agency.
On August 2003, petitioner filed a complaint-affidavit with the Office of the City Prosecutor of
Mandaluyong City against respondents for syndicated and large-scale illegal recruitment. Respondents
Antzoulatos and Gaza filed their joint counter-affidavit denying the complaint-affidavit’s allegations.
The City Prosecutor filed the corresponding criminal information with the RTC of Mandaluyong City.
Subsequently, the City Prosecutor filed a motion to withdraw the information, but the RTC denied it as it
found the existence of probable cause to hold the respondents for trial. Thus, the RTC ordered the
issuance of warrants of arrest against respondents. Alamil moved for reconsideration and for the
inhibition of Judge Capco-Umali. In a January 2006 order, Judge Umali voluntary inhibited herself from
the case. Then, the case was later re-raffled, which is presided by Judge Sorongon.
In a ruling, the RTC dismissed the case, and set aside the earlier issued warrants of arrest. On
May 2006, petitioner filed a notice of appeal, however, the RTC denied it since the petitioner filed it
without the conformity of the Solicitor General, who is mandated to represent the People of the Phils. in
criminal actions appealed to the CA. The petitioner elevated his case to the CA via Rule 65 petition for
certiorari assailing the RTCs orders. In its resolution, the CA dismissed outright the petition for lack of
legal personality to file on behalf of the People of the Phils.
Issue: ​
Whether the CA committed a reversible error in dismissing outright the petition for certiorari for
lack of legal personality to file on behalf of the People of the Phils.

Ruling: ​
No. It is well-settled that “every action must be prosecuted or defended in the name of the real
party in interest.” When the plaintiff or the defendant is not a real party in interest, the suit is
dismissible.
Procedural law basically mandates that “all criminal actions commenced by complaint or by
information shall be prosecuted under the direction and control of a public prosecutor.” In appeals of
criminal cases before the CA and before this Court, the OSG is the appellate counsel of the People. The
People is the real party in interest in criminal cases.
In this case, the petitioner has no legal personality to assail the dismissal of the criminal case
since the main issue raised by petitioner involved the criminal aspect of the case, i.e., the existence of
probable cause. The petitioner did not appeal to protect his alleged pecuniary interest as an offended
party of the crime, but to cause reinstatement of the criminal action against the respondents.

Case 11- Huggins


PEOPLE vs. VALDEZ
GR No. 175602 January 18, 2012
Doctrine:
For the complaint or information to be sufficient, description of the crime charged and the particular
facts must be therein recited in order to inform the accused of the nature of the accusation against him
so as to enable him to suitably prepare his defense.
Facts:
PO2 Eduardo Valdez and Edwin Valdez were convicted of three counts of murder for the killing
of Fernando, Moises, and Joselito Sayson by shooting them with a gun. The information against the
accused explicitly states the following: “That on or about the 1st day of March, 2000, in Quezon City,
Philippines, the above-named accused conspiring together, confederating with and mutually helping
each other, with intent to kill, qualified with treachery, evident premeditation and abuse of superior
strength did, then and there, willfully, unlawfully and feloniously, assault, attack and employ personal
violence upon the person…”. Eduardo Valdez contends, among others, that the State did not establish
the qualifying circumstance of treachery. Hence, the petition.
Issue:
Whether or not the information sufficiently alleged the qualifying circumstance of treachery
Ruling:
No, the information insufficiently alleged the attendance of the qualifying circumstance of
treachery. Treachery is the employment of means, methods, or forms in the execution of any of the
crimes against persons which tend to directly and specially insure its execution, without risk to the
offending party arising from the defense which the offended party might make.
What is controlling is not the title of the complaint, nor the designation of the offense charged or
the particular law or part thereof allegedly violated, these being mere conclusions of law made by the
prosecutor, but the description of the crime charged and the particular facts therein recited. The acts or
omissions complained of must be alleged in such form as is sufficient to enable a person of common
understanding to know what offense is intended to be charged, and enable the court to pronounce proper
judgment. No information for a crime will be sufficient if it does not accurately and clearly allege the
elements of the crime charged.
The averments of the informations to the effect that the two accused “with intent to kill, qualified
with treachery, evident premeditation and abuse of superior strength did assault, attack and employ
personal violence upon” the victims “by then and there shooting them with a gun, hitting them” on
various parts of their bodies “which were the direct and immediate cause of their deaths” did not
sufficiently set forth the facts and circumstances describing how treachery attended each of the killings.
Therefore, the Supreme Court modified CA’s decision in ruling that the accused are guilty of
three counts of homicide and not of murder.
Case 12-Oineza
MIGUEL vs. SANDIGANBAYAN
G.R. No. 172035 July 4, 2012

Doctrine:
Right of the Accused to be informed of the Nature and Cause of the accusation against him.
Facts:
Then Vice Mayor Mercelita M. Lucido and other local officials of Koronadal City, South
Cotabato filed a letter-complaint with the Office of the Ombudsman-Mindanao (Ombudsman) charging
the petitioner with violation of Anti-Graft And Corrupt Practices Acts Republic Act (R.A.) No. 3019.
The Ombudsman found probable cause against the petitioner for violation of R.A. No. 3019 and
Falsification of Public Document; by reason of which, the Ombudsman filed information.
On November 3, 2004, the petitioner was arraigned; he pleaded not guilty in both criminal cases.
On June 27, 2005, the petitioner filed his opposition based on the obvious and fatal defect of the
information in failing to allege that the giving of unwarranted benefits and advantages was done through
manifest partiality, evident bad faith or gross inexcusable negligence.
On January 25, 2006, the Sandiganbayan promulgated the assailed resolution suspending the for
a period of ninety (90) days only.
The petitioner assails the lack of hearing before the issuance of his suspension order. For the
petitioner, the requirement of a pre-suspension hearing can only be satisfied if the Sandiganbayan
ordered an actual hearing to settle the "defect" in the information.
Issue:
Whether the information, charging the petitioner with violation of Section 3(e) of R.A. No. 3019,
is valid?

Ruling:
Yes.
In deference to the constitutional right of an accused to be informed of the nature and the cause
of the accusation against him, Section 6, Rule 110 of the Revised Rules of Criminal Procedure requires
that the information shall state the designation of the offense given by the statute and the acts or
omissions imputed which constitute the offense charged. Additionally, the Rules requires that these acts
or omissions and its attendant circumstances must be stated in ordinary and concise language and in
terms sufficient to enable a person of common understanding to know what offense is being charged and
for the court to pronounce judgment.
The test of the information’s sufficiency is whether the crime is described in intelligible terms
and with such particularity with reasonable certainty so that the accused is duly informed of the offense
charged. In particular, whether an information validly charges an offense depends on whether the
material facts alleged in the complaint or information shall establish the essential elements of the offense
charged as defined in the law. The reason of the requirement in the Rules is to enable the accused to
suitably prepare his defense.
This test was satisfied in this case when the information contained the phrase "acting with
evident bad faith and manifest partiality." A reading of the information clearly reveals that the phrase
"acting with evident bad faith and manifest partiality" was merely a continuation of the prior allegation
of the acts of the petitioner, and that he ultimately acted with evident bad faith and manifest partiality in
giving unwarranted benefits and advantages to his co-accused private individuals.

Case 13-Olaivar
PEOPLE vs SORIA
685 SCRA 483

Doctrine:
Where an offense may be committed in any of the different modes and the offense is alleged to have been
committed in two or more modes specified, the indictment is sufficient, notwithstanding the fact that the
different means of committing the same offense are prohibited by separate sections of the statute. The
allegation in the information of the various ways of committing the offense should be regarded as a
description of only one offense and the information is not thereby rendered defective on the ground of
multifariousness.

Facts:
According to the prosecution, Appellant Benjamin Soria, father of AAA, brought home
merienda. After eating, AAA went to the bedroom to rest. Appellant also entered the room and
positioned himself on top of AAA, took off her clothes and inserted his penis into her vagina. AAA felt
intense pain from her breast down to her vagina and thus told her father that it was painful. At that point,
appellant apologized to his daughter, stood up, and left the room. The whole incident was witnessed by
AAA's brother, BBB. The pain persisted until AAA's vagina started to bleed. She was brought to the
hospital for treatment. AAA was taken into the custody of the DSWD. On the other hand, according to
the defense, appellant admitted that he was at home on the day and time of AAA's alleged rape but
denied committing the same. He claimed that the filing of the rape case against him was instigated by
his wife, whom he confronted about her illicit affair with a man residing in their community. According
to appellant, he could not have molested AAA because he treated her well. AAA was examined, which
yielded to the result that the subject is in virgin state physically, and that there are no external signs of
any form of physical trauma. Appellant pleaded not guilty to the crime charged. the trial court rendered
its Judgment finding appellant guilty beyond reasonable doubt of the crime of rape against “AAA”, his
daughter of minor age, as charged in the Information. It ruled that the lack of tenacious resistance on the
part of “AAA” is immaterial considering that appellant’s moralascendancy and influence over her
substitute for violence and intimidation. Upon appeal the appellate court was convinced that appellant
raped “AAA”, it nevertheless noted the prosecution’s failure to present her birth certificate as competent
proof of her minority. Hence this petition.

Issue:
Whether or not the allegations that caused ambiguity invalidate the Information or result in the
automatic dismissal of the case.
Ruling:
No. The Information in this case did not specify with certainty whether appellant committed the
rape through sexual intercourse under paragraph 1 of Article 266-A, or rape by sexual assault as
described in paragraph 2 thereof. The Information stated that appellant inserted his penis into the genital
of AAA, which constituted rape by sexual intercourse under the first paragraph of Article 266-A. At the
same time, the Information alleged that appellant used force and intimidation to commit an act of sexual
assault. While these allegations cause ambiguity, they only pertain to the mode or manner of how the
rape was committed and the same do not invalidate the Information or result in the automatic dismissal
of the case. Where an offense may be committed in any of the different modes and the offense is alleged
to have been committed in two or more modes specified, the indictment is sufficient, notwithstanding
the fact that the different means of committing the same offense are prohibited by separate sections of
the statute. The allegation in the information of the various ways of committing the offense should be
regarded as a description of only one offense and the information is not thereby rendered defective on
the ground of multifariousness. Any objection from the appellant with respect to the Information is held
to have been waived failing any effort to oppose the same before trial. He therefore can be convicted of
rape through sexual intercourse or rape by sexual assault, depending on the evidence adduced during
trial.
Case 14 – Remigio
UNION BANK vs PEOPLE
667 SCRA 113

Doctrine:
Venue of courts consists of the place where the crime was committed but also includes the essential
ingredients took place.

Facts:
Union Bank filed two complaints with a writ of replevin against spouses Tamondong and an
unknown person in the Branch 47, MeTC, Pasay City, and in the Branch 109, RTC, Pasay City. Desi
Thomas, the private respondent, executed and signed the certification of non-forum shopping in both
complaints. She was charged with violating Article 183 of the RTC for commiting forgeries under oath
in certificates against forum shopping in the second complaint. Thomas filed a motion to quash stating
that the MeTC has no jurisdiction over perjury and that the forgery was not committed without ill intent.
The MeTC of Makati ruled against Thomas for denying the Motion to quash and a Motion for
Reconsideration, and ruled that the crime of forgery can be charged against her. The RTC also ruled in
favor of the MeTC, stating that the court did not commit grave abuse of discretion. The petitioners filed
for a Motion for Reconsideration to the Supreme Court stating that the MeTC is incorrect.

Issue:
Whether or not the Makati city was the proper venue for it was the venue for the notarization of
the certificate of non-forum shopping.

Ruling:
The MeTC has Jurisdiction for the venue of action and of jurisdiction are deemed sufficiently
alleged where the Information states that the offense was committed or some of its essential ingredients
occurred at a place within the territorial jurisdiction of the court. There is a two fold rule which states
that the jurisdiction of trial courts is limited to well-defined territories such that a trial court can only
hear and try cases involving crimes committed within its territorial jurisdiction and laying the venue in
the locus criminis is grounded on the necessity and justice of having an accused on trial in the
municipality of province where witnesses and other facilities for his defense are available. As stated in
in the Revised Rules of Court, the venue and jurisdiction over criminal cases not only in the court where
the offense was committed, but also where any of its essential ingredients took place.

Case 15 – Marcelo
DR. FERNANDO P. SOLIDUM vs. PEOPLE OF THE PHILIPPINES
718 SCRA 263

Doctrine: Not all criminally liable are civilly liable

Facts:
Gerald Albert Gercayo (Gerald) was born on June 2, 1992, then three (3) years old, with an
imperforate anus. Two days after his birth, Gerald underwent colostomy, a surgical procedure to bring
one end of the large intestine out through the abdominal wall, enabling him to excrete through a
colostomy bag attached to the side of his body.
The operation was admitted at the Ospital ng Maynila for a pull-through operation and was
assisted by numbers of doctors including petitioner Dr. Fernando Solidum (Dr. Solidum). During the
operation, Gerald experienced bradycardia, and went into a coma. His coma lasted for two weeks,but he
regained consciousness only after a month. He could no longer see, hear or move. Agitated by her son’s
helpless and unexpected condition, Ma. Luz Gercayo (Luz) filed a complaint for reckless imprudence
resulting in serious physical injuries with the City Prosecutor’s Office of Manila against the attending
physicians, Dr. Solidum. It can be noted that Ospital ng Maynila was not included here as he party.
The Regional Trial Court rendered judgment finding the accused guilty beyond reasonable doubt.
It was affirmed by the CA. Hence, this petition.
ISSUE:
Whether or not Dr. Solidum is still civilly liable despite his acquittal on the case
RULING:
The SC stated that the acquittal of Dr. Solidum would not immediately exempt him from civil
liability. But the SC cannot declare him civilly liable because the circumstances that have been
established here do not present the factual and legal bases for validly doing so. His acquittal did not
derive only from reasonable doubt. There was really no firm and competent showing how the injury to
Gerard had been caused. That meant that the manner of administration of the anesthesia by Dr. Solidum
was not necessarily the cause of the hypoxia that caused the bradycardia experienced by Gerard.
Consequently, to adjudge Dr. Solidum civilly liable would be to speculate on the cause of the hypoxia.
We are not allowed to do so, for civil liability must not rest on speculation but on competent evidence.
In terms of liability of Ospital ng Maynila, Ospital ng Maynila was not at all a party in the
proceedings. Hence, its fundamental right to be heard was not respected from the outset. The RTC and
the CA should have been alert to this fundamental defect. Verily, no person can be prejudiced by a ruling
rendered in an action or proceeding in which he was not made a party. Such a rule would enforce the
constitutional guarantee of due process of law.
Moreover, Ospital ng Maynila could be held civilly liable only when subsidiary liability would
be properly enforceable pursuant to Article 103 of the Revised Penal Code. Firstly, pursuant to Article
103 of the Revised Penal Code, Ospital ng Maynila must be shown to be a corporation "engaged in any
kind of industry," meaning one that employs labor and capital, and is engaged in industry. However,
Ospital ng Maynila, being a public hospital, was not engaged in industry conducted for profit but purely
in charitable and humanitarian work. Secondly, Dr. Solidum was not such employee but a consultant of
the hospital. And, thirdly, assuming that civil liability was adjudged against Dr. Solidum as an employee
(which did not happen here), the execution against him was unsatisfied due to his being insolvent.
Case 16 – Sevilla
CASTILLO vs. SALVADOR
GR No. 191240 July 30, 2014

Doctrine:

Once a criminal action is extinguished by the acquittal of the accused any civil liability claimed by the
offended party can only be awarded if it is based on another cause of action, other than a delict.

Facts:
Petition for review on certiorari which assails the Decision of the Court of Appeals (CA) with
respect only to the civil aspect of the case as respondent Phillip R. Salvador had been acquitted of the
crime of Estafa.

The respondent Phillip R. Salvador was charged with Estafa under Article 315, paragraph 2 (a)
of the Revised Penal Code. While, petitioner Cristina B. Castillo is a businesswoman engaged in real
estate business, educational institution, boutique, and trading business. She was then enticed by Salvador
and his brother, Ramon Salvador to engage in freight and remittance business.

She agreed with respondent and Ramon that any profit derived from the business would be
equally divided among them and that respondent would be in charge of promotion and marketing in
Hong Kong, and Ramon would take charge of the operations of business in the Philippines and she
would be financing the business.

The business has not operated yet as petitioner was still raising the amount of US$100,000.00 as
capital for the actual operation. When petitioner already had the money, she handed the same to
respondent Salvador which was witnessed by her disabled half-brother Enrico B. Tan. However, the
proposed business never operated as respondent only stayed in Hong Kong for three days. When she
asked respondent about the money and the business, the latter told her that the money was deposited in a
bank. However, upon further query, respondent confessed that he used the money to pay for his other
obligations. Since then, the US$100,000.00 was not returned at all.

Petitioner files the instant petition on the civil aspect of the case alleging that even if the Court
Of Appeals decided to acquit him it should have at least retained the award of damages to the petitioner.

Issue:
Whether or not the award of damaged be retained despite the acquittal of the accused in the
criminal case?

Ruling:

The award of damages must be removed. Our law recognizes two kinds of acquittal, with
different effects on the civil liability of the accused. First is an acquittal on the ground that the accused is
not the author of the actor omission complained of. This instance closes the door to civil liability, for a
person who has been found to be not the perpetrator of any act or omission cannot and can never be held
liable for such act or omission. There being no delict, civil liability ex delicto is out of the question, and
the civil action, if any, which may be instituted must be based on grounds other than the delict
complained of. This is the situation contemplated in Rule 111 of the Rules of Court. The second instance
is an acquittal based on reasonable doubt on the guilt of the accused. In this case, even if the guilt of the
accused has not been satisfactorily established, he is not exempt from civil liability which may be
proved by preponderance of evidence only. This is the situation contemplated in Article 29 of the Civil
Code, where the civil action for damages is “for the same act or omission.

A reading of the CA decision would show that respondent was acquitted because the prosecution
failed to prove his guilt beyond reasonable doubt. Said the CA:

The evidence for the prosecution being insufficient to prove beyond reasonable doubt that the
crime as charged had been committed by appellant, the general presumption, “that a person is innocent
of the crime or wrong, stands in his favor. The prosecution failed to prove that all the elements of Estafa
are present in this case as would overcome the presumption of innocence in favor of appellant. For in
fact, the prosecution’s primary witness herself could not even establish clearly and precisely how
appellant committed the alleged fraud. She failed to convince us that she was deceived through
misrepresentations and/or insidious actions, in venturing into a remittance business. Quite the contrary,
the obtaining circumstance in this case indicate the weakness of her submissions.

Thus, since the acquittal is based on reasonable doubt, respondent is not exempt from civil
liability which may be proved by preponderance of evidence only. In Encinas v. National Bookstore,
Inc., the higher court explained the concept of preponderance of evidence as follows:

Preponderance of evidence is the weight, credit, and value of the aggregate evidence on either
side and is usually considered to be synonymous with the term “greater weight of the evidence” or
“greater weight of the credible evidence.” Preponderance of evidence is a phrase which, in the last
analysis, means probability of the truth. It is evidence which is more convincing to the court as worthy
of belief than that which is offered in opposition thereto. However, in this case, no such civil liability is
proved even by preponderance of evidence.

In discrediting petitioner’s allegation that she gave respondent US$100,000.00 in May 2002, the CA
found that: (1) petitioner failed to show how she was able to raise the money in such a short period of
time and even gave conflicting versions on the source of the same; (2) petitioner failed to require
respondent to sign a receipt so she could have a record of the transaction and offered no plausible reason
why the money was allegedly hand-carried to Hong Kong; (3) petitioner’s claim of trust as reason for
not requiring respondent to sign a receipt was inconsistent with the way she conducted her previous
transactions with him; and (4) petitioner’s behavior after the alleged fraud perpetrated against her was
inconsistent with the actuation of someone who had been swindled. The petition for the award of
damages is denied.
Case 17 – Sim
LIM vs KOU CO PING
679 SCRA 114

Doctrine:
The independent civil liabilities are separate from the criminal action and may be pursued
independently, as provided in Arts. 31 and 33 of the Civil Code

Facts:
FR Cement Corporation (FRCC) issued several withdrawal authorities for the account of cement
dealers and traders, Fil-Cement Center and Tigerbilt. FRCC and Tigerbilt then sold the withdrawal
authorities covering 50,000 bags of cement to Co for the amount of ₱ 3.15 million or ₱ 63.00 per bag.
Co then sold these withdrawal authorities to Lim allegedly at the price of ₱ 64.00 per bag or a total of ₱
3.2 million. Lim withdrew the cement bags from FRCC on a staggered basis. She successfully withdrew
2,800 bags of cement, and sold back some of the withdrawal authorities, covering 10,000 bags, to Co.
FRCC did not allow Lim to withdraw the remaining 37,200 bags covered by the withdrawal authorities.
Lim clarified the matter with Co and Borja, who explained that the plant implemented a price increase
and would only release the goods once Lim pays for the price difference or agrees to receive a lesser
quantity of cement. Lim objected and maintained that the withdrawal authorities she bought were not
subject to price fluctuations. ​
Lim then filed an Information for Estafa through Misappropriation or Conversion was filed
against Co. the case was dismissed and the civil liability was subsequently dismissed. Lim appealed the
dismissal of the civil liability before the CA. While the appeal before the CA was pending, she filed a
complaint for specific performance and damages before the RTC of Manila. The complaint asserted two
causes of actions breach of contract and abuse of rights. Thus this case.

Issue:
Whether or not Lim commit forum shopping in filing the civil case for specific performance and
damages during the pendency of her appeal on the civil aspect of the criminal case for Estafa?

Ruling:
NO, Lim did not commit forum shopping. A single act or omission that causes damage to an
offended party may give rise to two separate civil liabilities on the part of the offender (1) civil liability
ex delicto and (2) independent civil liability. The independent civil liability may be based on "an
obligation not arising from the act or omission complained of as a felony," as provided in Article 31 of
the Civil Code It may also be based on an act or omission that may constitute felony but, nevertheless,
treated independently from the criminal action by specific provision of Article 33 of the Civil Code.
Although the cases filed by Lim arose from the same act or omission of Co they are, however,
based on different causes of action. The criminal cases for Estafa are based on culpa criminal while the
civil action for collection is anchored on culpa contractual. Moreover, there can be no forum-shopping in
the instant case because the law expressly allows the filing of a separate civil action which can proceed
independently of the criminal action.

Case 18 – Velasco
CASUPANAN vs LAROYA
G.R. No. 145391 August 26, 2002

Doctrine: Rule 111, Rules of Court

Facts:
As a result of a vehicular accident between two vehicles, one driven by Mario Llavore Laroya
and the other owned by Roberto Capitulo and driven by Avelino Casupanan, two cases were filed before
the MCTC of Capas, Tarlac. Laroya filed a criminal case against Casupanan for reckless imprudence
resulting in damage to property. This case was on its preliminary investigation stage when Casupanan
and Capitulo filed a civil case against Laroya for quasi-delict. However, upon motion of Laroya on the
ground of forum-shopping, the MCTC dismissed the civil case. On Motion for Reconsideration,
Casupanan and Capitulo insisted that the civil case is a separate civil action which can proceed
independently of the criminal case. Casupanan and Capitulo then filed a petition for certiorari before the
Regional Trial Court (RTC) of Capas, Tarlac. But the RTC ruled that the order of dismissal issued by the
MCTC is a final order which disposes of the case and therefore, the proper remedy should have been an
appeal. Hence, Casupanan and Capitulo filed this petition.

Casupanan and Capitulo’s contention: that if the accused in a criminal case has a counterclaim
against the private complainant, he may file the counterclaim in a separate civil action at the proper
time. They contend that an action on quasi-delict is different from an action resulting from the crime of
reckless imprudence, and an accused in a criminal case can be an aggrieved party in a civil case arising
from the same incident. They maintain that under Articles 31 and 2176 of the Civil Code, the civil case
can proceed independently of the criminal action. Finally, they point out that Casupanan was not the
only one who filed the independent civil action based on quasi-delict but also Capitulo, the owner-
operator of the vehicle, who was not a party in the criminal case.

Laroya’s contention: that the petition is fatally defective as it does not state the real antecedents.
Laroya further alleges that Casupanan and Capitulo forfeited their right to question the order of
dismissal when they failed to avail of the proper remedy of appeal. Laroya argues that there is no
question of law to be resolved as the order of dismissal is already final and a petition for certiorari is not
a substitute for a lapsed appeal

Issue:

Whether an accused in a pending criminal case for reckless imprudence can validly file,
simultaneously and independently, a separate civil action for quasi-delict against the private complainant
in the criminal case.

Ruling: (Affirmative)

The Court held that the MCTC dismissed the civil action for quasi-delict on the ground of forum-
shopping under Supreme Court Administrative Circular No. 04-94. The MCTC did not state in its order
of dismissal that the dismissal was with prejudice. Under the Administrative Circular, the order of
dismissal is without prejudice to refiling the complaint, unless the order of dismissal expressly states that
it is with prejudice. Thus, the MCTC's dismissal, being silent on the matter, is a dismissal without
prejudice. Section 1 of Rule 41 provides that an order dismissing an action without prejudice is not
appealable. The remedy of the aggrieved party is to file a special civil action under Rule 65. Clearly, the
Capas RTC's order dismissing the petition for certiorari on the ground that the proper remedy is an
ordinary appeal, is erroneous.

Laroya filed the criminal case for reckless imprudence resulting in damage to property based on the
Revised Penal Code while Casupanan and Capitulo filed the civil action for damages based on Article
2176 of the Civil Code. Although these two actions arose from the same act or omission, they have
different causes of action. The criminal case is based on culpa criminal punishable under the Revised
Penal Code while the civil case is based on culpa aquiliana actionable under Articles 2176 and 2177 of
the Civil Code. And par 6, sec 1 of Rule 111.

Since the present Rules require the accused in a criminal action to file his counterclaim in a
separate civil action, there can be no forum-shopping if the accused files such separate civil action.

Under the present Rule 111, the offended party is still given the option to file a separate civil
action to recover civil liability ex-delicto by reserving such right in the criminal action before the
prosecution presents its evidence. Also, the offended party is deemed to make such reservation if he files
a separate civil action before filing the criminal action. If the civil action to recover civil liability ex-
delicto is filed separately but its trial has not yet commenced, the civil action may be consolidated with
the criminal action. The consolidation under this Rule does not apply to separate civil actions arising
from the same act or omission filed under Articles 32, 33, 34 and 2176 of the Civil Code.

Section 2, Rule 111 of the present Rules did not change the rule that the separate civil action,
filed to recover damages ex-delicto, is suspended upon the filing of the criminal action. Section 2 of the
present Rule 111 also prohibits the filing, after commencement of the criminal action, of a separate civil
action to recover damages ex-delicto.

Case 19 – Botor

PEOPLE vs. ROMERO


306 SCRA 90

Doctrine: Extinguishment of Civil Liability upon Death of Accused

Facts:
Complainant Ernesto A. Ruiz went to SAIDECOR office, where Romero was the president and
general manager of SAIDECOR, while Rodriguez was the operations manager, to make an
investment. After handing over the amount of one hundred fifty thousand pesos (P150,000.00) to
Ernesto Rodriguez, complainant received a postdated Butuan City Rural Bank check instead of the usual
redeemable coupon. When the check was presented to the bank for payment on October 5, 1989, it was
dishonored for insufficiency of funds, as evidenced by the check return slip issued by the bank. Both
accused could not be located and demand for payment was made only sometime in November 1989
during the preliminary investigation of this case. Accused responded that they had no money.
For their defense, accused Martin Romero testified that when he issued the check, the
corporation had a deposit of fourteen million pesos (P14,000,000.00) and four million pesos
(P4,000,000.00) at the time SAIDECOR stopped operations.
In this appeal, both accused did not deny that complainant made an investment with SAIDECOR
in the amount of P150,000.00. However, they denied that deceit was employed in the transaction.

Issue:
Where the accused guilty of Estafa? And is the civil liability of Rodriguez deemed extinguished
since he died while the case was on appeal?

Ruling:
Yes. In this case, there was deception when accused fraudulently represented to complainant that
his investment with the corporation would have an 800% return in 15 or 21 days.
Upon receipt of the money, accused-appellant Martin Romero issued a postdated check. Although
accused-appellant contends that sufficient funds were deposited in the bank when the check was issued,
he presented no officer of the bank to substantiate the contention. The check was dishonored when
presented for payment, and the check return slip submitted in evidence indicated that it was dishonored
due to insufficiency of funds.
Even assuming for the sake of argument that the check was dishonored without any fraudulent
pretense or fraudulent act of the drawer, the latters failure to cover the amount within three days after
notice creates a rebuttable presumption of fraud.
The Court notes that one of the accused-appellants, Ernesto Rodriguez, died pending appeal.
Pursuant to the doctrine established in People vs. Bayotas, the death of the accused pending appeal of
his conviction extinguishes his criminal liability as well as the civil liability ex delicto. The criminal
action is extinguished inasmuch as there is no longer a defendant to stand as the accused, the civil action
instituted therein for recovery of civil liability ex delicto is ipso facto extinguished, grounded as it is on
the criminal case. Corollarily, the claim for civil liability survives notwithstanding the death of the
accused, if the same may also be predicated on a source of obligation other than delict.
Case 20 – Bugtas

MAGISTRADO vs. PEOPLE


527 SCRA 125

Doctrine:
A prejudicial question is defined as that which arises in a case the resolution of which is a logical
antecedent of the issue involved therein, and he cognizance of which pertains to another tribunal.
Further, a prejudicial question must be determinative of the case before the court, but the jurisdiction
must be lodged in another court. “It is a question based on a fact distinct and separate from the crime
but so intimately connected with it that it determines the guilt or innocence of the accused.
Facts:
Private respondent Elena Librojo filed a criminal complaint of against accused Francisco
Magistrado before the the Office of the Prosecutor of Quezon City. Thereafter the prosecutor
recommended the filing of the complaint against accused. An Information was filed against the accused
for perjury before the MeTC of Quezon City.
The Information alleged that the accused subscribed and swore to an Affidavit of Loss before a
notary public stating that he lost his Owner’s Duplicate Certificate of TCT. The same affidavit was used
to by accused to support his Petition for Issuance of New Owner’s Duplicate Copy of Certificate of TCT
filed with the RTC of Quezon City. A verification was again signed and sworn into by the accused
before the notary public. However, the contents of the same affidavit, already known to the accused, are
false. It was later found out that the property subject of the TCT was mortgaged to respondent Librojo as
collateral for a loan. As a result, respondent suffered damages and prejudice due to the deliberate
assertion of falsehoods by the accused.
Subsequently, petitioner-accused Magistrado filed a motion to suspend the proceedings on the
ground of a prejudicial question. Petitioner alleged that two civil cases (for recovery of sum of money
and for cancellation of mortgage) were pending before the RTC of Quezon City, and that they must be
resolved first before the present criminal case. The RTC of Quezon City denied the motion. Hence this
petition.
Issue:
Whether or not the two civil cases (for Recovery of Sum of Money and for Cancellation of
Mortgage) constitute a prejudicial question that would warrant a suspension of the criminal case of
perjury
Ruling:
A prejudicial question is defined as that which arises in a case the resolution of which is a logical
antecedent of the issue involved therein, and he cognizance of which pertains to another tribunal.
Further, a prejudicial question must be determinative of the case before the court, but the jurisdiction
must be lodged in another court. “It is a question based on a fact distinct and separate from the crime but
so intimately connected with it that it determines the guilt or innocence of the accused.” The Court
enumerated the requisites of a prejudicial question that would suspend the criminal proceedings until
final resolution of the civil case:
1. The civil case involves facts intimately related to those upon which the criminal prosecution would
be based;
2. The guilt or innocence of the accused would necessarily be determined in resolving the issued raised
in the civil case;
3. Jurisdiction to try the question is lodged in another tribunal.
In conclusion the Court stated—it is evident that the civil cases and the criminal cases can proceed
independently of each other. Regardless of the outcome of the two civil cases, it will not establish the

innocence or guilt of the petitioner in the criminal case of perjury. The purchase by petitioner of the land
or his execution of a real estate mortgage will have no bearing whatsoever on whether petitioner
knowingly and fraudulently executed a false affidavit of loss of the TCT.

Case 21 – Cabalonga
PIMENTEL vs PIMENTEL
630 SCRA 436
Doctrine:

Annulment of marriage under Article 36 of the Family Code is not a prejudicial question in a criminal
case for parricide.

Facts:

On 25 October 2004, Maria Pimentel y Lacap(private respondent) filed an action for frustrated
parricide against Joselito Pimentel (petitioner) before the Regional Trial Court of Quezon City.

On 7 February 2005, petitioner received summons to appear before the Regional Trial Court of
Antipolo City for the pre-trial and trial of a civil case (Maria Pimentel v. Joselito Pimentel) for
Declaration of Nullity of Marriage under Article 36 of the Family Code on the ground of psychological
incapacity.

On 11 February 2005, petitioner filed an urgent motion to suspend the proceedings before the
RTC Quezon City on the ground of the existence of a prejudicial question. Petitioner asserted that since
the relationship between the offender and the victim is a key element in parricide, the outcome of the
civil case would have a bearing in the criminal case filed against him before the RTC Quezon City.

The RTC Quezon City held that the pendency of the case before the RTC Antipolo is not a
prejudicial question that warrants the suspension of the criminal case before it.

Petitioner filed a petition for certiorari with application for a writ of preliminary injunction
and/or temporary restraining order before the Court of Appeals. However, The Court of Appeals ruled
that even if the marriage between petitioner and respondent would be declared void, it would be
immaterial to the criminal case because prior to the declaration of nullity, the alleged acts constituting
the crime of frustrated parricide had already been committed.

Issue:

Whether the resolution of the action for annulment of marriage is a prejudicial question that
warrants the suspension of the criminal case for frustrated parricide against petitioner.

Ruling:

No. Section 7, Rule 111 of the 2000 Rules on Criminal Procedure provides that elements of a
prejudicial question are: (a) the previously instituted civil action involves an issue similar or intimately
related to the issue raised in the subsequent criminal action and (b) the resolution of such issue
determines whether or not the criminal action may proceed.

In the case at bar, the civil case for annulment was filed after the filing of the criminal case for
frustrated parricide. As such, the requirement of Section 7, Rule 111 of the 2000 Rules on Criminal
Procedure was not met since the civil action was filed subsequent to the filing of the criminal action.
The relationship between the offender and the victim is a key element in the crime of parricide,
which punishes any person “who shall kill his father, mother, or child, whether legitimate or illegitimate,
or any of his ascendants or descendants, or his spouse.” However, the issue in the annulment of marriage
is not similar or intimately related to the issue in the criminal case for parricide. Further, the relationship
between the offender and the victim is not determinative of the guilt or innocence of the accused.

The issue in the civil case for annulment of marriage under Article 36 of the Family Code is
whether petitioner is psychologically incapacitated to comply with the essential marital obligations. The
issue in parricide is whether the accused killed the victim. In this case, since petitioner was charged with
frustrated parricide, the issue is whether he performed all the acts of execution which would have killed
respondent as a consequence but which, nevertheless, did not produce it by reason of causes independent
of petitioner’s will. At the time of the commission of the alleged crime, petitioner and respondent were
married. The subsequent dissolution of their marriage will have no effect on the alleged crime that was
committed at the time of the subsistence of the marriage. In short, even if the marriage between
petitioner and respondent is annulled, petitioner could still be held criminally liable since at the time of
the commission of the alleged crime, he was still married to respondent.

We cannot accept petitioner’s reliance on Tenebro v. Court of Appeals that “the judicial
declaration of the nullity of a marriage on the ground of psychological incapacity retroacts to the date of
the celebration of the marriage insofar as the vinculum between the spouses is concerned x x x.” First,
the issue in Tenebro is the effect of the judicial declaration of nullity of a second or subsequent marriage
on the ground of psychological incapacity on a criminal liability for bigamy. There was no issue of
prejudicial question in that case. Second, the Court ruled in Tenebro that “[t]here is x x x a recognition
written into the law itself that such a marriage, although void ab initio, may still produce legal
consequences.” In fact, the Court declared in that case that “a declaration of the nullity of the second
marriage on the ground of psychological incapacity is of absolutely no moment insofar as the State’s
penal laws are concerned.”

Case 22 – Casiano

JM DOMINGUEZ AGRONOMIC COMPANY, INC., et al vs. CECILIA LICLICAN et al


G.R. No. 208587. July 29, 2015

Doctrine:
Prejudicial Question exists in a situation where a civil action and a criminal action are both pending,
and there exists in the former an issue that must be preemptively resolve before te later may proceed
because the issue raised in civil action would be determinative juris et de jure of the guilt or innocence
of the accused in the criminal case.
Essential elements:
(i) the action involves an issue or intimately related to the issue raised in the criminal action
(ii) the resolution of such issue determines whether or not the criminal action may proceed

Facts:
JM Dominguez Agronomic Company hold its annual stockholders meeting at Baguio Country Club
for election for its new set of directors. It is presided by then President Cecilia Liclican. During the
meeting conflict ensued when Kenneth and Patrick Pacis were not allowed to vote on the ground that the
shares of their mother and grandmother in the company were not yet registered on their name Since
there is still no settlement on the estates of the latter.
The respondents walked out of the meeting. But since the remaining stockholders with outstanding
shares constituted a quorum, the election of officers still proceeded to which petitioner Helen Dagdagan
was elected as President. Consequently, after staging the walk-out, respondents, executed a Board
Resolution certifying that in the stockholders meeting Cecile Liclican was President.
Petitioner filed complaint against respondents before RTC for the nullification of the meetings and
board resolutions. The two groups failed to come up with an agreement. Meanwhile, Petitioner
stockholder immediately took hold of the company and presents itself as the lawful directors.
Subsequently, while the civil case is ongoing to resolve who among the two groups has legitimate
right to hold the position. Dagdagan and her group find separate affidavit complaint against Cecilia
Liclican for qualified theft for unauthorize withdrawals against the corporate funds.
Prosecutor ordered and information be filed against Cecilia Liclican and upon finding of probable
cause Judge Tiongson -Tabora cause the issuance of warrant of arrest against Cecilia Liclican.
On her part, Liclican file for petition for certiorari with CA to annul and set aside the orders of Judge
Tabora on the allege existence of prejudicial question

Issue:
Whether or not Civil Case constituted a prejudicial question warranting the suspension of the
proceedings in Criminal Case filed by Petitioner

Ruling:
Yes. The Civil Case is a prejudicial question to Criminal Case. It involves the same parties which
seeks the court intervention to determine between the two constesting group of officers should rightfully
be seated at the company. Without Civil Case resolution, petitioners' authority to commence and
prosecute Criminal Case against respondents for qualified theft in JMD's behalf remained questionable,
warranting the suspension of the criminal proceedings because a juridical person can only act through its
officers. The question as to who the legitimate directors of JMD needs to be resolve first hence the
criminal action needs to be suspended.

Case 23 – Cuevas
FENEQUITO vs. VERGARA, JR.
677 SCRA 113
Doctrine:
Probable cause, for the purpose of filing a criminal information, has been defined as such facts as are
sufficient to engender a well-founded belief that a crime has been committed and that respondent is
probably guilty thereof. The term does not mean "actual and positive cause" nor does it import absolute
certainty. It is merely based on opinion and reasonable belief. Probable cause does not require an
inquiry into whether there is sufficient evidence to procure a conviction. It is enough that it is believed
that the act or omission complained of constitutes the offense charged.
Facts:
A criminal complaint for falsification of public documents was filed by herein respondent against
herein petitioners with the Office of the City Prosecutor of Manila. On February 11, 2004, an
Information for falsification of public documents was filed with the Metropolitan Trial Court of Manila
by the Assistant City Prosecutor of Manila against herein petitioners. On April 23, 2004, herein
petitioners filed a Motion to Dismiss the Case Based on Absence of Probable Cause. After respondent's
Comment/Opposition was filed, the MeTC issued an Order dismissing the case on the ground of lack of
probable cause. Aggrieved, respondent, with the express conformity of the public prosecutor, appealed
the case to the Regional Trial Court (RTC) of Manila. On July 21, 2005, the RTC rendered judgment
setting aside the July 9, 2004 Order of the MeTC and directing the said court to proceed to trial.
Petitioners then elevated the case to the CA via a Petition for Review. On March 9, 2006, the CA
rendered dismissed the petition and ruled that the Decision of the RTC is interlocutory in nature and,
thus, is not appealable. Petitioners filed a Motion for Reconsideration, but the CA denied it. Petitioners
now claim that assuming for the sake of argument that the remedy they availed of is not proper, the facts
of the case would readily show that there exist just and compelling reasons to warrant the relaxation of
the rules in the interest of substantial justice. Petitioners contend that the PNP Crime Laboratory
Questioned Document Report, submitted as evidence by respondent to the prosecutor's office, showed
that the findings therein are not conclusive and, thus, insufficient to support a finding of probable cause.

Issue:
​Whether the petitioners are correct in contenting that the PNP Crime Laboratory Questioned
Document Report showing findings that are not conclusive are insufficient to support a finding of
probable cause.

Ruling:
A finding of probable cause needs only to rest on evidence showing that, more likely than not, a
crime has been committed by the suspects. It need not be based on clear and convincing evidence of
guilt, not on evidence establishing guilt beyond reasonable doubt, and definitely not on evidence
establishing absolute certainty of guilt. In determining probable cause, the average man weighs facts and
circumstances without resorting to the calibrations of the rules of evidence of which he has no technical
knowledge. He relies on common sense. What is determined is whether there is sufficient ground to
engender a well-founded belief that a crime has been committed, and that the accused is probably guilty
thereof and should be held for trial. It does not require an inquiry as to whether there is sufficient
evidence to secure a conviction.

It is clear from a perusal of the cited PNP Crime Laboratory Questioned Document Report No.
048-03 that the document examiner found that the signatures appearing in the questioned Deed of Sale
as compared to the standard signatures “reveal divergences in the manner of execution and stroke
structure [which is] an indication that they WERE NOT WRITTEN BY ONE AND THE SAME
PERSON.”20 The Court agrees with the prosecutor's pronouncement in its Resolution21 dated
September 22, 2003, that although the findings of the PNP Crime Laboratory were qualified by the
statement contained in the Report that “no definite conclusion can be rendered due to the fact that
questioned signatures are photocopies wherein minute details are not clearly manifested,” the fact that
an expert witness already found that the questioned signatures were not written by one and the same
person already creates probable cause to indict petitioners for the crime of falsification of public
document. WHEREFORE, the instant petition is DENIED.

Case 24 – Custodio

BURGUNDY REALTY CORPORATION vs. JOSEFA "JING" C. REYES


G.R. No. 181021 December 10, 2012

Doctrine:

A preliminary investigation constitutes a realistic judicial appraisal of the merits of a case. Its purpose is
to determine whether (a) a crime has been committed; and (b) whether there is a probable cause to
believe that the accused is guilty thereof.

In a preliminary investigation, the public prosecutor merely determines whether there is probable cause
or sufficient ground to engender a well-founded belief that a crime has been committed, and that the
respondent is probably guilty thereof and should be held for trial. It does not call for the application of
rules and standards of proof that a judgment of conviction requires after trial on the merits. The
complainant need not present at this stage proof beyond reasonable doubt. A preliminary investigation
complainant need not present at this stage proof beyond reasonable doubt. A preliminary investigation
does not require a full and exhaustive presentation of the parties' evidence. Precisely, there is a trial to
allow the reception of evidence for both parties to substantiate their respective claims.

Facts:

Sometime in 1996, Josefa “Jing” C. Reyes offered her services as real estate agent to Burgundy
Realty Corporation in buying parcels of land in Calamba, Laguna, to be developed into a golf course.

Convinced of her representations, petitioner released the amount of ₱23,423,327.50 in her favor
to be used in buying those parcels of land.

However, Reyes, instead of buying those parcels of land, allegedly converted and
misappropriated the money given by petitioner to her personal use and benefit.

Despite a formal demand for Reyes to return the amount of ₱23,423,327.50, it was left unheeded.
Thus, Burgundy Realty Corporation filed a complaint for the crime of Estafa against Reyes before the
Assistant City Prosecutor’s Office of Makati City.

Issue:

Whether or not there is sufficient ground to engender a well-founded belief that the crime of
Estafa has been committed, and that Josefa “Jing” C. Reyes is probably guilty thereof and should be
held for trial

Ruling:

A review of the records would show that the investigating prosecutor was correct in finding the
existence of all the elements of the crime of estafa.

Reyes did not dispute that she received in trust the amount of ₱23,423,327.50 from petitioner as
proven by the checks and vouchers to be used in purchasing the parcels of land. Petitioner wrote a
demand letter for Reyes to return the same amount but was not heeded. Hence, the failure of Reyes to
deliver the titles or to return the entrusted money, despite demand and the duty to do so,
constituted prima facie evidence of misappropriation.

In proving the element of conversion or misappropriation, a legal presumption of


misappropriation arises when the accused fails to deliver the proceeds of the sale or to return the items to
be sold and fails to give an account of their whereabouts. Thus, the mere presumption of
misappropriation or conversion is enough to conclude that a probable cause exists for the indictment of
Reyes for Estafa. As to whether the presumption can be rebutted by Reyes is already a matter of defense
that can be best presented or offered during a full-blown trial.
Case 25 – De Luna

CITY PROSECUTOR ARMANDO P. ABANADO vs. JUDGE ABRAHAM A. BA YONA


677 SCRA 595

Doctrine: Preliminary Investigation as purely an Executive Function


Facts:
2
City Prosecutor Armando P. Abanado filed the Information in the Municipal Trial Court in
Cities, Bacolod City, which was eventually raffled to Branch 7 thereof presided by respondent Judge
Abraham A. Bayona.
Respondent Judge asked the petitioner to present additional evidence, relevant records and
documents to enable the Court to evaluate and determine the existence of probable cause for issuance of
the warrant of arrest.
With respect to Memorandum of the transfer of case assignment from designated Investigating
Prosecutor to the City Prosecutor required by the judge, petitioner discussed that the case was initially
handled by ACP Jarder who found no probable cause against Cresencio Palo, Sr. However, complainant,
upon review pursuant to Section 4, Rule 112 of the Revised Rules of Criminal Procedure, found
otherwise; that is, there was probable cause against Palo. Thus, complainant disapproved ACP Jarder’s
Resolution and filed the Information in court.
Respondent was nonetheless dissatisfied with the explanation of the Office of the City
Prosecutor. He ruled that Jarder Resolution (dismissing the complaint) was part and parcel of the official
records of the case and, for this reason, must form part of the records of the preliminary investigation.
The petitioner again insisted on the impossibility of submitting the Jarder Resolution to the court
pursuant to Circular Resolution No. 12.
Respondent did not accept the explanations made by the Office of the City Prosecutor and
insisted instead that the Jarder Resolution should form part of the records of the case and ordered the
petitioner in contempt.
Issue:
Whether the court may be allowed to interfere in the rules of procedures of the executive.
Ruling:
30
The conduct of a preliminary investigation is primarily an executive function. Thus, the courts
must consider the rules of procedure of the Department of Justice in conducting preliminary
investigations whenever the actions of a public prosecutor is put in question.
We also note that attaching such a resolution to an information filed in court is optional under the
aforementioned manual. The DOJ-NPS Manual states that the resolution of the investigating prosecutor
should be attached to the information only "as far as practicable." Thus, such attachment is not
mandatory or required under the rules.
In view of the foregoing, the Court finds that respondent erred in insisting on the production of
the Jarder Resolution when all other pertinent documents regarding the preliminary investigation have
been submitted to his court, and in going so far as to motu proprio initiating a proceeding for contempt
against complainant.

Case 26 – Dionisio
HEIRS OF THE LATE NESTOR TRIA vs. ATTY. EPIFANIA OBIAS
G.R. NO. 175887 NOV. 24, 2010

DOCTRINE: Preliminary Investigation; Appeal/Petition for Review


The President shall only entertain appeal or petition for review of decisions/orders/resolutions of the
Secretary of Justice on preliminary investigations of criminal cases involving (1) offenses punishable by
reclusion perpetua to death wherein (2) new and material issues are raised which (3) were not
previously presented before the Department of Justice and (4) were not ruled upon in the subject
decision/order/resolution; When Jurisdictional Requirement is proved; President shall order the
Secretary of Justice to reopen/review the preliminary investigation on said criminal case; Period of
Appeal or Petition for Review to the Office of the President - within 30 days from notice of the
questioned decision/order/resolution of the Secretary of Justice; Prescription – Offense must not due to
lapse within 6 months from notice of the questioned decision/order/resolution of the Secretary of Justice.

Facts:
Engr. Nestor Tria, RD of DPWH Region V was shot by a gunman while waiting to board his
flight to Manila. He was brought to a hospital but died the following day from the lone gunshot wound
on his nape. NBI Regional Director Alejandro R. Tenerife, Chairman of Task Force Tria, recommended
to the Provincial Prosecutor of Camarines Sur the indictment of Obet Aclan, Totoy Ona and Atty.
Epifania “Fanny” Gonzales-Obias, for the murder of Engr. Tria. During the preliminary investigation
respondent filed her Counter-Affidavit denying that she was in anyway involved with the killing of
Engr. Tria and further asserted that from the totality of evidence gathered by the NBI, it has not
established prima facie the existence of conspiracy as to implicate her in the death of Engr. Tria.
The Prosecutor issued a resolution directing the filing of an information for murder against Aclan
and Ona but dismissed the case for insufficiency of evidence as against Obias. Justice Secretary Cuevas
issued a Resolution directing the Prosecutor to include Obias in the information.
The DOJ was convinced that the sequence of events and respondent’s conduct before, during and
after the killing of Engr. Tria undeniably points to her complicity with Aclan and Ona. The DOJ denied
respondent’s motion for reconsideration stating that the proper procedure is the filing of an appeal or
petition for review with the OP and not before the DOJ. Hence, the case was considered closed and
terminated. OP dismissed the murder charge, CA affirmed OP’s decision.

Issue:
Whether or not the CA gravely abused uts discretion in affirming the OP’s dismissal of the
murder charge.

Ruling:
NO. In arguing that the CA gravely abused its discretion when it affirmed the OP’s dismissal of
the murder charge against respondent, petitioner invoked our ruling in Crespo v. Mogul that any
disposition of the case rests on the sound discretion of the court once an information has been filed with
it. A refinement of petitioners’ understanding of the Crespo ruling is in order. In Crespo, we ruled that
after the information has already been filed in court, the court’s permission must be secured should the
fiscal find it proper that reinvestigation be made. Thereafter, the court shall consider and act upon the
findings and recommendations of the fiscal. In Ledesma v. Court of Appeal we clarified that the justice
secretary is not precluded from exercising his power of review over the investigating prosecutor even
after the information has already been filed in court. However, the justice secretary’s subsequent
resolution withdrawing the information or dismissing the case does not cause the court to lose
jurisdiction over the case.
In fact, the court is duty bound to exercise judicial discretion and its own independent judgment
in assessing the merits of the resulting motion to dismiss filed by the prosecution. In resolving the issue
of whether the CA gravely abused its discretion in affirming the OP’s reversal of the ruling of the
Secretary of Justice, it is necessary to determine whether probable cause exists to charge the respondent
for conspiracy in the murder of Engr. Tria. A prosecutor, by the nature of his office, is under no
compulsion to file a particular criminal information where he is not convinced that he has evidence to
prop up its averments, or that the evidence at hand points to a different conclusion.
The decision whether or not to dismiss the criminal complaint against respondent is necessarily
dependent on the sound discretion of the investigating prosecutor and ultimately, that of the Secretary of
Justice. The findings of the prosecutor with respect to the existence or non-existence of probable cause
is subject to the power of review by the DOJ. Indeed, the Secretary of Justice may reverse or modify the
resolution of the prosecutor, after which he shall direct the prosecutor concerned either to file the
corresponding information without conducting another preliminary investigation, or to dismiss or move
for dismissal of the complaint or information with notice to the parties.
​Ordinarily, the determination of probable cause is not lodged with this Court. Its duty in an
appropriate case is confined to the issue of whether the executive or judicial determination, as the case
may be, of probable cause was done without or in excess of jurisdiction or with abuse of discretion
amounting to want of jurisdiction. However, this Court may ultimately resolve the existence or non-
existence of probable cause by examining the records of the preliminary investigation when necessary
for the orderly administration of justice, or to avoid oppression or multiplicity of action. To begin with,
whether or not respondent actually conspired with Aclan and Ona need not be fully resolved during the
preliminary investigation. The absence or presence of conspiracy is factual in nature and involves
evidentiary matters. The same is better left ventilated before the trial court during trial, where the parties
can adduce evidence to prove or disprove its presence. Preliminary investigation is executive in
character. It does not contemplate a judicial function. It is essentially an inquisitorial proceeding, and
often, the only means of ascertaining who may be reasonably charged with a crime. Prosecutors control
and direct the prosecution of criminal offenses, including the conduct of preliminary investigation,
subject to review by the Secretary of Justice. The duty of the Court in appropriate cases is merely to
determine whether the executive determination was done without or in excess of jurisdiction or with
grave abuse of discretion. Resolutions of the Secretary of Justice are not subject to review unless made
with grave abuse.
Case 27– Dolor
UY vs. JAVELLANA
680 SCRA 13

Doctrine:
The Revised Rule on Summary Procedure does not provide for a preliminary investigation prior to the
filing of a criminal case under said Rule. A criminal case within the scope of the Rule shall be
commenced in the following manner: SEC. 11. How commenced.―The filing of criminal cases falling
within the scope of this Rule shall be either by complaint or by information; Provided, however, That in
Metropolitan Manila and in Chartered Cities, such cases shall be commenced only by information,
except when the offense cannot be prosecuted de oficio. The complaint or information shall be
accompanied by the affidavits of the complainant and of his witnesses in such number of copies as there
are accused plus two (2) copies for the court’s files. If this requirement is not complied with within five
(5) days from date of filing, the case may be dismissed.

Facts:
This administrative case arose from a verified complaint for "gross ignorance of the law and
procedures, gross incompetence, neglect of duty, conduct improper and unbecoming of a judge, grave
misconduct and others," filed by Public Attorneys Uy and Bascug of PAO against Presiding Judge
Javellana of the Municipal Trial Court, La Castellana, Negros Occidental.
Public Attorneys Uy and Bascug alleged the following in their complaint: Judge Javellana was
grossly ignorant of the Revised Rule on Summary Procedure. Public Attorneys Uy and Bascug cited
several occasions as examples: Judge Javellana issued a warrant of arrest after the filing of said case
despite Section 16 of the Revised Rule on Summary Procedure; did not grant the motion to dismiss for

non-compliance with the Lupon requirement under Sections 18 and 19(a) of the Revised Rule on
Summary Procedure, insisting that said motion was a prohibited pleading; refused to dismiss outright the
complaint even when the same was patently without basis or merit, as the affidavits of therein
complainant and her witnesses were all hearsay evidence; and did not apply the Revised Rule on
Summary Procedure and, instead, conducted a preliminary examination and preliminary investigation in
accordance with the Revised Rules of Criminal Procedure, then set the case for arraignment and pre-
trial, despite confirming that therein complainant and her witnesses had no personal knowledge of the
material facts alleged in their affidavits, which should have been a ground for dismissal of said case.
Judge Javellana violated Section 6(b), Rule 112 of the Revised Rules of Criminal Procedure and
issued warrants of arrest without propounding searching questions to the complainants and their
witnesses to determine the necessity of placing the accused under immediate custody. As a result, Judge
Javellana issued warrants of arrest even when the accused had already voluntarily surrendered or when a
warrantless arrest had been effected.
Judge Javellana failed to observe the constitutional rights of the accused as stated in Section
12(1), Article III of the Constitution. Judge Javellana set Crim. Case No. 03-097, entitled People v.
Bautista, for preliminary investigation even when the accused had no counsel, and proceeded with said
investigation without informing the accused of his rights to remain silent and to have a counsel Judge
Javellana stressed that the charges against him were baseless and malicious; and the acts being
complained of involved judicial discretion and, thus, judicial in nature and not the proper subject of an
administrative complaint.
Consequently, Judge Javellana sought the dismissal of the instant complaint against him. The
Office of the Court Administrator (OCA), in its report, found Judge Javellana liable for gross ignorance
of the law or procedure when he did not apply the Revised Rule on Summary Procedure in cases
appropriately covered by said Rule.

Issue:
Whether or not Judge Javellana was grossly ignorant of the Revised Rule on Summary
Procedure.

Ruling:
YES. Without any showing that the accused in People v. Cornelio and People v. Lopez, et al.
were charged with the special cases of malicious mischief particularly described in Article 328 of the
Revised Penal Code the appropriate penalty for the accused would be arresto mayor in its medium and
maximum periods which under Article 329(a) of the Revised Penal Code, would be imprisonment for
two (2) months and one (1) day to six (6) months. Clearly, these two cases should be governed by the
Revised Rule on Summary Procedure.
Judge Javellana’s issuance of a Warrant of Arrest for the accused in People v. Cornelio is in
violation of Section 16 of the Revised Rule on Summary Procedure, categorically stating that "the court
shall not order the arrest of the accused except for failure to appear whenever required.

Case 28 – Evagelista

PEOPLE OF THE PHILIPPINES vs. SEGUNDINO VALENCIA


G.R. No. 143032 October 14, 2002

Doctrine:

In a buy-bust operation, the idea to commit a crime originates from the offender, without anybody
inducing or prodding him to commit the offense. Its opposite is instigation or inducement, wherein the
police or its agent lures the accused into committing the offense in order to prosecute him. Instigation is
deemed contrary to public policy and considered an absolutory cause.

Facts:
​Accused-appellants Segundino Valencia y Blanca, Johnny Tadena y Torda and Domingo Deroy,
Jr. y Sarocam were charged and convicted by the Regional Trial Court of Quezon City for violation of
Section 15 of Republic Act (R.A.) 6425, otherwise known as the Dangerous Drugs Act, for unlawfully
selling or offering to sell 634.0 grams of Psuedophedrine Hydrochloride which is a regulated drug. The
trial court sentenced each of the accused to the supreme penalty of death and to pay a fine of
P500,000.00. Hence, the case is now before the Supreme Court for automatic review.
​From the prosecution evidence, a confidential informant of the PNP Narcotics Group confided to
the group that he was able to negotiate the purchase of one kilo of drugs from a certain Junior and
Johnny. The information was passed to the operatives team leader, Insp. Ramon Arsenal and then to
their commanding officer, Supt. Arturo Castillo. Supt. Castillo immediately formed a buy-bust operation
team composed of P/Insp. Arsenal, P/Insp. Beasa, SPO2 Estrada and SPO1 Facto.
​From the defense, Valencia was allegedly engaged in the business of buying and selling used
cars. The passengers of the van who appeared to be police officers approached them. The police brought
Valencia and Tadena to Camp Crame. Tadena was placed in a jail cell while Valencia was brought before
Col. Castillo. Col. Castillo showed Valencia a plastic bag and said that he would use it as evidence
against him. Valencia claimed that the police mauled him and extorted from him the amount of
P20,000.00.
​Giving more weight to the testimony of the police officers who conducted the buy-bust
operation, the trial court convicted the accused of the crime charged. It held that the denial and alibi of
the accused were not sufficient to overturn the prosecution evidence which established the guilt of the
accused.

Issue:
​Whether or not what the police officers conducted an entrapment or instigation
Ruling:
Accused-appellants were caught in flagrante delicto in a buy-bust operation. A buy-bust
operation is a form of entrapment whereby ways and means are resorted to for the purpose of trapping
and capturing the lawbreakers in the execution of their criminal plan. Unless there is clear and
convincing evidence that the members of the buy-bust team were inspired by any improper motive or
were not properly performing their duty, their testimony on the operation deserves full faith and
credit. When the police officers involved in the buy-bust operation have no motive to falsely testify
against the accused, the courts shall uphold the presumption that they have performed their duties
regularly. It is correctly upheld the testimony of the prosecution witnesses, the police officers who
conducted the buy-bust operation. It did not err in applying the presumption of regularity in the
performance of duty by law enforcement agents. ​
​SPO1 Facto, the poseur-buyer, gave the complete details of how the transaction was conducted
from beginning to end -- the negotiation between the confidential agent and the drug dealers, the
preparation made by the buy-bust team before conducting the operation, when the informant introduced
him as the supposed buyer to the drug dealers, the exchange of the stuff and the payment between the
pushers and the poseur buyer, and the arrest of said drug dealers. SPO1 Facto positively identified
accused-appellants as the drug dealers.
​In a buy-bust operation, the idea to commit a crime originates from the offender, without
anybody inducing or prodding him to commit the offense. Its opposite is instigation or inducement,
wherein the police or its agent lures the accused into committing the offense in order to prosecute
him. Instigation is deemed contrary to public policy and considered an absolutory cause. In this case,
accused-appellants, apparently, have, for some time, been engaged in drug dealing. They were in fact the
subject of a surveillance conducted by the operatives of the PNP Narcotics Group. The police engaged
the services of a confidential informant to lead them to transact with them. The confidential agent
facilitated the meeting of accused-appellants and the poseur buyer. Hence, it was neither the police nor
the confidential agent who induced accused-appellants to commit a violation of the Dangerous Drugs
Law. They were already violating the law and the police only used the buy-bust operation to apprehend
them in the act of unlawfully selling drugs. This is certainly a legitimate entrapment operation and not
instigation.
​In accordance with Article 83 of the Revised Penal Code, as amended by Section 25 of Republic
Act No. 7659, upon finality of this decision, let the records of these cases be forwarded to the Office of
the President for possible exercise of executive clemency.
Case 29 – Gabi
PCGG vs. NAVARRO-GUTIERREZ
GR No. 194159 21 October 2015
Doctrines:
Probable Cause; It must be emphasized that in determining the elements of the crime charged for
purposes of arriving at a finding of probable cause, only facts sufficient to support a prima facie case
against the respondents are required, not absolute certainty.
Preliminary Investigation; Probable Cause; The conduct of preliminary investigation proceedings
whether by the Ombudsman or by a public prosecutor — is geared only to determine whether or not
probable cause exists to hold an accused-respondent for trial for the supposed crime that he committed.
Preliminary investigation is merely an inquisitorial mode of discovering whether or not there is
reasonable basis to believe that a crime has been committed and that the person charged should be held
responsible for it. Being merely based on opinion and belief, a finding of probable cause does not
require an inquiry as to whether there is sufficient evidence to secure a conviction.
Preliminary Investigation; Hearsay Evidence Rule; Hearsay evidence is admissible in determining
probable cause in preliminary investigations because such investigation is merely preliminary and does
not finally adjudicate rights and obligations of parties.

Facts:
On 8 October 1993, then President Ramos issued Administrative Order No. 14 creating the
Presidential Ad Hoc Fact-finding Committee on Behest Loans. The same was created to identify the
behest loans entered into by the past administrations. Subsequently, Memorandum No. 619 was issued.
It laid down the criteria to aid the said ad hoc committee in the determination of whether or not a loan is
behest in character.
The ad hoc committee and the technical working group then examined documents evidencing
loans granted to different institutions during the administration of former President Marcos. Among
those was a series of loans granted by the Developmental Bank of the Philippines (DBP) to the National
Galleon Shipping Corporation.
In total, the value of Galleon's obligations to DBP amounted to Php 2,039,284,390.85 while the
value of its collaterals was merely Php 539,000,000. The ad hoc committee concluded that the loans
obtained by Galleon from DBP possessed positive characteristics of behest loans, considering that: (a)
Galleon was undercapitalized; (b) the loan was undercollateralized; (c) the major stockholders of
Galleon were known to be Marcos cronies; and (d) certain documents pertaining to the loans were found
to bear marginal notes of the former President Marcos.
The PCGG then filed a criminal complaint before the Ombudsman against the individual
respondents. The respondents contended that the action was already barred by prescription for more than
20 years has passed. Moreover, Roque also averred that he was not a crony of the President Marcos.
The Ombudsman dismissed the criminal complaint filed by the PCGG. It held that the complaint
lacked probable cause as the executive summaries and technical reports were consisted mostly of
hearsay and of little probative value.

Issue:
Whether or not the Ombudsman gravely abused its discretion in finding that there was no
probable cause for the complaint filed by the PCGG against the respondents.

Ruling:
Yes. Ombudsman gravely abused its discretion. The petition of PCGG has merit.
The conduct of preliminary investigation proceedings is geared to determine whether or not
probable cause exists to hold an accused-respondent for trial for the supposed crime that he committed.
In this light, probable cause is defined as such facts as are sufficient to engender a well-founded belief
that a crime has been committed and that respondent is probably guilty of the same. It is merely based
on opinion and reasonable belief. It does not import absolute certainty.
A finding of probably cause needs only to rest on evidence that a crime has been committed by
the suspects. It does not need to be based on clear and convincing evidence of guilt beyond reasonable
doubt. It does not require an inquiry as to whether there is sufficient evidence to secure a conviction.
Hence, the determination of probable cause can rest partially, or even entirely, on hearsay as long as the
person making the hearsay statement is credible and there is substantial basis for the same.
Hearsay evidence is admissible in determining probable cause in a preliminary investigation
because such is merely preliminary and does not finally adjudicate rights and obligations of the parties.
The ad hoc committee, as contained by the narration of facts in their reports, found that the loans
or accommodations extended by the DBP to Galleon were behest in nature. There was substantial basis
to credit the findings as such were based on official documents prepared by the DBP itself. It was
Ombudsman's error to discredit the findings in the executive summary for being hearsay and of little
probative value.

Case 30 – Huggins
DE LIMA vs REYES
GR No. 209330 January 11, 2016

Doctrine:
A preliminary investigation is merely preparatory to a trial, and it is not a trial on the merits. Hence,
any alleged irregularity in an investigation's conduct does not render the information void nor impair its
validity.
Facts:
Mario Joel T. Reyes is alleged to be one of the suspects for the murder of Dr. Gerardo Ortega.
Secretary of Justice, Leila De Lima, issued a Department Order creating a special panel of prosecutors to
conduct preliminary investigation. Subsequently, Dr. Inocenio-Ortega, Dr. Ortega’s wife, filed a
Supplemental Affidavit-Complaint implicating former Governor Reyes as the mastermind of her
husband’s murder. The First Panel dismissed the affidavit-complaint. Dr. Inocencio-Ortega filed a
Motion to Re-open Preliminary Investigation which sought the admission of mobile phone
communications between Governor Reyes and Edrad. In September 2011, the Secretary of Justice issued
Department Order No. 710 creating a new panel of investigators to conduct reinvestigation of the case.
Pursuant to Department Order No. 710, the Second Panel issued a Subpoena requiring former
Governor Reyes to appear before them on October 6 and 13, 2011 and to submit his counter-affidavit
and supporting evidence. On March 12, 2012, the Second Panel issued the Resolution finding probable
cause and recommending the filing of Informations on all accused, including former Governor Reyes.

Reyes questioned the issuance of the assailed Department Order before the Court of Appeals. The Court
of Appeals found that she should have modified or reversed the Resolutions of the First Panel pursuant
,
to the 2000 NPS Rule on Appeal instead of creating a Second Panel. Hence, the petition.

Issue:
Whether or not the re-investigation ordered by the Secretary of Justice is proper
Ruling:
Yes, the re-investigation ordered by the Secretary of Justice is proper. The Court found that the
Secretary of Justice issued Department Order No. 710 because she had reason to believe that the First
Panel's refusal to admit the additional evidence may cause a probable miscarriage of justice to the
parties. The Second Panel was created not to overturn the findings and recommendations of the First
Panel but to make sure that all the evidence, including the evidence that the First Panel refused to admit,
was investigated. Therefore, the Secretary of Justice did not act in an arbitrary and despotic manner, by
reason of passion or personal hostility.
The Court also explained that a preliminary investigation is merely preparatory to a trial. It is not
a trial on the merits. An accused's right to a preliminary investigation is merely statutory and it is not a
right guaranteed by the Constitution. Hence, any alleged irregularity in an investigation's conduct does
not render the information void nor impair its validity.
Also, the trial court has already determined that probable cause exists for the issuance of the
warrant of arrest against Reyes. Jurisdiction over the case, therefore, has transferred to the trial court.
Hence, the petition for certiorari questioning the validity of the preliminary investigation in any other
venue has been rendered moot by the issuance of the warrant of arrest and the conduct of the
arraignment.

Case 31 – Marcelo

AMADO I. SARAUM vs. PEOPLE OF THE PHILIPPINES


781 SCRA January 25, 2016
Doctrine: Valid Warrantless Arrest
Facts:
This is a case against Amado I. Saraum who was charged with violation of Section 12, Article II
(Possession of Paraphernalia for Dangerous Drugs) of Republic Act (R.A.) No. 9165, or the
Comprehensive Dangerous Drugs Act of 2002.
According to the prosecution, a telephone call was received by PO3 Larrobis regarding the
illegal drug activities in Sitio Camansi, Barangay Lorega, Cebu City.Hence, a buy-bust team was then
formed against a certain "Pata.". After preparing all the necessary documents, the team proceeded to the
subject area.
In the operation, “Pata" eluded arrest as he tried to run towards his shanty. Inside the house,
which was divided with a curtain as partition, the buy-bust team also saw Saraum and Peter Esperanza,
who were holding drug paraphernalia apparently in preparation to have a "shabu" pot session. Then, they
recovered from Saraum some paraphernalias such as lighter, rolled tissue paper, and aluminum tin foil
(tooter). After the case was filed, the subject items were turned over to the property custodian of the
Office of City Prosecutor.
In his arraignment, Saraum, with the assistance of a counsel, pleaded not guilty to the offense
charged. Trial ensued.
As per Saraum, he denied the commission of the alleged offense. He testified that on the date and
time in question, he was passing by Lorega Cemetery on his way to the house of his parents-in-law
when he was held by men with firearms. They were already with "Antik" and "Pata," both of whom
were his neighbors. Believing that he had not committed anything illegal, he resisted the arrest. He
learned of the criminal charge only when he was brought to the court.
The trial court rendered its Decision finding Saraum guilty beyond reasonable doubt of the crime
of violation of Section 12, Article II of R.A. 9165. The Court of Appeals sustained the judgment of
conviction.
Hence, this petition.
Issue:
Whether or not, the arrest of Saraum was valid?
Ruling:
Yes, the arrest of Saraum was valid. According to the Supreme Court, the case is clearly one of
hot pursuit of "Pata," who, in eluding arrest, entered the shanty where Saraum and Esperanza were
incidentally caught in possession of the illegal items. Saraum did not proffer any satisfactory explanation
with regard to his presence at the vicinity of the buy-bust operation and his possession of the seized
items that he claims to have "countless, lawful uses."
Considering that Saraum failed to show any arbitrariness, palpable error, or capriciousness on the
findings of fact of the trial and appellate courts, such findings deserve great weight and are deemed
conclusive and binding.
Another reason is that Saraum was arrested during the commission of a crime, which instance
does not require a warrant in accordance with Section 5 (a), Rule 113 of the Revised Rules on Criminal
Procedure. In arrest in flagrante delicto, the accused is apprehended at the very moment he is
committing or attempting to commit or has just committed an offense in the presence of the arresting
officer. To constitute a valid in flagrante delicto arrest, two requisites must concur: (1) the person to be
arrested must execute an overt act indicating that he has just committed, is actually committing, or is
attempting to commit a crime; and (2) such overt act is done in the presence or within the view of the
arresting officer.
Therefore, the arrest is within the ambit of the law, hence, a valid arrest.
Case 32 – Martinez

COMERCIANTE vs PEOPLE
GR No. 205926, July 22, 2015
Doctrines:
Normally, “stop and frisk” searches do not give the law enforcer an opportunity to confer with a judge
to determine probable cause. “Stop and frisk” searches (sometimes referred to as Terry searches) are
necessary for law enforcement. That is, law enforcers should be given the legal arsenal to prevent the
commission of offenses. However, this should be balanced with the need to protect the privacy of citizens
in accordance with Article III, Section 2 of the Constitution. The balance lies in the concept of
“suspiciousness” present where the police officer finds himself or herself in. This may be undoubtedly
based on the experience of the police officer. Experienced police officers have personal experience
dealing with criminals and criminal behavior. Hence, they should have the ability to discern —based on
facts that they themselves observe — whether an individual is acting in a suspicious manner. Clearly, a
basic criterion would be that the police officer, with his or her personal knowledge, must observe the
facts leading to the suspicion of an illicit act. Normally, “stop and frisk” searches do not give the law
enforcer an opportunity to confer with a judge to determine probable cause.
In the absence of such warrant, such search and seizure becomes, as a general rule, “unreasonable”
within the meaning of said constitutional provision. Section 2, Article III of the Constitution mandates
that a search and seizure must be carried out through or on the strength of a judicial warrant predicated
upon the existence of probable cause; in the absence of such warrant, such search and seizure becomes,
as a general rule, “unreasonable” within the meaning of said constitutional provision. To protect people
from unreasonable searches and seizures, Section 3(2), Article III of the Constitution provides an
exclusionary rule which instructs that evidence obtained and confiscated on the occasion of such
unreasonable searches and seizures are deemed tainted and should be excluded for being the proverbial
fruit of a poisonous tree. In other words, evidence obtained from unreasonable searches and seizures
shall be inadmissible in evidence for any purpose in any proceeding.
The law requires that there first be a lawful arrest before a search can be made. The process cannot be
reversed.—The exclusionary rule is not, however, an absolute and rigid proscription. One of the
recognized exceptions established by jurisprudence is a search incident to a lawful arrest. In this
instance, the law requires that there first be a lawful arrest before a search can be made — the process
cannot be reversed. Section 5, Rule 113 of the Revised Rules on Criminal Procedure lays down the rules
on lawful warrantless arrests, as follows: SEC. 5. Arrest without warrant; when lawful.—A peace officer
or a private person may, without a warrant, arrest a person: (a) When, in his presence, the person to be
arrested has committed, is actually committing, or is attempting to commit an offense; (b) When an
offense has just been committed and he has probable cause to believe based on personal knowledge of
facts or circumstances that the person to be arrested has committed it; and (c) When the person to be
arrested is a prisoner who has escaped from a penal establishment or place where he is serving final
judgment or is temporarily confined while his case is pending, or has escaped while being transferred
from one confinement to another. In cases falling under paragraphs (a) and (b) above, the person
arrested without a warrant shall be forthwith delivered to the nearest police station or jail and shall be
proceeded against in accordance with Section 7 of Rule 112.

For a warrantless arrest under Section 5(a) to operate, two (2) elements must concur, namely: (a) the
person to be arrested must execute an overt act indicating that he has just committed, is actually
committing, or is attempting to commit a crime; and (b) such overt act is done in the presence or within
the view of the arresting officer.

Facts:
Per the prosecution, at 10PM, 2 agents of the narcotics group Agent Radan and PO3 Calag II,
while riding their motorcycle at about 30km per hour, spotted two men behaving suspiciously while
standing from 10 meters away. Dasilla and Comerciante were allegedly handing plastic sachets to each
other. They stopped and approached, allegedly confiscated 2 plastic containers of shabu and the two
were subsequently charged with possession of illegal drugs. After the prosecution rested its case, Dasilla
filed a demurrer to evidence, granted by RTC therefore acquitting him. Comerciante failed to file
demurrer, hence the RTC considered his right waived and ordered him to present evidence. He refuted
the agents statements stating that they were merely standing in front of a jeepney, arrested and taken to a
police station. There, the police officers claimed to have confiscated illegal drugs from them and were
asked money in exchange for their release. When they failed to accede to the demand, they were brought
to another police station to undergo inquest proceedings, and thereafter, were charged with illegal
possession of dangerous drugs.
RTC found Comerciante guilty beyond reasonable doubt. CA affirmed stating that it was in flagrante
delicto.
Issue:
Whether there had been a valid warrantless arrest, hence valid search and seizure.

Ruling:
Section 2, Article III of the Constitution mandates that a search and seizure must be carried out
through or on the strength of a judicial warrant predicated upon the existence of probable cause; in the
absence of such warrant, such search and seizure becomes, as a general rule, "unreasonable" within the
meaning of said constitutional provision. To protect people from unreasonable searches and seizures,
Section 3 (2), Article III of the Constitution provides an exclusionary rule which instructs that evidence
obtained and confiscated on the occasion of such unreasonable searches and seizures are deemed tainted
and should be excluded for being the proverbial fruit of a poisonous tree. In other words, evidence
obtained from unreasonable searches and seizures shall be inadmissible in evidence for any purpose in
any proceeding. The exclusionary rule is not, however, an absolute and rigid proscription. One of the
recognized exceptions established by jurisprudence is a search incident to a lawful arrest. In this
instance, the law requires that there first be a lawful arrest before a search can be made the process
cannot be reversed. Section 5, Rule 113 of the Revised Rules on Criminal Procedure lays down the rules
on lawful warrantless arrests.
For a warrantless arrest under Section 5 (a) to operate, two (2) elements must concur, namely (a) the
person to be arrested must execute an overt act indicating that he has just committed, is actually
committing, or is attempting to commit a crime; and (b) such overt act is done in the presence or within
the view of the arresting officer. However, Section 5 (b) requires for its application that at the time of the
arrest, an offense had in fact just been committed and the arresting officer had personal knowledge of
facts indicating that the accused had committed it. In both instances, the officer's personal knowledge of
the fact of the commission of an offense is absolutely required. Under Section 5 (a), the officer himself
witnesses the crime; while in Section 5 (b), he knows for a fact that a crime has just been committed.
On review, there could have been no warrantless arrest made on Comerciante. PO3 Calag himself
admitted that he was aboard a motorcycle cruising at a speed of around 30 kilometers per hour when he
saw Comerciante and Dasilla standing around and showing "improper and unpleasant movements," with
one of them handing plastic sachets to the other. On the basis of the foregoing, he decided to effect an
arrest. PO3 Calag's testimony on direct examination is revelatory and it can be deduced that they could
not have reasonable spotted the two while standing from distance of 10 meters, it was late night and the
motorbike was moving at a speed of 30kph.

Case 33 – Masungsong
LUZ vs. PEOPLE OF THE PHILIPPINES
GR No. 197788 February 29, 2012

Doctrine : Warrantless Arrests (Sec. 5); Delivery to the nearest police station or precinct

Facts:
On March 10, 2003, PO2 Alteza was assigned at the Sub-Station 1 of the Naga City Police
Station as a traffic enforcer. Then he saw the accused, which was coming from the direction of
Panganiban Drive and going to Diversion Road, Naga City, driving a motorcycle without a helmet. This
prompted the police officer to stop Luz for violating a municipal ordinance which requires all
motorcycle drivers to wear helmet while driving the motorcycle. Thereafter the accused was invited to
the police station while in the police station he was issued a citation ticket by SPO1 Brillante for
violation of the said ordinance, he saw the accused was uneasy and kept on getting something on his
jacket and the police officer was alerted and so, he told the accuse to take out the contents of his jacket.
The police officer saw a weapon and a container take out into his jacket, and then the accused was to
open the said container upon the instruction the police officer, the accused spilled out the content of the
container which turned out to be four plastic sachets; two of which contained suspected shabu while the
other two were empty.
The RTC convicted Luz of illegal possession of dangerous drugs. The prosecution shows that he
had been lawfully arrested for a traffic violation and then subjected to a valid search, which led to the
discovery of two plastic sachets contained suspected shabu. The CA affirmed the decision of the RTC.
The accused, however, contends that the there was no lawful search and seizure, because there was no
lawful arrest. He claims that the finding that there was a lawful arrest was erroneous, since he was not
even issued a citation ticket or charged with violation of the city ordinance. Even assuming there was a
valid arrest, he claims that he had never consented to the search conducted upon him.
But the RTC held thus: It is beyond dispute that the accused was flagged down and apprehended
in this case by Police Officers Alteza and Brillante for violation of City Ordinance No. 98-012, an
ordinance requiring the use of crash helmet by motorcycle drivers and riders thereon in the City of Naga
and prescribing penalties for violation thereof. The accused himself admitted that he was not wearing a
helmet at the time when he was flagged down by the said police officers, albeit he had a helmet in his
possession. Obviously, there is legal basis on the part of the apprehending officers to flag down and
arrest the accused because the latter was actually committing a crime in their presence, that is, a
violation of City Ordinance No. 98-012. In other words, the accused, being caught in flagrante
delicto violating the said Ordinance, he could therefore be lawfully stopped or arrested by the
apprehending officers.

Issue:
1) Whether or not there was a valid arrest of the accused?
ACTIONS of the Court:
RTC: The appellant was guilty for illegal possession dangerous drugs.
CA: Affirmed the decision of the RTC.
SC: The decision of the CA was reversed the judgment and acquits the appellant.
Ruling:

No, there was no valid arrest of the accused. When he was flagged down for committing a traffic
violation, he was not, ipso facto and solely for this reason, arrested.

Arrest is the taking of a person into custody in order that he or she may be bound to answer for
the commission of an offense. It is effected by an actual restraint of the person to be arrested or by that
persons voluntary submission to the custody of the one making the arrest. Neither the application of
actual force, manual touching of the body, or physical restraint, nor a formal declaration of arrest, is
required. It is enough that there be an intention on the part of one of the parties to arrest the other, and
that there be an intent on the part of the other to submit, under the belief and impression that submission
is necessary.

Under R.A. 4136, or the Land Transportation and Traffic Code, the general procedure for dealing
with a traffic violation is not the arrest of the offender, but the confiscation of the drivers license of the
latter:

SECTION 29. Confiscation of Driver's License. Law enforcement and peace officers of
other agencies duly deputized by the Director shall, in apprehending a driver for any violation of
this Act or any regulations issued pursuant thereto, or of local traffic rules and regulations not
contrary to any provisions of this Act, confiscate the license of the driver concerned and issue a
receipt prescribed and issued by the Bureau therefore which shall authorize the driver to operate
a motor vehicle for a period not exceeding seventy-two hours from the time and date of issue of
said receipt. The period so fixed in the receipt shall not be extended, and shall become invalid
thereafter. Failure of the driver to settle his case within fifteen days from the date of
apprehension will be a ground for the suspension and/or revocation of his license.

Similarly, the Philippine National Police (PNP) Operations Manual provides the following
procedure for flagging down vehicles during the conduct of checkpoints:

SECTION 7. Procedure in Flagging Down or Accosting Vehicles While in Mobile Car.


This rule is a general concept and will not apply in hot pursuit operations. The mobile car crew
shall undertake the following, when applicable:

If it concerns traffic violations, immediately issue a Traffic Citation Ticket (TCT) or Traffic
Violation Report (TVR). Never indulge in prolonged, unnecessary conversation or argument with
the driver or any of the vehicles occupants;

At the time that he was waiting for PO3 Alteza to write his citation ticket, petitioner could not be
said to have been under arrest. There was no intention on the part of PO3 Alteza to arrest him, deprive
him of his liberty, or take him into custody. Prior to the issuance of the ticket, the period during which
petitioner was at the police station may be characterized merely as waiting time. In fact, as found by the
trial court, PO3 Alteza himself testified that the only reason they went to the police sub-station was that
petitioner had been flagged down almost in front of that place. Hence, it was only for the sake of
convenience that they were waiting there. There was no intention to take petitioner into custody.

WHEREFORE, the Petition is GRANTED. The 18 February 2011 Decision of the Court of
Appeals in CA-G.R. CR No. 32516 affirming the judgment of conviction dated 19 February 2009 of the
th
Regional Trial Court, 5 Judicial Region, Naga City, Branch 21, in Criminal Case No. RTC 2003-0087,
is hereby REVERSED and SET ASIDE. Petitioner Rodel Luz y Ong is hereby ACQUITTED and
ordered immediately released from detention, unless his continued confinement is warranted by some
other cause or ground.

Case 34 – Morada

GEORGE ANTIQUERA Y CODES vs. PEOPLE OF THE PHILIPPINES


G.R. No. 180661 December 11, 2013

Doctrine:
In Flagrante Delicto Section 5(a), Rule 113 of the Rules of Criminal Procedure provides that a “peace
officer or a private person may, without a warrant, arrest a person when, in his presence, the person to
be arrested has committed, is actually committing, or is attempting to commit an offense.”
–Section 5(a), Rule 113 of the Rules of Criminal Procedure provides that a “peace officer or a private
person may, without a warrant, arrest a person when, in his presence, the person to be arrested has
committed, is actually committing, or is attempting to commit an offense.” This is an arrest in flagrante
delicto. The overt act constituting the crime is done in the presence or within the view of the arresting
officer.
Same; Same; Searches and Seizures; Considering that his arrest was illegal, the search and seizure that
resulted from it was likewise illegal. Consequently, the various drug paraphernalia that the police
officers allegedly found in the house and seized are inadmissible, having proceeded from an invalid
search and seizure.
–Clearly, no crime was plainly exposed to the view of the arresting officers that authorized the arrest of
accused Antiquera without warrant under the above-mentioned rule. Considering that his arrest was
illegal, the search and seizure that resulted from it was likewise illegal. Consequently, the various drug
paraphernalia that the police officers allegedly found in the house and seized are inadmissible, having
proceeded from an invalid search and seizure. Since the confiscated drug paraphernalia is the very
corpus delicti of the crime charged, the Court has no choice but to acquit the accused. One final note.
The failure of the accused to object to the irregularity of his arrest by itself is not enough to sustain his
conviction. A waiver of an illegal warrantless arrest does not carry with it a waiver of the
inadmissibility of evidence seized during the illegal warrantless arrest.

Facts:
On 11 February 2004, PO1 Gregorio Recio, PO1 Laurence Cabutihan, P/Insp. Eric Ibon, PO1
Rodelio Rania, and two other civilian operatives on board a patrol car and a tricycle were conducting a
police visibility patrol on David Street, Pasay City, when they saw two unidentified men rush out of a
house and immediately boarded a jeep.
Suspecting that a crime had been committed, the police officers approached the house from
where the men came and peeked through the partially opened door. PO1 Recio and PO1 Cabutihan saw
accused George Antiquera holding an improvised tooter and a pink lighter. Beside him was his live-in
partner, Corazon Cruz, who was holding an aluminum foil and an improvised burner. This prompted the
police officers to enter the house, introduce themselves, and arrest Antiquera and Cruz.While inspecting
the immediate surroundings, PO1 Cabutihan saw a wooden jewelry box on a table. It contained drug
paraphernalia. These paraphernalia were found positive for traces of methamphetamine hydrochloride or
“shabu”.
The Regional Trial Court (RTC) held that the arrest of the two accused without a warrant was
justified pursuant to Section 5, Rule 113 of the Rules of Criminal Procedure. The RTC said that the
prosecution proved beyond reasonable doubt that the police caught both the accused in the act of using
shabu and having drug paraphernalia in their possession.
On appeal, the Court of Appeals affirmed the decision of the RTC.

Issue:
Whether or not the warrantless arrest of Antiquera and Cruz was justified under Section 5, Rule
113 of the Rules of Criminal Procedure.

Ruling:
No. The warrantless arrest of Antiquera and Cruz was not justified under Section 5, Rule 113 of
the Rules of Criminal Procedure. Under the said provision, a “peace officer or a private person may,
without a warrant, arrest a person when, in his presence, the person to be arrested has committed, is
actually committing, or is attempting to commit an offense.” This is an arrest in flagrante delicto, where
the overt act constituting the crime is done in the presence or within the view of the arresting officer.
The Supreme Court found that the circumstances in the case at bar do not constitute an arrest made in
flagrante delicto. The police officers themselves claim that they were alerted when they saw two
unidentified men rush out of a house. This made them suspect that a crime had been committed.
However, instead of running after the two fleeing men, what the police officers did was to go to the
house where the two men came from. Even as they peeked through the partially opened door of the
house, they saw no activity that warranted their entering into it. Thus, it was apparent that no crime was
plainly exposed to the view of the arresting officers that authorized their arrest of Antiquera and Cruz.
Considering that the arrest was illegal, it also follows that the search and seizure which resulted
from it was illegal and that the drug paraphernalia allegedly found inside the house were inadmissible as
evidence.

Case 35 – Profugo
PEOPLE OF THE PHILIPPINES vs. DONALD VASQUEZ
GR No. 200304 January 15, 2014
Doctrine: Warrantless arrest when valid states in Rule 113 Sec 5.
Facts:
Initially the case of illegal possession of drugs was raffled but upon motion it was consolidated
with the case of illegal sale of drugs. On arraignment, the appellant pleaded not guilty to both charges.
The pre-trial conference of the cases was held, but the same was terminated without the parties entering
into any stipulation of facts. During the trial of the case the prosecution stated the events. There was a
confidential informant reported to PO2 Trambulo about the illegal drug activities. Fajardo form a buy-
bust team. It was in the buy-bust operation that Don was arrested.
​RTC convicted the appellant of the crimes charged. The RTC gave more credence to the
prosecution’s evidence given that the presumption of the regularity in the performance of official duty
on the part of the police officers was not overcome. On appeal, the court of Appeals affirmed the
conviction of the appellant. He argues that the police officers, did not have a search warrant or a warrant
of arrest against him. Since his arrest was illegal, the appellant avers that the evidence obtained as a
result thereof was inadmissible in court.
Issue:
Whether or not the appellant Don may assail the validity of arrest.
Ruling:
No, at the outset, the court rule that the appellant can no longer assail the validity of his arrest.
Supreme court held in People vs. Tampis that “any objection, defect or irregularity attending an arrest
must be made before the accused enters his plea on arraignment. Having failed to move for the quashing
of the information against them before their arraignment, appellants are now estopped from questioning
the legality of their arrest. Any irregularity was cured upon their voluntary submission to the trial court’s
jurisdiction. Having established the validity of the warrantless arrest in this case, the court hold that the
warrantless seizure of the illegal drugs from the appellant is likewise valid. SC held in People vs.
Cabagutan that: The interdiction against warrantless searches and seizures have long been deemed
permissible by jurisprudence in instances of 1) search of moving vehicles 2) seizure in plain view 3)
customs searches 4) waiver or consented searches 5) stop and frisk situation and search incidental to a
lawful arrest. The last includes a valid warrantless arrest, for while as a rule, an arrest is considered
legitimate effected with a valid warrant of arrest, Rules of Court recognize permissible warrantless arrest
to wit: 1) arrest in flagrante delicto, 2) arrest effected in hot pursuit, and 3) arrest of escaped prisoners.
Thus, the appellant cannot seek exculpation by invoking belatedly the invalidity of his arrest and the
subsequent search upon his person.
Case 36 – Roxas
FLORESTA vs. UBIADAS
426 SCRA 270
Doctrine:

Whether bail is a matter of right or discretion, and even if no charge has yet been filed in court against a
respondent-suspect-detainee, reasonable notice of hearing is required to be given to the prosecutor, or at
least his recommendation must be sought. So Fortuna v. Penaco-Sitaca instructs: [A]dmission to bail as
a matter of discretion presupposes the exercise thereof in accordance with law and guided by the
applicable legal principles. The prosecution must first be accorded an opportunity to present evidence
because by the very nature of deciding applications for bail, it is on the basis of such evidence that
judicial discretion is weighed against in determining whether the guilt of the accused is strong. In other
words, discretion must be exercised regularly, legally and within the confines of procedural due process,
that is, after the evaluation of the evidence submitted by the prosecution. Any order issued in the absence
thereof is not a product of sound judicial discretion but of whim and caprice and outright arbitrariness.

Facts:

By a Sworn Complaint dated January 24, 2000, then Provincial Prosecutor, Dorentino Z. Floresta
administratively charged Judge Eliodoro G. Ubiadas with “gross ignorance of the law, grave abuse of
authority and violations of the Code of Judicial Conduct.”

Complainant further faults respondent for granting, “without giving notice to the prosecution,”
the petition for bail of Jose Mangohig, Jr. who was arrested by virtue of a warrant which found probable
cause against him for violation of Section 5(b), Art. III of Republic Act No. 7610 (“Special Protection of
Children Against Child Abuse, Exploitation and Discrimination Act”), which is punishable by reclusion
temporal to reclusion perpetua, and subsequently indicted for statutory rape qualified by relationship
which is punishable by death.

The petition for bail of Mangohig who was then under preliminary investigation, which motion
was filed on January 3, 2000 on which same date a copy of said petition was furnished the public
prosecutor, a hearing was set and heard on the morning of January 4, 2000 during which there was no
appearance from the Prosecutor’s Office; and that as the offense for which Mangohig was charged is
ordinarily a bailable offense, respondent granted him bail.
Issue:
Whether or not the respondent Judge erred on granting Mangohig bail on the ground that there is
a failure of notice to the prosecution.

Ruling :

Yes, Judge Ubiadas committed a violation in granting bail to the accused.

Whether bail is a matter of right or discretion, and even if no charge has yet been filed in court
against a respondent- suspect-detainee, reasonable notice of hearing is required to be given to the
prosecutor, or at least his recommendation must be sought.

While in Section 18 of Rule 114 on applications for bail, no period is provided as it merely
requires the court to give a “reasonable notice” of the hearing to the prosecutor or require him to submit
his recommendation, and the general rule on the requirement of a three-day notice for hearing of
motions under Section 4 of Rule 15 allows a court for good cause to set the hearing on shorter notice,
there is, in the case of Mangohig, no showing of good cause to call for hearing his petition for bail on
shorter notice.

Case 37 – Roxas
ZUNO vs. CABEBE
444 SCRA 382
Doctrine :

Under the present Rules, a hearing is mandatory in granting bail whether it is a matter of right or
discretion. It must be stressed that the grant or the denial of bail in cases where bail is a matter of
discretion, hinges on the issue of whether or not the evidence of guilt of the accused is strong, and the
determination of whether or not the evidence is strong is a matter of judicial discretion which remains
with the judge. In order for the latter to properly exercise his discretion, he must first conduct a hearing
to determine whether the evidence of guilt is strong. In fact, even in cases where there is no petition for
bail, a hearing should still be held. There is no question that respondent judge granted bail to the
accused without conducting a hearing, in violation of Sections 8 and 18, Rule 114 of the Revised Rules
of Criminal Procedure.

Facts :

This is an administrative case filed by Chief State Prosecutor Jovenado R. Zuño of the DOJ
against the respondent judge for knowingly rendering an unjust judgment, gross ignorance of the law
and partiality relative to a case for illegal possession of prohibited or regulated drugs. Respondent judge
allegedly granted bail to the accused in the said case without accused's application or motion for bail and
without conducting any hearing. The respondent judge denied the charges. Among others, he argued that
he cannot be held administratively liable for not conducting a hearing because the prosecution did not
object to the grant of bail.

Issue :
Whether or not respondent judge can be held administratively liable for granting bail without
conducting a hearing.

Ruling :

Yes. The Court found the respondent judge guilty of violation of Rule 114 of the Revised Rules
of Criminal Procedure on the grant of bail for which he was fined in the sum of Twenty Thousand Pesos
(P20,000.00). The Court stressed that a hearing is mandatory in granting bail whether it is a matter of
right or discretion. The failure to raise or absence of an objection on the part of the prosecution in
application for bail does not dispense with the requirement of a bail hearing. The Code of Judicial
Conduct enjoins judges to be conversant with the law and the Rules and maintain professional
competence; and by the very nature of his office, should be circumspect in the performance of his duties.
He must render justice without resorting to shortcuts clearly uncalled for. The respondent judge failed to
live up to these standards.
Case 38 – Cajurao
GOVERNMENT OF HONGKONG SPEC. ADM. REGION VS OLALIA
521 SCRA

Doctrine:

The modern trend in public international law is the primacy placed on the worth of the individual person
and the sanctity of human rights. If bail can be granted in deportation cases, the Court sees no
justification why it should not also be allowed in extradition cases—clearly, the right of a prospective
extraditee to apply for bail must be viewed in the light of the various treaty obligations of the
Philippines concerning respect for the promotion and protection of human rights.

Facts:

Private respondent Muñoz was charged before Hong Kong Court. Warrants of arrest were issued
and by virtue of a final decree the validity of the Order of Arrest was upheld. The petitioner Hong Kong

Administrative Region filed a petition for the extradition of the private respondent. In the same case, a
petition for bail was filed by the private respondent. The petition for bail was denied by reason that there
was no Philippine law granting the same in extradition cases and that the respondent was a high “flight
risk”. Private respondent filed a motion for reconsideration and was granted by the respondent judge
subject to the following conditions:

1. Bail is set at Php750,000.00 in cash with the condition that accused hereby undertakes that he will
appear and answer the issues raised in these proceedings and will at all times hold himself amenable to
orders and processes of this court, will further appear for judgment. If accused fails in this undertaking,
the cash bond will be forfeited in favor of the government;

2. Accused must surrender his valid passport to this Court;

3. The Department of Justice is given immediate notice and discretion of filing its own motion for hold
departure order before this Court even in extradition proceeding; and

4. Accused is required to report to the government prosecutors handling this case or if they so desire to
the nearest office, at any time and day of the week; and if they further desire, manifest before this Court
to require that all the assets of accused, real and personal, be filed with this Court soonest, with the
condition that if the accused flees from his undertaking, said assets be forfeited in favor of the
government and that the corresponding lien/annotation be noted therein accordingly.

Petitioner filed a motion to vacate the said order but was denied by the respondent judge. Hence, this
instant petition.

Issue:

Whether or Not a potential extraditee is entitled to post bail

Ruling:
A potential extraditee is entitled to bail. Petitioner alleged that the trial court committed grave
abuse of discretion amounting to lack or excess of jurisdiction in admitting private respondent to bail;
that there is nothing in the Constitution or statutory law providing that a potential extraditee has a right
to bail, the right being limited solely to criminal proceedings. On the other hand, private respondent
maintained that the right to bail guaranteed under the Bill of Rights extends to a prospective extraditee;
and that extradition is a harsh process resulting in a prolonged deprivation of one’s liberty.

In this case, the Court reviewed what was held in Government of United States of America v.
Hon. Guillermo G. Purganan, Presiding Judge, RTC of Manila, Branch 42, and Mark B. Jimenez, a.k.a.
Mario Batacan Crespo GR No. 153675 April 2007, that the constitutional provision on bail does not
apply to extradition proceedings, the same being available only in criminal proceedings. The Court took
cognizance of the following trends in international law:

1. the growing importance of the individual person in public international;


2. the higher value now being given to human rights;
3. the corresponding duty of countries to observe these universal human rights in fulfilling their treaty
obligations; and
4. the duty of this Court to balance the rights of the individual under our fundamental law, on one hand,
and
the law on extradition, on the other.

In light of the recent developments in international law, where emphasis is given to the worth of
the individual and the sanctity of human rights, the Court departed from the ruling in Purganan, and
held that an extraditee may be allowed to post bail.

Case 39 – Morada

PEOPLE OF THE PHILIPPINES vs. SANDIGANBAYAN (FIRST DIVISION), VICTORINO A.


BASCO, ROMEO S. DAVID, and ROGELIO L. LUIS
G.R. No. 164577. July 5, 2010.*

Doctrine:
Double Jeopardy; Appeals; The rule barring an appeal from a judgment of acquittal is, however, not
absolute—the following are the recognized exceptions thereto: (i) when the prosecution is denied due
process of law; and (ii) when the trial court commits grave abuse of discretion amounting to lack or
excess of jurisdiction in dismissing a criminal case by granting the accused’ demurrer to evidence.—
Procedurally, the petitioner resorted to a wrong remedy. Section 1 of Rule 122 allows “any party” to
appeal from a judgment or final order, unless the right of the accused against double jeopardy will be
violated. It is axiomatic that an appeal in criminal cases throws the whole case wide open for review by
an appellate court. As a consequence, an appeal by the prosecution from a judgment of acquittal
necessarily places the accused in double jeopardy. The rule barring an appeal from a judgment of
acquittal is, however, not absolute. The following are the recognized exceptions thereto: (i) when the
prosecution is denied due process of law; and (ii) when the trial court commits grave abuse of discretion
amounting to lack or excess of jurisdiction in dismissing a criminal case by granting the accused’
demurrer to evidence. Such issues are brought to the attention of a reviewing court through the special
civil action of certiorari under Rule 65 on the ground of grave abuse of discretion, amounting to lack or
excess of jurisdiction. In assailing the resolution of the Sandiganbayan, the petitioner resorted to this
petition for review on certiorari under Rule 45, purportedly raising pure questions of law. This is
erroneous for which reason this petition is dismissible outright.
Demurrer to Evidence; The demurrer to evidence in criminal cases is “filed after the prosecution had
rested its case,” and as such, it calls “for an appreciation of the evidence adduced by the prosecution
and its sufficiency to warrant conviction beyond reasonable doubt, resulting in a dismissal of the case
on the merits, tantamount to an acquittal of the accused.”—The demurrer to evidence in criminal cases,
such as the one at bench, is “filed after the prosecution had rested its case.” As such, it calls “for an
appreciation of the evidence adduced by the prosecution and its sufficiency to warrant conviction
beyond reasonable doubt, resulting in a dismissal of the case on the merits, tantamount to an acquittal
of the accused.” Judicial action on a motion to dismiss or demurrer to evidence is best left to the
exercise of sound judicial discretion. Accordingly, unless the Sandiganbayan acted without jurisdiction
or with grave abuse of discretion, its decision to grant or deny the demurrer may not be disturbed.
Questions of Law; There is a question of law in a given case when the doubt or difference arises as to
what the law is on certain state of facts; Judicial review under Rule 45 does not envisage a re-evaluation
of the sufficiency of the evidence upon which respondent court’s action was predicated; It bears
reiterating that a judgment of acquittal, “even if seemingly erroneous,” is the final verdict.—Granting

arguendo that petitioner’s recourse under Rule 45 was proper, nevertheless, petitioner failed to raise
pure questions of law. For a question to be one of law, the same must not involve an examination of the
probative value of the evidence presented. There is a question of law in a given case when the doubt or
difference arises as to what the law is on certain state of facts. Contrary to petitioner’s contention, the
determination of whether the established facts fall squarely within the provisions of the law, that is,
Section 3 (e) of R.A. No. 3019, would require us to reassess and reexamine the evidence, and essentially
to supplant the lower courts’ finding. This is beyond the province of Rule 45. Judicial review under Rule
45 does not envisage a re-evaluation of the sufficiency of the evidence upon which respondent court’s
action was predicated. It bears reiterating that a judgment of acquittal, “even if seemingly erroneous,”
is the final verdict.
Criminal Law; Administrative Law; If the criminal case will be prosecuted based on the same facts and
evidence as that in the administrative case, and the court trying the latter already squarely ruled on the
absence of facts and/or circumstances sufficient to negate the basis of the criminal indictment, then to
still burden the accused to present controverting evidence despite the failure useless exercise.—Although
the dismissal of the criminal case cannot be pleaded to abate the administrative proceedings primarily
on the ground that the quantum of proof required to sustain administrative charges is significantly lower
than that necessary for criminal actions, the same does not hold true if it were the other way around,
that is, the dismissal of the administrative case is being invoked to abate the criminal case. The reason is
that the evidence presented in the administrative case may not necessarily be the same evidence to be
presented in the criminal case. The prosecution is certainly not precluded from adducing additional
evidence to discharge the burden of proof required in the criminal cases. However, if the criminal case
will be prosecuted based on the same facts and evidence as that in the administrative case, and the court
trying the latter already squarely ruled on the absence of facts and/or circumstances sufficient to negate
the basis of the criminal indictment, then to still burden the accused to present controverting evidence
despite the failure of the prosecution to present sufficient and competent evidence, will be a futile and
useless exercise.

Facts:
Respondents were charged for having violated Section 3(e) of Republic Act No. 3019(anti- graft
and corrupt practices act) before the Sandiganbayan. That the accused being high ranking officials did
then and there, willfully, unlawfully and criminally enter into contracts/transactions for the construction
of the Mabalacat-Clark Spur Road and the Clark Perimeter Road, without the benefit of public bidding
and at the price higher by 60 to 167% than the typical roadway construction cost, thus, depriving the
government of the opportunity of obtaining the most advantageous construction cost. During the trial,
the prosecution presented its lone witness, Atty. Pagunuran, legal counsel of the Office of the
Ombudsman. Consequently, instead of presenting their evidence, the respondents filed their respective
motions for leave to file their demurrer to evidence and alleged that the witness had no personal
knowledge of the transaction and thus it’s a hearsay and that the prosecution failed to prove that there
was an overpricing and that the ruling of the Court of Appeals in an administrative case (C.A. G.R. SP
No. 62084), which upheld the validity of the direct negotiated contracts, even in the absence of a public
bidding, was already the law of the case. The demurrer to evidence was however denied. It opined that
the prosecution’s evidence substantiated the elements of the crime and that the respondents must present
controverting evidence. The respondents then filed a MR and was granted and the case was dismissed.
The SB based its ruling on the ground that that there being want of substantial evidence to support an
administrative charge, there could be no sufficient evidence to warrant a conclusion that there is
probable cause for a violation of Section 3(e) of R.A. No. 3019. Thus this petition.
Issue:
Whether or not an appeal can be made

Ruling:
Procedurally, the petitioner resorted to a wrong remedy. Section 1 of Rule 122 allows "any party"
to appeal from a judgment or final order, unless the right of the accused against double jeopardy will be
violated. It is axiomatic that an appeal in criminal cases throws the whole case wide open for review by
an appellate court. As a consequence, an appeal by the prosecution from a judgment of acquittal
necessarily places the accused in double jeopardy. The rule barring an appeal from a judgment of
acquittal is, however, not absolute. The following are the recognized exceptions thereto: (i) when the
prosecution is denied due process of law; 12 and (ii) when the trial court commits grave abuse of
discretion amounting to lack or excess of jurisdiction in dismissing a criminal case by granting the
accused’ demurrer to evidence. Where appeal is available, certiorari will not prosper. In the dismissal of
a criminal case upon demurrer to evidence, appeal is not available as such an appeal will put the accused
in double jeopardy. Certiorari, however, is allowed.

Case 40 – Fetilo

TERESITA TANGHAL OKABE vs. HON. PEDRO DE LEON GUTIERREZ


G.R. No. 150185. May 27, 2004

Doctrine:

Section 26, Rule 114 of the Revised Rules on Criminal Procedure. Bail not a bar to objections on illegal
arrest, lack of or irregular preliminary investigation.— An application for or admission to bail shall not
bar the accused from challenging the validity of his arrest or the legality of the warrant issued therefor,
or from assailing the regularity or questioning the absence of a preliminary investigation of the charge
against him, provided that he raises them before entering his plea. The court shall resolve the matter as
early as practicable but not later than the start of the trial of the case.

Rights Of The Accused - Right to be Informed. It is essential that the accused of be informed of the
facts that are imputed to him as “he is presumed to have no independent knowledge of the facts that
constitute the offense.” The noble object of written accusations cannot be over emphasized. This was
explained in the US v. Karelsen: “the object of this written accusations was – First. To furnish the
accused with such a description of the charge against him as will enable to make his defense; and
second to avail himself of his conviction or acquittal for protection against a further prosecution of the
same cause; and third, to inform the court of the facts alleged, so that it may decide whether they are
sufficient in law to support a conviction if one should be had. In order that this requirement that may be
satisfied, facts must be stated, not conclusions of law.

Facts:

Cecilia Maruyama filed a case, charging Lorna Tanghal and petitioner Teresita Tanghal Okabe,
a.k.a. Shiela Okabe, with estafa.

Maruyama alleged that, on December 11, 1998, she entrusted Y11, 410,000.00 with the peso
equivalent of P3,993,500.00 to the petitioner, who was engaged in the business of “door-to-door
delivery” from Japan to the Philippines. It was alleged that the petitioner failed to deliver the money as
agreed upon, and at first, denied receiving the said amount but later returned only 1,000 US Dollar
through Lorna Tanghal. During the preliminary investigation, the complainant submitted the affidavit of
nd
her witnesses and other documentary evidence. After requisite preliminary investigation, 2 Assistant
City Prosecutor, Joselito J. Vibandor came out with a resolution, finding probable cause for estafa
against the petitioner which was subsequently approved by the City Prosecutor. The trial court then
issued a warrant for the arrest of the petitioner with a recommended bond of P40,000. Petitioner posted a
personal bail bond in the said amount.

The petitioner left the Philippines for Japan on June 17, 2000 without the trial court’s permission,
and returned to the Philippines on June 28, 2000. She left the Philippines anew on July 1, 2000, and
returned on July 12, 2000. Private prosecutor filed an urgent ex parte motion for the issuance of the hold
departure order. Trial Court approved the same. Meanwhile, the petitioner filed a verified motion for
judicial determination of probable cause and to defer proceedings/arraignment, alleging that the only
documents appended to the Information submitted by the investigating prosecutor were respondent
Maruyama’s complaint-affidavit for estafa and the resolution of the investigating prosecutor; the
affidavits of the witnesses of the complainant; the respondent’s counter-affidavit and the other evidence
adduced by the parties were not attached thereto. On July 19, 2000, the petitioner also filed a Very
Urgent Motion To lift/Recall Hold Departure Order dated July 17, 2000 and/or allow her to regularly
travel to Japan. Petitioner also questioned the irregularity of the determination of probable cause during
the preliminary investigation.

Petitioner filed a manifestation objecting to her arraignment prior to the resolution of her pending
motions. She alleged that her arraignment for the crime charged should not be made a condition for the
granting of her motion to recall the hold departure order issued against her.
Petitioner filed a motion for the postponement of her arraignment alleging that, in case the trial
court ruled adversely thereon, she would refuse to enter a plea and seek relief from the appellate court.
The court denied the petitioners motions on the ground that when she posted a personal bail bond for her
provisional liberty, she thereby waived her right to question the courts finding of the existence of
probable cause for her arrest and submitted herself to the jurisdiction of the court, more so when she
filed the motion for the lifting of the hold departure order the court issued, and the motion to defer the
proceedings and her arraignment.

Issue:

Whether the Court of Appeals committed a reversible error in not applying Section 26, Rule 114
of the Revised Rules on Criminal Procedure.

Ruling:

Yes. The Court of Appeals committed a reversible error in not applying Section 26, Rule 114 of
the Revised Rules on Criminal Procedure.

The court The court agrees with the contention of the petitioner. It bears stressing that Section
26, Rule 114 of the Revised Rules on Criminal Procedure is a new one, intended to modify previous
rulings of this Court that an application for bail or the admission to bail by the accused shall be
considered as a waiver of his right to assail the warrant issued for his arrest on the legalities or
irregularities thereon. This new rule is curative in nature because precisely, it was designed to supply
defects and curb evils in procedural rules. Hence, the rules governing curative statutes are applicable.
Curative statutes are by their essence retroactive in application.

Moreover, considering the conduct of the petitioner after posting her personal bail bond, it
cannot be argued that she waived her right to question the finding of probable cause and to assail the
warrant of arrest issued against her by the respondent judge. There must be clear and convincing proof
that the petitioner had an actual intention to relinquish her right to question the existence of probable
cause. When the only proof of intention rests on what a party does, his act should be so manifestly
consistent with, and indicative of, an intent to voluntarily and unequivocally relinquish the particular
right that no other explanation of his conduct is possible.

Thus, the posting of a personal bail bond was a matter of imperative necessity to avert her
incarceration; it should not be deemed as a waiver of her right to assail her arrest. So this Court ruled in
People v. Red: In view of these circumstances and the number of the accused, it may properly be held
that the furnishing of the bond was prompted by the sheer necessity of not remaining in detention, and in
no way implied their waiver of any right, such as the summary examination of the case before their
detention. That they had no intention of waiving this right is clear from their motion of January 23,
1929, the same day on which they furnished a bond, and the fact that they renewed this petition on
February 23, 1929, praying for the stay of their arrest for lack of the summary examination;
Case 41– Garcia
LEVISTE vs CA
615 SCRA 619

Doctrine
The discretionary nature of bail mentioned in Section 5 of Rule 114 does not mean automatic grant of
bail in case of appeal.
Facts:
Charged with the murder of Rafael de las Alas, petitioner Jose Antonio Leviste was convicted by
the Regional Trial Court of Makati City for the lesser crime of homicide and sentenced to suffer an
indeterminate penalty of six years and one day of prision mayor as minimum to 12 years and one day of
reclusion temporal as maximum. He appealed his conviction to the Court of Appeals. Pending appeal, he
filed an urgent application for admission to bail pending appeal, citing his advanced age and health
condition, and claiming the absence of any risk or possibility of flight on his part. The Court of Appeals
denied petitioner’s application for bail. It invoked the bedrock principle in the matter of bail pending
appeal, that the discretion to extend bail during the course of appeal should be exercised “with grave
caution and only for strong reasons.” Petitioner now questions as grave abuse of discretion the denial of
his application for bail, considering that none of the conditions justifying denial of bail under the third
paragraph of Section 5, Rule 114 of the Rules of Court was present. Petitioner’s theory is that, where the
penalty imposed by the trial court is more than six years but not more than 20 years and the
circumstances mentioned in the third paragraph of Section 5 are absent, bail must be granted to an
appellant pending appeal.
Issue:
In an application for bail pending appeal by an appellant sentenced by the trial court to a penalty
of imprisonment for more than six years, does the discretionary nature of the grant of bail pending
appeal mean that bail should automatically be granted absent any of the circumstances mentioned in the
third paragraph of Section 5, Rule 114 of the Rules of Court?

Ruling:
No, after conviction by the trial court, the presumption of innocence terminates and, accordingly,
the constitutional right to bail ends. From then on, the grant of bail is subject to judicial discretion. At
the risk of being repetitious, such discretion must be exercised with grave caution and only for strong
reasons. Considering that the accused was in fact convicted by the trial court, allowance of bail pending
appeal should be guided by a stringent-standards approach. This judicial disposition finds strong support
in the history and evolution of the rules on bail and the language of Section 5, Rule 114 of the Rules of
Court. It is likewise consistent with the trial court’s initial determination that the accused should be in
prison. Furthermore, letting the accused out on bail despite his conviction may destroy the deterrent
effect of our criminal laws. This is especially germane to bail pending appeal because long delays often
separate sentencing in the trial court and appellate review. In addition, at the post-conviction stage, the
accused faces a certain prison sentence and thus may be more likely to flee regardless of bail bonds or
other release conditions. Finally, permitting bail too freely in spite of conviction invites frivolous and
time-wasting appeals which will make a mockery of our criminal justice system and court processes.

Case 42 – Lasam Jr.


JUAN PONCE ENRILE vs. SANDIGANBAYAN
G.R. No. 213847 August 18, 2015

Doctrines:
Any person, before being convicted of any criminal offense, shall be bailable, unless he is charged with
a capital offense, or with an offense punishable with reclusion perpetua or life imprisonment and the
evidence of guilt is strong.
All criminal cases within the competence of the Metropolitan Trial Court, Municipal Trial Court,
Municipal Trial Court in Cities or Municipal Circuit Trial Court are bailable as a matter of right
because these courts have no jurisdiction to try capital offenses, or offenses punishable by reclusion
perpetua or life imprisonment.
The granting of bail is discretionary: a) upon conviction by the RTC of an offense not punishable by
death, reclusion perpetua or life imprisonment; or b) if the RTC has imposed a penalty of imprisonment
exceeding 6 years, provided none of the circumstances enumerated under paragraph 3 of Section 5, Rule
114 is present.

Facts:
On June 5, 2014, the Office of the Ombudsman charged Enrile and several others with plunder in
the Sandiganbayan on the basis of their purported involvement in the diversion and misuse of
appropriations under the Priority Development Assistance Fund (PDAF).
On July 3, 2014, the Sandiganbayan issued its resolution denying Enrile’s motion, particularly on
the matter of bail, on the ground of its prematurity considering that Enrile had not yet then voluntarily
surrendered or been placed under the custody of the law.
On the same day that the warrant for his arrest was issued, Enrile voluntarily surrendered to
Director Benjamin Magalong of the Criminal Investigation and Detection Group (CIDG) in Camp
Crame,
Enrile filed his Motion for Detention at the PNP General Hospital, and his Motion to Fix Bail
both dated July 7, 2014, which were heard by the Sandiganbayan on July 8, 2014.
Enrile argued that he should be allowed to post bail because:
The Prosecution had not yet established that the evidence of his guilt was strong;
Although he was charged with plunder, the penalty as to him would only be reclusion temporal, not
reclusion perpetua ; and
He was not a flight risk, and his age and physical condition must further be seriously considered.
Sandiganbayan issued its first assailed resolution denying Enrile’s Motion
Issue:
Whether or not Enrile should be granted bail.

Ruling:
Bail protects the right of the accused to due process and to be presumed innocent
The presumption of innocence is rooted in the guarantee of due process. The purpose of bail is to
guarantee the appearance of the accused at the trial, or whenever so required by the trial court. Thus, bail
acts as a reconciling mechanism to accommodate both the accused’s interest in his provisional liberty
before or during the trial, and the society’s interest in assuring the accused’s presence at trial.
Bail may be granted as a matter of right or of discretion.
Section 7, Rule 114 provides Capital offense or an offense punishable by reclusion perpetua or
life imprisonment, not bailable. — No person charged with a capital offense, or an offense
punishable by reclusion perpetua or life imprisonment, shall be admitted to bail when evidence
of guilt is strong, regardless of the stage of the criminal prosecution.A capital offense in the
context of the rule refers to an offense that, under the law existing at the time of its commission
and the application for admission to bail, may be punished with death.
In all criminal cases within the jurisdiction of MTC or MCTC bail is a matter of right since these
courts has no jurisdiction to try cases with capital offenses or offenses punishable by reclusion perpetua
or life imprisonment.
However in RTC it is only a matter of right prior to conviction for any offense not punishable by
death, reclusion perpetua , or life imprisonment, or even prior to conviction for an offense punishable by
death, reclusion perpetua , or life imprisonment when evidence of guilt is not strong. It becomes
discretionary for RTC when the following circumstances are attendant:
Upon conviction by the RTC of an offense not punishable by death, reclusion perpetua or life
imprisonment;
If the RTC has imposed a penalty of imprisonment exceeding six years, provided none of the
circumstances enumerated under paragraph 3 of Section 5, Rule 114 is present, as follows:

1. That he is a recidivist, quasi-recidivist, or habitual delinquent, or has committed the crime


aggravated by the circumstance of reiteration;
2. That he has previously escaped from legal confinement, evaded sentence, or violated the
conditions of his bail without valid justification
3. That he committed the offense while under probation, parole, or conditional pardon;

4. That the circumstances of hi s case indicate the probability of flight if released on bail

5. That there is undue risk that he may commit another crime during the pendency of the appeal.

6. Admission to bail in offenses punished by death, or life imprisonment, or reclusion perpetua is


subject to judicial discretion.

In resolving bail applications of the accused who is charged with a capital offense, or an offense
punishable by reclusion perpetua or life imprisonment, the trial judge is expected to comply with the
guidelines outlined in Cortes v. Catral:

In all cases, whether bail is a matter of right or of discretion, notify the prosecutor of the hearing of the
application for bail or require him to submit his recommendation (Section 18, Rule 114 of the Rules of
Court, as amended);

Where bail is a matter of discretion, conduct a hearing of the application for bail regardless of whether
or not the prosecution refuses to present evidence to show that the guilt of the accused is strong for the
purpose of enabling the court to exercise its sound discretion; (Section 7 and 8, supra)

Decide whether the guilt of the accused is strong based on the summary of evidence of the prosecution;

If the guilt of the accused is no t strong, discharge the accused upon the approval of the bailbond

Case 43 – Roxas
DEL CASTILLO vs. PEOPLE OF THE PHILIPPINES
GR NO. 185128 January 30, 2012

Doctrine:

While it is not necessary that the property to be searched or seized should be owned by the person
against whom the search warrant is issued, there must be sufficient showing that the property is under
appellant’s control or possession. The CA, in its Decision, referred to the possession of regulated drugs
by the petitioner as a constructive one. Constructive possession exists when the drug is under the
dominion and control of the accused or when he has the right to exercise dominion and control over the
place where it is found. The records are void of any evidence to show that petitioner owns the nipa hut in
question nor was it established that he used the said structure as a shop. The RTC, as well as the CA,
merely presumed that petitioner used the said structure due to the presence of electrical materials, the
petitioner being an electrician by profession.

The accused, in all criminal prosecutions, is presumed innocent of the charge laid unless the contrary is
proven beyond reasonable doubt.—In considering a criminal case, it is critical to start with the law’s
own starting perspective on the status of the accused—in all criminal prosecutions, he is presumed
innocent of the charge laid unless the contrary is proven beyond reasonable doubt. Proof beyond
reasonable doubt, or that quantum of proof sufficient to produce a moral certainty that would convince
and satisfy the conscience of those who act in judgment, is indispensable to overcome the constitutional
presumption of innocence.

Facts:

Pursuant to a confidential information that Del Castillo was engaged in selling shabu, the police
officers conducted surveillance and test buy operation at the house of Del Castillo. The authorities later
secured a search warrant from the RTC. Upon arrival, somebody shouted “raid” which prompted them to
immediately disembark the jeep and went directly to the house of Del Castillo. The said house is a two-
storey house and Del Castillo was staying in the second floor. When they went upstairs, the police
operatives informed the wife of Del Castillo that they will implement the search warrant. However,
before they can search the area, Del Castillo ran towards a nipa hut in front of his house. The police
chased him but failed, because they were not familiar with the entrance and exit of the place. They all
went back to the house of Del Castillo, and searched the area in the presence of a barangay tanod and the
elder sister of Del Castillo. They found nothing in the house however one barangay tanod was able to
confiscate from the nipa hut several plastic packs contacting crystalline substance which was later found
out to be shabu.

Issue:

Whether or not there is a violation of petitioner’s constitutional rights against unreasonable


search and presumption of innocence.

Ruling:

Yes. The search warrant specifically designates the residence of Del Castillo as the place to be
searched. Incidentally, the items were seized by a barangay tanod in a nipa hut 20 meters away from the
residence of the petitioner. The confiscated items, having found in a place other than the one described
in the search warrant, can be considered fruits of an invalid warrantless search, and the presentation of
which as an evidence is a violation of constitutional rights of Del Castillo against unreasonable searches
and seizure.

While it is not necessary that the property to be searched or seized should be owned by the
person against whom the search warrant is issued, there must be sufficient showing that the property is
under appellant’s control or possession. The CA, in its Decision, referred to the possession of regulated
drugs by the petitioner as a constructive one. Constructive possession exists when the drug is under the
dominion and control of the accused or when he has the right to exercise dominion and control over the
place where it is found. The records are void of any evidence to show that petitioner owns the nipa hut
in question nor was it established that he used the said structure as a shop. The RTC, as well as the CA,
merely presumed that petitioner used the said structure due to the presence of electrical materials, the
petitioner being an electrician by profession.

In considering a criminal case, it is critical to start with the law’s own starting perspective on the
status of the accused—in all criminal prosecutions, he is presumed innocent of the charge laid unless the
contrary is proven beyond reasonable doubt. Proof beyond reasonable doubt, or that quantum of proof
sufficient to produce a moral certainty that would convince and satisfy the conscience of those who act
in judgment, is indispensable to overcome the constitutional presumption of innocence.

Case 44 – Lukban

PEOPLE OF THE PHIPPINES vs. ARTURO LARA y ORBISTA


G.R. No. 199877 August 13, 2012
Doctrine:

Criminal Procedure; Courts; Jurisdiction; Jurisdiction over the person of the accused may be acquired
through compulsory process such as a warrant of arrest or through his voluntary appearance, such as
when he surrenders to the police or to the court. Any objection to the arrest or acquisition of jurisdiction
over the person of the accused must be made before he enters his plea, otherwise the objection is
deemed waived.- Jurisdiction over the person of the accused may be acquired through compulsory
process such as a warrant of arrest or through his voluntary appearance, such as when he surrenders to
the police or to the court. Any objection to the arrest or acquisition of jurisdiction over the person of the
accused must be made before he enters his plea, otherwise the objection is deemed waived. An accused
submits to the jurisdiction of the trial court upon entering a plea and participating actively in the trial
and this precludes him invoking any irregularities that may have attended his arrest. Furthermore, the
illegal arrest of an accused is not a sufficient ground to reverse and set aside a conviction that was
arrived upon a complaint duly filed and a trial conducted without error.
Facts:
Lara is charged with robbery with homicide filed in the RTC. The ground of the charge was for
the robbery from an accounting staff of San Sebastian Allied Services, Enrique Sumulong, and the
killing of Joselito M. Bautista whilst doing the robbery.
On May 31, 2001 at around 9 am, Sumulong withdrew 230K from Metrobank-Mabini Branch,
Pasig City to defray the salaries of employees of San Sebastian. Going to the bank, he rode a pick-up
accompanied by Virgilio Manacob, Jeff Atie and Joselito Baustista. Upon withdrawal, he put the money
in a black bag and immediately left. At around 10:30 am, in an intersection Lara appeared at the front of
the passenger side and pointed a gun to Sumulong asking him to give him the money. Baustista at the
back said not to and Sumulong threw the bag at Bautista’s direction. Bautista went out with the bag,
Lara followed. Sumulong went towards Mercedez Plaza and called up the office of San Sebastian to
relay the accident. When he went back, he saw blood on the ground. The bystanders said that Bautista
was shot and the bag was taken from him. While on his way to San Miguel, Pasig City, he saw Lara
walking and alerted the police and thereafter he was arrested. Atie and Manacob testified against Lara.
On Lara’s defense, on May 31, 2001, he was at his house digging a sewer trench while his
brother was working on constructing a comfort room. They worked from 8am until 9am. At around June
7, 2001, while he was at his cousin’s, police officers arrived, ask and confirmed that he is Arturo Lara.
The police asked him to come with them to the barangay hall. He was brought to the police station
instead of the barangay hall where he was investigated for robbery with homicide. He told the police that
he was at home when the subject took place. His statement was challenged and was asked to produce
witnesses. The police told Sumulong and Atie to point on Lara so they can go home already. Witnesses
arrived the next day and Lara was told that he will be subjected to inquest.
In the RTC, Lara was convicted. He appealed to CA but the latter affirmed his conviction.
Issue:
Whether or Not the identification made by Sumulong, Atie, and Manacob in the police line-up is
inadmissible because Lara stood there in without the assistance of counsel
Ruling:
NO. When he was placed in a police line-up did not validate the proceedings leading to his conviction.
That he stood at the police line-up without the assistance of counsel did not render Sumulong’s
identification of Lara inadmissible. The right to counsel is deemed to have arisen at the precise moment
custodial investigation begins and being made to stand in a police line-up is not the starting point or a
part of custodial investigation. As the court previously ruled in People v. Amestuzo:
The guarantees of Sec. 12 (1), Art. III of the 1987 Constitution or the so-called Miranda Rights, may be
invoked only by a person while he is under custodial investigation. Custodial investigation starts when
the police investigation is no longer a general inquiry into an unsolved crime but has began to focus on
particular suspect taken into custody by the police who starts the interrogation and propounds questions
to the person to elicit incriminating statements. Police line-up is not part of the custodial investigation;
hence the right to counsel guaranteed by the Constitution cannot yet be invoked at this stage.

Case 45– Marcos


JOSE SANICO vs. PEOPLE OF THE PHILIPPINES
G.R. No. 198753 March 25, 2015

Doctrine:
Although the right to appeal is statutory, it must be respected and observed because it is an essential
component of due process.

Facts:
​Jose Sanico was charged with trespassing and theft of minerals in the Municipal Circuit Trial
Court of Catmon-Sogod, Cebu. After trial, the MCTC convicted him and his co-accused, Marsito
Batiguin, with violation of Section 103 of the Philippine Mining Act of 1995, but acquitted them of
trespassing. It also awarded damages in favor of the private complainant, Jennifer Tenio. Thus, on April
22, 2009, Jose’s counsel filed a Notice of Appeal of the MCTC Judgment On January 5. 2010, the RTC
ordered him to file his memorandum on appeal, to which Jose did not comply, hence upon motion of the
private complainant, the RTC dismissed his appeal. Another lawyer for Jose, Atty Canete, filed his
motion for reconsideration, averring that Jose was preoccupied with the condition of his ailing wife who
subsequently died, likewise, his counsel Atty. Baring suffered from a medical condition which caused
her to forget how she got the case and whom to contact as principal counsel. The RTC denied the motion
for its lack of verification and affidavitt of senvice. Thus, his counsel filed a petition for review with the
Court of Appeals. This time, the CA dismissed the petition for review for various reasons: (a) the docket
fees were not paid: (b) there was no proper proof of service of a copy of the petition for review on the
adverse party; (b) the petitioner did not fumish to the RTC a copy of the petition for review; (d) there
was no affidavit of service; (e) no written explanation for not resorting to personal filing was filed; (f)
the documents appended to the petition were only plain photocopies of the certified true copies; (g) no
copies of pleadings and other material portions of the record were attached; (h) the verification and
certification of non-forum shopping were defective due to failure to contain a statement that the
allegations therein were based on the petitioner's personal knowledge; (i) the venfication and
certification of non-forum shopping did not contain competent evidence of identity of the petitioner; and
(j) the serial number of the commission of the notary public and the office address of the notary public
were not properly indicated.

​On motion for reconsideration, the CA denied it, assuring that Jose was bound by the mistakes of
his counsel In the meantime, the RTC granted Jennifer Tenio moved for issuance of writ of execution,
and the record of the case indicated that it was granted and several properties of Jose were levied on
execution

​Jose elevated his case to the Supreme Court. He argues that the RTC erred in dismissing his
appeal for non-filing of he memorandum on appeal. The CA erred in not reversing the RTC orders, and

in finding him liable for the mistakes of his counsels.

Issue:
​Whether or not the non-filing of the Memorandum of Appeal by Jose merits the dismissal of his
appeal by the RTC.

Ruling:
​No. The failure to file the memorandum on appeal is a ground for the RTC to dismiss the appeal
only in civil cases. The same rule does not apply in criminal cases, because Section 9(c), imposes on the
RTC the duty to decide the appeal "on the basis of the entire record of the case and of such memoranda
or briefs as may have been filed" upon the submission of the appellate memoranda or briefs, or upon the
expiration of the period to file the same. Hence, the dismissal of the petitioner‘s appeal cannot be
properly premised on the failure to file the memorandum on appeal

​Having timely perfected his appeal by filing the notice of appeal in the MCTC. the petitioner was
entitled to expect that the RTC would resolve his appeal in due course, whether he filed his
memorandum on appeal or not. The unwarranted dismissal of the appeal by the RTC was, therefore, an
outright denial of due process to him in a manner that occasioned severe prejudice.

Case 46– Casiano

THE PEOPLE OF THE PHILIPPINES vs. JUDGE RUBEN AYSON


G.R. No. 85215. July 7, 1989
Doctrine:
Rights during custodial investigation does not encompass statements made during an administrative

inquiry

Facts:
Felipe Ramos was a ticket freight clerk of the Philippine Airlines (PAL), assigned at its Baguio
City. The audit by PAL found irregularities in the sales of plane ticket which involved Ramos. PAL
management notified him that investigation will be conducted to investigate the matter. On the day of
investigation Ramos gave his superiors handwritten notes to which he offers to pay the amount. During
the investigation conducted by PAL Branch Manager - Baguio Edgardo Cruz. Ramos was in informed
about the findings of the audit team and his answers were taken down in writing. Ramos didn’t not made
any disclosures of the tickets but the proceeds has been misused the funds. As such he offered to pay
back the money in staggered basis.
Two months later, information was filed against Felipe Ramos charging him for the crime of
Estafa. On arraignment Ramos entered a plea of Not Guilty and trial ensued. During the trial, private
prosecutors of PAL made a written offer of evidence which included the statement and written note
given by Ramos taken during PAL's investigation mark as exhibit K and A.

Judge Ayson admitted all evidence except exhibit K and A. He declared that those are inadmissible
evidence since it does not appear that accused was reminded of his constitutional rights to remain silent
and to have counsel, and that when he waived the same and gave his statement, it was without the
assistance actually of a counsel. Therefore, its inadmissible. Private prosecutors moved for
reconsideration but was denied.

Issue:
Whether or Not it was grave abuse of discretion for respondent Judge to have excluded the
People's Exhibits A and K?
Ruling:
Yes, it is clear from the undisputed facts of this case that Felipe Ramos was not in any sense
under custodial interrogation. The constitutional rights of a person under custodial interrogation under
Section 20, Article IV of the 1973 Constitution is not applicable and not relavant to the inquiry.
Ramos had voluntarily answered questions posed to him on the first day of the administrative
investigation and agreed that the proceedings should be recorded, the record having thereafter been
marked during the trial of the criminal action subsequently filed against him as Exhibit A, just as it is
obvious that the note (later marked as Exhibit K) that he sent to his superiors on February 8, 1986, the
day before the investigation, offering to compromise his liability in the alleged irregularities, was a free
and even spontaneous act on his part. Since Ramos is not under custodial investigation Judge Ayson
should have accepted the evidence since the Miranda rights afforded during custodial investigation is not
applicable in the case.
Case 47– Mortel

VILLARUEL vs. PEOPLE OF THE PHILIPPINES


G.R. No. 151258 February 1, 2012

Doctrine:
Right to speedy trial is deemed violated when proceeding is with unjustified postponements of trial.
Facts: ​
In February 1991, seven freshmen law students of the Ateneo de Manila University School of
Law signified their intention to join Aquila Legis Juris Fraternity (Aquila Fraternity). They were
Asuncion, Belleza, Marquez III, Navera, Recinto, Sy Jr., and Villa (neophytes). On the night of 8
February 1991, the neophytes were met by some members of the Aquilans. They went to the house of
Michael Musngi, also an Aquilan, who briefed the neophytes on what to expect during the initiation
rites. The latter were informed that there would be physical beatings, and that they could quit at any
time. After their briefing, they were brought to Almeda Compund in Caloocan City for the
commencement of their initiation.
Even before the neophytes got off the van, they had already received threats and insults from the
Aquilans. The neophytes were then subjected to traditional forms of Aquilan initiation rites. And so they
survived their first day of initiation. On the morning of their second day, the neophytes were subjected to
the same manner of hazing that they endured on the first day of initiation. After a few hours, the
initiation for the day officially ended. After a while, accused non-resident alumni fraternity members
Fidelito Dizon and Artemio Villareal demanded that the rites be re-opened. Upon the insistence, it was
re-opened, the fraternity members, including Dizon and Villareal, and then subjected neophytes to
paddling and additional rounds of physical pain. Villa received several paddle blows, one of which was
strong; it sent him sprawling to the ground which Villa could no longer walk. Again, the initiation was
officially ended. They then slept at the carport. After an hour of sleep, Villa’s condition worsened, so the
Aquilans rushed him to the hospital, but Villa was pronounced dead on arrival.
Consequently, a criminal case for homicide was filed against 35 Aquilans. Twenty-six of the
accused in Crim. Case No. C-38340 (91) were jointly tried. On the other hand, trial against the
remaining nine accused in Crim. Case No. C-38340 was held in abeyance. The trial court rendered
judgment in Crim. Case No. C-38340 (91), holding the 26 accused guilty beyond reasonable doubt.
However, the CA set aside the finding of the trial court.
On August 2002, the trial court in Crim. Case No. C-38340 dismissed the charge against accused
Concepcion on the ground of violation of his right to speedy trial, while it denied the respective Motions
to Dismiss of accused Escalona, Ramos, Saruca, and Adriano. The CA, however, reversed the decision
and dismissed the criminal case against the four accused also on the basis of violation of their right to
speedy trial.
Issue: ​ ​
Whether the CA committed grave abuse of discretion when it dismissed the case against
Escalona, Ramos, Saruca, and Adriano for the violation of their right to speedy trial.

Ruling: ​
No. The right of the accused to speedy trial has been enshrined in Sections 14(2) and 16, Art. III,
1987 Constitution. This right requires that there be a trial free from vexatious, capricious, or oppressive
delays. The right is deemed violated when the proceeding is attended with unjustified postponements of
trial, or when a long period of time is allowed to elapse without the case being tried and for no cause or
justifiable motive. As in this case, on November 1993, they were all arraigned. Unfortunately, the initial
trial of the case did not commence until March 2005 or almost 12 years after arraignment.

Case 48– Marcelo


PEOPLE OF THE PHILIPPINES vs. ESTOMACA
G.R. Nos. 117485-86. April 22, 1996

DOCTRINE: Arraignment and Please, an Integral Aspect of the Due Process Clause of the
Constitution.

FACTS:

​The case involve the heinous crime of rape and were repressed by the sentence of death. The
parties are Melchor Estomaca, an illiterate laborer, who rape his eldest daughter, Melita for five times
from July 1993 to March 1994. Thereafter, the trial continues.

According to the records of the case, Estomaca entered a plea of guilty to two (2) cases and not
guilty to three (3) cases during the arraignment.

The court provided a clearer appreciation of what actually transpired in the so-called arraignment of
appellant in the court Accused was not specifically warned that on his plea of guilty, he would definitely
and in any event be given the death penalty under the New Law, as the trial court calls Republic Act No.
7659. He was also not categorically advised that his plea of guilty would not under any circumstance
affect or reduce the death sentence as he may have believed or may have been erroneously advised.
Also, the significance of this distinction is found right in the provisions of Section 1(a) of Rule 116

which, cognizant of the aforestated linguistic variations, deliberately required that the complaint or
information be read to the accused in the language or the dialect known to him, to ensure his
comprehension of the charges.

Issue:

Whether or not, the method of arraignment held in the by the trial court in the case constitutes an
invalid procedure?

Ruling:

Yes, the method of arraignment held in the by the trial court in the case constitutes an invalid
procedure. According to the Supreme Court the application of the rule in criminal case deserved the
paramount importance of a valid arraignment, it being the stage where the issues are joined in the
criminal action and without which the proceedings cannot advance further or, if held, will otherwise be
void.

Further, according to Section 1(a) of Rule 116 requires that the arraignment should be made in
open court by the judge himself or by the clerk of court furnishing the accused a copy of the complaint
or information with the list of witnesses stated therein, then reading the same in the language or dialect
that is known to him, and asking him what his plea is to the charge.The requirement that the reading be
made in a language or dialect that the accused understands and knows is a mandatory requirement,

Here in this case, the Court is then left to speculate on whether all five criminal complaints were
actually read, translated or explained to appellant on a level within his comprehension, considering his
limited education. The Supreme Court then finds that not the Ilonggo dialect is known to the accused but
rather the kinaray-a, a local dialect of native of San Joaquin, where the crime has committed.

Furthermore, questions of these nature are undoubtedly crucial and no truer is this than in the
case of appellant for, again, the original records and rollo of this case now under review are completely
bereft of any document or record concerning his apprehension, detention and prior
investigation, whether custodial or preliminary.

Finally, the bottom line of the rule is that a plea of guilt must be based on a free and informed
judgment. Thus, the searching inquiry of the trial court must be focused on: (1) the voluntariness of the
plea; and (2) the full comprehension of the consequences of the plea.

The Supreme Court states that, tt cannot, therefore, hold a life forfeit, no matter how despicable
the offender, when effective protection for his basic rights was denied because of poverty or ignorance.
Case 49– Oineza
PEOPLE OF THE PHILIPPINES vs. ALFREDO PANGILINAN
G.R. NO. 171020; MARCH 14, 2007
Doctrine:
The arrest of the accused and not his arraignment confers jurisdiction over his person to the courts.
Absence of arraignment can be cured by the active participation of the counsel of the accused by reason
that it is an implication that the accused is fully aware of the charges against him.
Facts:
Two informations were charged against Alfredo Pangilinan for allegedly raping AAA, his
daughter. In 1997, Prosecution formally offered its evidence. In 1998, petitioner filed for bail which was
denied and in 1999 the case was submitted for a decision.
The RTC found petitioner guilty beyond reasonable doubt for two counts of rape and was upheld
by the CA even though finding that the same was not arraigned in the Trial Courts; however,
arraignment was made after the submission of the decision. By reason of the irregularity of improper
arraignment, he argues that there is a procedural error which is prejudicial to him and is tantamount to
denial of his constitutional right to be informed of the nature and cause of the accusation against him for
the purpose of setting up an intelligible defense. He claims that his subsequent arraignment did not cure
the defect in the trial proceedings because at the time the petition for bail was heard, the trial court had
not yet acquired jurisdiction over his person. Hence the petitioner filed for a petition for review with the
Supreme Court.
Issue:
Whether or Not the Courts have no jurisdiction over the person of Pangilinan by reason that he
was not properly arraigned?
Ruling:
Yes, Pangilinan is mistaken.
When the hearings for his petition for bail were conducted, the trial court had already acquired
jurisdiction over his person. Settled is the rule that jurisdiction over the person of the accused is acquired
upon his arrest or voluntary appearance before the court. In the case at bar, the trial court acquired
jurisdiction over the person of Pangilinan when he was arrested notwithstanding Pangilinan sought for
an affirmative relief in the form of bail. His arrest, not his arraignment conferred on the trial court
jurisdiction over his person.
Case 50– Olaivar
DAAN VS SANDIGANBAYAN
560 SCRA 233

Doctrine:
Plea bargaining in criminal cases is a process whereby the accused and the prosecution work out a
mutually satisfactory disposition of the case subject to court approval. It usually involves the defendant's
pleading guilty to a lesser offense or to only one or some of the counts of a multi-count indictment in
return for a lighter sentence than that for the graver charge.
During the pre-trial stage,trial court's exercise of its discretion should neither be arbitrary nor should it
amount to a capricious and whimsical exercise of discretion. Grave abuse of discretion implies such
capricious and whimsical exercise of judgment as is equivalent to lack of jurisdiction or, in other words,
where the power is exercised in an arbitrary manner by reason of passion, prejudice, or personal
hostility; and it must be so patent or gross as to amount to an evasion of a positive duty or to a virtual
refusal to perform the duty enjoined by law, or to act at all in contemplation of law.

Facts:
Joselito Daan together with Benedicto Kuizon, were charged for three counts of malversation of
public funds involving the sums of P3,293.00, P1,869.00, and P13,528.00, respectively, which they
purportedly tried to conceal by falsifying the time book and payrolls for given period making it appear
that some laborers worked on the construction of the new municipal hall building of Bato, Leyte and
collected their respective salaries when, in truth and in fact, they did not. In addition to the charge for
malversation, the accused were also charged of three counts of falsification of public document by a
public officer or employee. the accused offered to withdraw their plea of not guilty and substitute the
same with a plea of guilty, provided the mitigating circumstances of confession or plea of guilt and
voluntary surrender will be appreciated in their favor.if the proposal is not acceptable, said accused
proposed instead to substitute their plea of not guilty to the crime of falsification of public document by
a public officer or employee with a plea of guilty but to the lesser crime of falsification of a public
document by a private individual. In the malversation cases, the accused offered to substitute their plea
of not guilty thereto with a plea of guilty, but to the lesser crime of failure of an accountable officer to
render accounts. The Sandiganbayan,assailed its Resolution,denying petitioners Motion to Plea Bargain,
despite favorable recommendation by the prosecution on the main ground that no cogent reason was
presented to justify its approval. Petitioner argues that the Sandiganbayan committed grave abuse of
discretion in denying his plea bargaining offer hence this petition

Issue:
Whether the Sandiganbayan committed grave abuse of discretion in denying his plea bargaining?
Ruling:
Yes, Section 2, Rule 116 of the Rules of Court presents the basic requisites upon which plea
bargaining may be made, that it should be with the consent of the offended party and the prosecutor, and
that the plea of guilt should be to a lesser offense which is necessarily included in the offense charged.
The rules however use word may in the second sentence of Section 2, denoting an exercise of discretion
upon the trial court on whether to allow the accused to make such plea. Trial courts are exhorted to keep
in mind that a plea of guilty for a lighter offense than that actually charged is not supposed to be allowed
as a matter of bargaining or compromise for the convenience of the accused.
In the present case, the Sandiganbayan rejected petitioner's plea offer on the ground that
petitioner and the prosecution failed to demonstrate that the proposal would redound to the benefit of the
public.
Section 5, Rule 120 of the Rules of Court states when an offense includes or is included in the
other, to wit: SEC. 5. When an offense includes or is included in another. An offense charged
necessarily includes the offense proved when some of the essential elements or ingredients of the
former, as alleged in the complaint or information, constitute the latter. And an offense charged is
necessarily included in the offense proved, when the essential ingredients of the former
constitute or form part of those constituting the latter.
Case 51– Remigio

PEOPLE vs. JANJALANI, ET. AL.


639 SCRA 157

Doctrine:
All trial judges must refrain from accepting with alacrity an accused's plea of guilty. An extrajudicial
confession may be given in evidence against the confessant but not against his co-accused as they are
deprived of the opportunity to cross-examine him.

Facts:
After the infamous Valentines Day bombing where a RRCG bus was blown in EDSA corner
Ayala Avenue by alleged members of the Abu Sayyaf members namely Khadaffy Janjalani, Gamal
Baharan, Angelo Trinidad, Gappal Bannah Asali, Rohmat Abdurrohim and other John & Jane Does.
Abu Sayyaf claims it as a present for President Gloria Macapagal-Arroyo. All of them are charged with
Multiple Murder and Multiple Frustrated Murder with only Baharan, Trinidad, Asali, and Rohmat on
custody. Trinidad followed by Baharan and Asali confessed on live television on their involvement in
the attack. On their arraignment for the multiple murder charge Baharan, Trinidad, and Asali all entered
a plea of guilty but upon arraignment for the multiple frustrated murder charge, accused Asali pled
guilty. Accused Trinidad and Baharan pled not guilty. Rohmat pled not guilty to both charges. The trial
court asked whether accused Baharan and Trinidad were amenable to changing their not guilty pleas to
the charge of multiple frustrated murder, considering that they pled guilty to the heavier charge of
multiple murder, creating an apparent inconsistency in their pleas. After the Information was read to
them, Baharan and Trinidad pled guilty to the charge of multiple frustrated murder.
Issues:
1. Whether or Not the trial court erred in the acceptance of the plea of guilt despite the
insufficiency of searching inquiry into the voluntariness and full comprehension of the consequences of
the said plea.
2. Whether or not the trial court gravely erred in finding that the guilt of accused-appellants for
the crimes charged had been proven beyond reasonable doubt.

Ruling:
Yes, The Supreme Court has ruled that all trial judges must refrain from accepting with alacrity
an accused's plea of guilty, for while justice demands a speedy administration, judges are duty bound to
be extra solicitous in seeing to it that when an accused pleads guilty, he understands fully the meaning of
his plea and the import of an inevitable conviction.

No, Section 30, Rule 130 of the Rules of Court applies only to extrajudicial acts or admissions
and not to testimony at trial where the party adversely affected has the opportunity to cross-examine the
declarant. The Court ruled in People v. Buntag, if the declarant repeats the statement in court, his
extrajudicial confession becomes a judicial admission, making the testimony admissible as to both
conspirators

Case 52– Olaivar


ABS-CBN vs GOZON
753 SCRA 1

Doctrine:
Upon motion by the proper party, the arraignment shall be suspended in the following cases: (a) The
accused appears to be suffering from an unsound mental condition which effectively renders him unable
to fully understand the charge against him and to plead intelligently thereto. In such case, the court
shall order his mental examination and, if necessary his confinement for such purpose; (b) There exist a
prejudicial question; and (c) A petition for review of the resolution of the prosecutor is pending at either
the Department of Justice, or the Office of the President; provided, that the period of suspension shall
not exceed sixty (60) days counted from the filing of the petition with the reviewing office.

Facts:
The controversy arose from GMA-7’s news coverage on the homecoming of Filipino overseas
worker and hostage victim Angelo dela Cruz. GMA Network, Inc. and ABS-CBN made their respective
broadcasts and coverage of the live event. ABS-CBN conducted live audio-video coverage and
broadcasted the arrival of Angelo dela Cruz at the Ninoy Aquino International Airport. ABS-CBN
allowed Reuters Television Service to air the footages it had taken earlier under a special embargo
agreement stating that “No other Philippine subscriber of Reuters would be allowed to use ABS- CBN
footage without the latter’s consent”. GMA-7, to which Dela Peña-Reyes and Manalastas (Respondents)

are connected, assigned and stationed news reporters at the NAIA for its live broadcast and non-live
news coverage of the arrival of dela Cruz. GMA-7 subscribes to both Reuters and Cable News Network.
It received a live video feed of the coverage of Angelo dela Cruz’s arrival from Reuters and immediately
carried the live news feed in its program Flash Report prompting ABS-CBN’s filing of Complaint for
copyright infringement of Intellectual Property Code. Assistant City Prosecutor issued a Resolution
finding probable cause indicting respondents. On January 4, 2005, respondents filed the Petition for
Review before the Department of Justice and Secretary Gonzales issued a Resolution referred to as
Gonzales Resolution favoring respondents and held that good faith may be raised as a defense in the
case. Both parties moved for reconsideration of the Gonzales Resolution. On January 19, 2005, the trial
court granted the Motion to Suspend Proceedings filed by respondents. Under Section 11 (c), Rule 116
of the Rules of Criminal Procedure, once a petition for review is filed with the Department of Justice, a
suspension of the criminal proceedings may be allowed by the court. On July 29, 2010 Department of
Justice Under Secretary Agra issued a Resolution referred to as Agra Resolution reversing Gonzales
Resolution.

Issue:
Whether or not the suspension of arraignment/criminal proceedings be allowed.
Ruling:
In Samson v. Daway, this court acknowledged the applicability of Rule 116, Section (c) in a
criminal prosecution for infringement under the Intellectual Property Code. However, this court
emphasized the limits of the order of deferment under the Rule: While the pendency of a petition for
review is a ground for suspension of the arraignment, the provision limits the deferment of the
arraignment to a period of 60 days reckoned from the filing of the petition with the reviewing office. It
follows, therefore, that after the expiration of said period, the trial court is bound to arraign the accused
or to deny the motion to defer arraignment. The suspension of the arraignment should always be within
the limits of the Law. The trial court erred in failing to resume the proceedings after the designated
period.
Case 53 - Sevilla

ENRILE vs PEOPLE
GR No. 213455 August 11, 2015

Doctrine:
A motion to quash is a jurisdictional defect on account that the facts charged in the Information does not
constitute an offense. At any time before entering the plea, the accused may move to quash the
complaint or information.
Sec. 2. Form and contents. – The motion to quash shall be inwriting, signed by the accused or
his counsel and shall specify its factual and legal grounds. The court shall consider no ground
other than those stated in the motion, except lack of jurisdiction over the offense charged.
Sec. 3. Grounds. – The accused may move to quash the complaint or information on any of the
following grounds:

(a) That the facts charged do not constitute an offense;


(b) That the court trying the case has no jurisdiction over the offense charged
(c) That the court trying the case has no jurisdiction over the person of the accused;
(d) That the officer who filed the information had no authority to do so;
(e) That it does not conform substantially to the prescribed form;
(f) That more than one offense is charged except when a single punishment for various offenses is
prescribed by law;
(g) That the criminal action or liability has been extinguished;
(h) That it contains averments which, if true, would constitute a legal excuse or justification; and
(i) That the accused has been previously convicted or acquitted of the offense charged, or the case
against him was dismissed or otherwise terminated without his express consent.

Facts:
On June 5, 2014, the Office of the Ombudsman filed an Information for plunder against Enrile, Jessica
Lucila Reyes, Janet Lim Napoles, Ronald John Lim, and John Raymund de Asis before the
Sandiganbayan.

Enrile responded by filing before the Sandiganbayan (1) an urgent omnibus motion (motion to dismiss
for lack of evidence on record to establish probable cause and ad cautelam motion for bail), and (2)
a supplemental opposition to issuance of warrant of arrest and for dismissal of Information, on June 10,
2014, and June 16, 2014, respectively. The Sandiganbayan heard both motions on June 20, 2014.
On July 10, 2014, Enrile filed a motion for bill of particulars before the Sandiganbayan. On the same
date, he filed a motion for deferment of arraignment since he was to undergo medical examination at the
Philippine General Hospital (PGH).

On July 11, 2014, Enrile was brought to the Sandiganbayan pursuant to the Sandiganbayan’s order and
his motion for bill of particulars was called for hearing.
When the court session resumed, PJ Cabotaje-Tang announced the Court’s denial of Enrile’s motion for
bill of particulars essentially on the following grounds:

the details that Enrile desires are “substantial reiterations” of the arguments he raised in his
(1)
supplemental opposition to the issuance of warrant of arrest and for dismissal of information; and

(2) the details sought are evidentiary in nature and are best ventilated during trial.

The Court denied Enrile’s motion for bill of particulars.


Issue: Whether or not a Motion to Quash, the proper remedy if the information is vague or indefinite
resulting in the serious violation of Enrile’s constitutional right to be informed of the nature and cause of
the accusation against him?
Ruling: No. When allegations in an information are vague or indefinite, the remedy of the accused is
not a motion to quash, but a motion for a bill of particulars.
The purpose of a bill of particulars is to supply vague facts or allegations in the complaint or information
to enable the accused to properly plead and prepare for trial. It presupposes a valid Information, one that
presents all the elements of the crime charged, albeit under vague terms. Notably, the specifications that
a bill of particulars may supply are only formal amendments to the complaint or Information.
Notably, the failure of the accused to move for the specification of the details desired deprives him of
the right to object to evidence that could be introduced and admitted under an Information of more or
less general terms but which sufficiently charges the accused with a definite crime.
Thus, if the Information is lacking, a court should take a liberal attitude towards its granting and order
the government to file a bill of particulars elaborating on the charges. Doubts should be resolved in favor
of granting the bill to give full meaning to the accused’s Constitutionally guaranteed rights.

Notably, the government cannot put the accused in the position of disclosing certain overt acts through
the Information and withholding others subsequently discovered, all of which it intends to prove at the
trial. This is the type of surprise a bill of particulars is designed to avoid. The accused is entitled to the
observance of all the rules designated to bring about a fair verdict. This becomes more relevant in the
present case where the crime charged carries with it the severe penalty of capital punishment and entails
the commission of several predicate criminal acts involving a great number of transactions spread over
a considerable period of time.​
Case 54 - Sim
PEOPLE vs LACSON
400 SCRA 267

Doctrine:
A mere provisional dismissal of a criminal case does not terminate a criminal case. Also, the time bar
rule does not reduce the periods under Art. 90 of the RPC. It is but a limitation of the right of the state to
revive a criminal case against the accused after the Information had been filed but subsequently
provisionally dismissed with the consent of the accused. Time bar also should not be applied
retroactively.

Facts:
Before the court is the petitioner’s motion of reconsideration of the resolution dated May 23,
2002, for the determination of several factual issues relative to the application of Sec. 8 Rule 117 of
RPC on the dismissal of the cases Q-99- 81679 and Q-99-81689 against the respondent. The respondent
was charged with the shooting and killing of eleven male persons known as members of the Kuratong
Baleleng Gang (KBG) in a shootout with police elements near the fly-over along Commonwealth
Avenue, Quezon City. The court confirmed the express consent of the respondent in the provisional
dismissal of the aforementioned cases when he filed for judicial determination. The court also ruled the
need to determine whether the other facts for its application are attendant.

Issues:
1. Whether or not Sec. 8, Rule 117 of 2000 Rules on Criminal Procedure is applicable in the case at bar.
2. WON time-bar in Sec 8 Rule 117 should be applied prospectively or retroactively.

Ruling:
1).No, Section 8, Rule 117 is not applicable to the case since the conditions for its applicability are not
present in this case.
The requisites for provisional dismissal are (1) whether the provisional dismissal of the cases had the
express consent of the accused; (2) whether it was ordered by the court after notice to the offended party,
(3) whether the 2-year period to revive has already lapsed, and (4) whether there is any justification for
the filing of the cases beyond the 2-year period.
In the case at bar, the defendant never filed and denied unequivocally in his statements, through counsel
at the Court of Appeals, that he filed for dismissal nor did he agree to a provisional dismissal thereof.
Also, no notice of motion for provisional dismissal, hearing and subsequent dismissal was given to the
heirs of the victims. Thus provisional dismissal may not apply in this case.

2). Time-bar should not be applied retroactively. Though procedural rules may be applied retroactively,
it should not be if to do so would work injustice or would involve intricate problems of due process.
Statutes should be construed in light of the purposes to be achieved and the evils to be remedied. The
two-year period fixed in the new rule is for the benefit of both the State and the accused. It should not be
emasculated and reduced by an inordinate retroactive application of the time-bar therein provided
merely to benefit the accused. To do so would cause an injustice of hardship to the state and adversely
affect the administration of justice.
Case 55 – Velasco

PANAGUITON VS DOJ
G.R. NO. 167571 NOVEMBER 25, 2008

Doctrine: Remedies against denial/grant of motion

Facts:

This is a petition for Review of CA resolutions dismissing Luis Panaguiton, Jr. petition for
certiorari and motion for reconsideration

In 1992, Cawili borrowed money from petitioner and later issued checks as payment both signed
by Cawili and his business associate Tongson. But checks were dishonored either for insufficiency of

funds or closure of account. Panaguiton then made a formal demands to Cawili and Tongson to pay but
to no avail. So Panaguiton filed a complaint against Cawili and Tongson for violating BP Blg. 22 before
QC Prosecutor's Office.

During Preliminary Investigation, Tongson filed his counter-affidavit claiming that he had been
unjustly included as party-respondent since petitioner had lent money to Cawili in Cawili's personal
capacity. He averred that he was not Cawili's business associate and claimed that he himself has criminal
cases against Cawili. Tongson also denied that he had issued bounced checks and that his signatures on
the checks had been falsified.

As counter, Panaguiton presented documents showing Tongson's signature which was the same
as the signatures on the checks. Panaguiton presented also an affidavit of adverse claim wherein
Tongson claimed to be Cawili's business associate.

December 1995, Prosecutor found probable cause only against Cawili and dismissed the charges
against Tongson. Panaguiton filed a partial appeal before DOJ even the case against Cawili was filed
before the proper court.
Later on July 1997, after finding that Tongson was possible to co-sign the bounced checks and
had altered his signature in pleadings submitted during Preliminary Investigation, Chief State Prosecutor
directed the City Prosecutor of QC to conduct reinvestigation of the case against Tongson and refer the
signatures to NBI.

On March 1999, Asst. City Prosecutor dismissed the complaint against Tongson without
referring to the NBI, holding that the case had already prescribed pursuant to Act. No. 3326, stating that
in this case the 4 year period started on the date the checks were dishonored and that the filing of
complaint in QC prosecutor's office did not interrupt the running of the prescriptive period as the law
contemplates judicial and not administrative proceedings. Four years had elapsed and no information
was filed against Tongson. And the order to refer the matter to NBI could no longer be sanctioned under
Section 3, Rule 112 of rules of criminal procedure because the initiative should come from the petitioner
himself and not from the investigating prosecutor.

Petitioner appealed to DOJ through undersecretary Teehankee but was dismissed. Petitioner then
filed a motion for reconsideration of DOJ and through undersecretary Gutierrez ruled in his favor and
declared that the prescription period was interrupted by the filing of the complaint in the Prosecutor's
office.

However, in August 2004, DOJ acting on the motion for reconsideration filed by Tongson ruled
the subject offense had already prescribed and ordered the withdrawal of 3 informations for violation of
BP Blg. 22 against Tongson. DOJ explained that Act No. 3326 applies to violations of special acts that
do not provide for a prescriptive period for the offenses thereunder.

Panaguiton thus filed a petition for Certiorari before CA assailing the august resolution of the
DOJ, but was dismissed by CA in view of failure to attach a proper verification and certification of non-
forum shopping.

Panaguiton then filed for instant petition claiming that CA committed grave error on dismissing
his petition on technical grounds and in ruling that the petition before it was without merit and questions
are too unsubstantial. The DOJ stated that CA did not err in dismissing the petition for non-compliance
with the rules of court.

Then Cawili and Tongson submitted their comment arguing that CA did not err in dismissing the
petition for certiorari, and they also claim that the offense of violation of BP Blg. 22 has prescribed and
the long delay, attributable to petitioner and the State violated their constitutional right to speedy
disposition of cases. The petition is meritorious.

Isssues:

(1) Technical Issues


(2) Substantive Aspects
Ruling:

1) Verification is merely formal requirement intended to secure an assurance that matters which are
alleged are true and correct-the court may simply order the correction of unverified pleadings or act on
them and waive strict compliance so that the ends of justice may be served. We find that by attaching
pertinent verification to his motion for reconsideration, petitioner has sufficiently complied with the
verification requirement. We also agree that CA erred in dismissing the petition on the ground of failure
to attach a certified copy or duplicate original of the 3 resolution of DOJ.

(2) This court ruled that the filing of the complaint with the fiscal's office for Preliminary Investigation
suspends the running of the prescriptive period. The delay was beyond petitioner's control but that of the
DOJ's flip-flopping resolutions and misapplications.
Case 56-Botor

PEOPLE v DUMLAO
580 SCRA 490

Doctrine:

(1) Insufficiency of evidence is not a ground to quash;

(2) Dismissal on the ground of insufficiency of evidence must be made after the prosecution rests
its case; and

(3) Double jeopardy will not set in when the prosecution was deprived of its opportunity to
prosecute its case and to prove the accused’s culpability.

Facts:

An Amended Information was filed before the Sandiganbayan charging respondents Dumlao and
Lao … with violation of Section 3(g) of Republic Act No. 3019, as amended, otherwise known as the
Anti-Graft and Corrupt Practices Act. As agreed, a Joint Stipulation of Facts and Admission of
Exhibits was submitted to the court. On the basis thereof, the court issued a Pre-Trial Order.

From said Pre-Trial Order, respondent Dumlao filed a Motion to Dismiss/Quash on the ground
[7]
that the facts charged do not constitute an offense. He stated that the prosecutions main thrust against
him was the alleged approval by the GSIS Board of Trustees -- of which he was a member -- of the
Lease-Purchase Agreement entered into by and among the GSIS, the Office of the Government
Corporate Counsel (OGCC) and respondent Lao (buyer-lessee). He argued that the allegedly approved
Board Resolution was not in fact approved by the GSIS Board of Trustees, contrary to the allegations in
the information. Since the signatures of the other Board members did not appear in the minutes of the
meeting, he said it was safe to conclude … that there was no quorum of the board to approve the
supposed resolution authorizing the sale of the GSIS property. The unapproved resolution, he added,
proved his innocence.

The Sandiganbayan issued the assailed resolution. It ruled that: “… In order to validly pass a
resolution at least a majority of four (4) members of the Board of Trustees must sign and approve the
same … Thus, it never had the force and effect of a valid resolution and did not in effect approve the
Lease and Purchase Agreement subject matter hereof.”

Petitioner argues it was denied its right to due process when the court a quo dismissed the case
against respondent Dumlao after pre-trial and before it could present its witnesses and formally offer its
exhibits.
Issue:
Whether or Not the Petitioner deprived of due process when the Sandiganbayan dismissed the
case based on the Pre-Trial Order on the ground of insufficiency of evidence?

Ruling:
Yes. The ground raised by respondent Dumlao in his Motion to Quash/Dismiss is that the facts
charged do not constitute an offense. The fundamental test in determining the sufficiency of the material
averments of an information is whether the facts alleged therein, which are hypothetically admitted,
would establish the essentials elements of the crime defined by law. Evidence aliunde, or matters
[11]
extrinsic of the Information, are not be considered.

Thus, an examination of the facts alleged, if hypothetically admitted, will prove all the elements
of Section 3(g) of Republic Act No. 3019 as against respondent Dumlao.

Further, from the reasoning given by the Sandiganbayan, it is clear that it dismissed the case
because of insufficiency of evidence. Insufficiency of evidence is not one of the grounds of a Motion to
Quash as enumerated in Section 3, Rule 117 of the Revised Rules of Criminal Procedure.

Insufficiency of evidence is a ground for dismissal of an action only after the prosecution rests its
case provided in Section 23, Rule 119 of the Revised Rules of Criminal Procedure.

The Sandiganbayan dismissed the case against respondent for insufficiency of evidence, even
without giving the prosecution the opportunity to present its evidence. In so doing, it violated the
prosecutions right to due process. It deprived the prosecution of its opportunity to prosecute its case and
to prove the accused’s culpability. Respondent Dumlao also contends that to give due course to the
Ombudsmans petition would place him in double jeopardy, in violation of his constitutional rights.

However, double jeopardy has not yet set in. The first jeopardy has not yet attached. The last
element valid conviction, acquittal, dismissal or termination of the case is wanting. The prosecution was
deprived of its opportunity to prosecute its case and to prove the accused’s culpability. The
Sandiganbayan dismissed the case for insufficiency of evidence, while the ground invoked by the
respondent was that the facts charged did not constitute an offense. The dismissal was clearly premature,
because any dismissal based on insufficiency of evidence may only be made after the prosecution rests
its case and not at any time before then.

Respondent Dumlao would not be placed in double jeopardy because, from the very beginning,
the Sandiganbayan had acted without jurisdiction. Precisely, any ruling issued without jurisdiction is, in
[29]
legal contemplation, necessarily null and void and does not exist. Otherwise put, the dismissal of the
case below was invalid for lack of a fundamental prerequisite, that is, due process.

As heretofore explained, the Sandiganbayan gravely abused its discretion amounting to lack of
jurisdiction when it dismissed the case against respondent Dumlao based only on the stipulations made
by the parties during pre-trial. The erroneous equation of the number of members who signed the
minutes of the meeting with the number of members who approved the alleged resolution necessarily led
to the Sandiganbayans faulty conclusion that there was no evidence showing that the GSIS Board of
Trustees approved the alleged Lease-Purchase Agreement. As we have said, the minutes issued by the
Depute Corporate Secretary were enough, at that time, to set the case for trial and to allow the
prosecution to prove its case and to present all its witnesses and evidence.
Case 57-Bugtas
SORIANO vs. PEOPLE
591 SCRA 244

Doctrine:
The fundamental test in considering a motion to quash anchored on Section 3(a), Rule 117 of the 1985

Rules on Criminal Procedure, is the sufficiency of the averments in the information; that is, whether the
facts alleged, if hypothetically admitted, would establish the essential elements of the offense charged as
defined by law. Facts that constitute the defense of the petitioners against the charge under the
information must be proved by them during trial. Such facts or circumstances do not constitute proper
grounds for a motion to quash the information on the ground that the material averments do not
constitute the offense.

Facts:
Soriano and Ilagan were the President and General Manager, respectively, of the Rural Bank of
San Miguel (Bulacan), Inc. (RBSM). Allegedly, on June 27, 1997 and August 21, 1997, during their
incumbency as president and manager of the bank, petitioners indirectly obtained loans from RBSM.
They falsified the loan applications and other bank records, and made it appear that Virgilio J. Malang
and Rogelio Mañaol obtained loans of P15M each, when in fact they did not.
The prosecutor charged Soriano in the RTC with violation of Section 83 of R.A. No. 337 or the
General Banking Act, as amended or Violation of the Director, Officer, Stockholder or Related Interest
(DOSRI) Rules (DOSRI Rules). An information for estafa thru falsification of commercial document
was also filed against Soriano and Ilagan.
Petitioners moved to quash the informations arguing that the prosecutor charged more than one
offense for a single act. Soriano was charged with violation of DOSRI rules and estafa thru falsification
of commercial document for allegedly securing fictitious loans. They further argued that the facts as
alleged in the information do not constitute an offense. RTC denied the motion to quash. CA sustained
the denial of petitioners’ separate motions to quash.
Issue:
Whether or not the contention of the petitioner has merit.
Ruling:
The contention has no merit. In Loney v. People, this Court, in upholding the filing of multiple charges
against the accused, held:
As early as the start of the last century, this Court had ruled that a single act or incident might
offend against two or more entirely distinct and unrelated provisions of law thus justifying the
prosecution of the accused for more than one offense. The only limit to this rule is the Constitutional
prohibition that no person shall be twice put in jeopardy of punishment for the same offense. In People
v. Doriquez, we held that two (or more) offenses arising from the same act are not the same
x x x if one provision [of law] requires proof of an additional fact or element which the other does not, x
x x. Phrased elsewise, where two different laws (or articles of the same code) define two crimes, prior
jeopardy as to one of them is no obstacle to a prosecution of the other, although both offenses arise from
the same facts, if each crime involves some important act which is not an essential element of the other.
In this case, however, Soriano was faced not with one information charging more than one offense, but
with more than one information, each charging a different offense—violation of DOSRI rules in one,
and estafa thru falsification of commercial documents in the others. Ilagan, on the other hand, was
charged with estafa thru falsification of commercial documents in separate informations. Thus,
petitioners erroneously invoke duplicity of charges as a ground to quash the Informations.
The fundamental test in considering a motion to quash anchored on Section 3(a), Rule 117 of the 1985
Rules on Criminal Procedure, is the sufficiency of the averments in the information; that is, whether the
facts alleged, if hypothetically admitted, would establish the essential elements of the offense charged as
defined by law.
Case 58-Cabalonga

CEREZO VS PEOPLE
650 SCRA 222

Doctrine:

No double jeopardy from void judgement. There is no double jeopardy where the judge did not make an
independent assessment of the merits of the motion to withdraw information filed by the prosecutor.

Facts:

Petitioner filed a complaint for libel against respondents. Finding probable cause, the Prosecutor
filed the corresponding Information against them, but reversed its earlier finding and recommended the
withdrawal of the Information.

Relying on the recommendation o f the prosecutor, the RTC ordered the criminal case dismissed
on the ground that it is a settled rule that the determination of the persons to be prosecuted rests
primarily with the Public Prosecutor who is vested with quasi-judicial discretion in the discharge of this
function. Being vested with such power, he can reconsider his own resolution if he finds that there is
reasonable ground to do so.

However, upon petitioner’s motion for reconsideration, the RTC granted the same and reinstated
the case after the DOJ Secretary reversed the resolution of the prosecutor.

Issue:
Whether or not double jeopardy exists in the present case.

Ruling:

NO. One of the requisites in order for double jeopardy to be present is the acquittal, conviction,
dismissal or otherwise termination of the case without the offender’s express consent. The same is not
present in the case.

The order of the RTC based on prosecutor’s resolution as well as the order conforming with the
DOJ’s decision were made with grave abuse of discretion. In rendering such rulings, the RTC blindly
followed the decision of the prosecutor and DOJ without making an independent assessment and
evaluation of the merits of the case. By relying solely on the manifestation of the public prosecutor and
the resolution of the DOJ Secretary, the trial court abdicated its judicial power and refused to perform a
positive duty enjoined by law. Thus, the assailed orders were rendered with grave abuse of discretion
and are now void and without legal effect.

Since the orders are void and do not have a legal effect, the respondents were never acquitted nor was
there a valid and legal dismissal or termination of the case. With the fifth requisite missing, there can be
no double jeopardy.
Case 59- Casiano

WILLIAM CO a.k.a. XU QUING HE vs. NEW PROSPERITY PLASTIC PRODUCTS


G.R. No. 183994. June 30, 2014

Doctrine:
Rule 117 Sec 8- The essential requisites of the first paragraph of Section 8 are conditions sine qua non
to the application of the time bar in the second paragraph. In this case, there is no notice of any motion
for the provisional dismissal or the hearing which was served on private complainant

Facts:
Respondent Company was represented be Elizabeth Uy filed complaint for violation of BP 22
against petitioner William Co. In the absence of Uy and the private counsel the cases were provisionally
dismissed on June 9, 2003 in open court pursuant to Section Rule 117 of the Revised Rules of Criminal
Procedure. Uy received a copy of June 9, 2003 order on July 2, 2003. Through her counsel Uy filed
Motion to Revive the criminal cases. Co posits that the dismissal should be considered final on the
ground that his right to speedy trial was denied. Further, Co insists that assuming the criminal case was
only provisionally dismissed such dismissal became permanent one year after the issuance of June 9,
2003order not after the notice to the offended party. Moreover, Co insists that the motion to revive must
be filed within one year period, that said motion was filed one day late since year 2004 is a leap year.
Issue:
Whether or not the provisional dismissal of the criminal case becomes permanent
Ruling:
No. Co is burdened to establish the essential requisites of the first paragraph of Section 8, Rule 117 of
the Rules, which are conditions sine qua non to the application of the time-bar in the second paragraph
thereof, to wit: (1) the prosecution with the express conformity of the accused or the accused moves for
a provisional (sin perjuicio) dismissal of the case; or both the prosecution and the accused move for a
provisional dismissal of the case; (2) the offended party is notified of the motion for a provisional
dismissal of the case; (3) the court issues an order granting the motion and dismissing the case
provisionally; and (4) the public prosecutor is served with a copy of the order of provisional dismissal of
the case.
In this case, it is apparent from the records that there is no notice of any motion for the provisional
dismissal of Criminal Cases or of the hearing thereon which was served on the private complainant at
least three days before said hearing as mandated by Section 4, Rule 15 of the Rules. 32 The fact is that it
was only in open court that Co moved for provisional dismissal considering that, as per records,
complainant had not shown any interest to pursue her complaint. second paragraph of the new rule states
that the order of dismissal shall become permanent one year after the issuance thereof without the case
having been revived, the provision should be construed to mean that the order of dismissal shall become
permanent one year after service of the order of dismissal on the public prosecutor who has control of
the prosecution without the criminal case having been revived. The public prosecutor cannot be
expected to comply with the timeline unless he is served with a copy of the order of dismissal.
Case 60- Cuevas
PEOPLE vs DE LEON
754 SCRA 147

Doctrine:
Double jeopardy attaches if the following elements are present: (1) a valid complaint or information; (2)
a court of competent jurisdiction; (3) the defendant had pleaded to the charge; and (4) the defendant
was acquitted or convicted or the case against him was dismissed or otherwise terminated without his
express consent.

Facts:
The accused-appellants were charged with Robbery with Homicide under an Information which
reads: That on or about the 2nd day of March, 2002, in Quezon City, Philippines, the above-named
accused, conspiring together, confederating with and mutually helping one another, with intent to gain,
by means of violence and/or intimidation against [sic] person, did then and there willfully, unlawfully
and feloniously rob one EMILIO A. PRASMO, in the following manner, to wit: on the date and place
aforementioned, while victim/deceased Emilio A. Prasmo was walking along A. Bonifacio Street,
Barangay Sta. Lucia, Novaliches, this City, together with his wife and daughter in-law, accused pursuant
to their conspiracy armed with sumpak, samurai, lead pipe and .38 cal. revolver rob EMILIO A.
PRASMO and took and carried away P7,000.00, Philippine currency, and by reason or on the occasion
thereof, with evident premeditation, abuse of superior strength and treachery, accused with intent to
kill[,] attack, assault and employ personal violence upon EMILIOA. PRASMO by then and there
shooting and hacking the victim with the use of said weapons, thereby inflicting upon him serious and
grave wounds which were the direct and immediate cause of his untimely death, to the damage and
prejudice of the heirs of said Emilio A. Prasmo. When arraigned, all the accused-appellants entered a
plea of not guilty except accused Antonio. Thus, the RTC ordered a reverse trial in so far as Antonio is
concerned. The RTC did not find the accused guilty of the crime of robbery with homicide as charged in
the Information but found all the accused guilty of the crime of murder. According to the RTC, contrary
to the charge of robbery with homicide, the accused is guilty of the crime of murder because the
prosecution failed to establish the crime of robbery. The RTC, citing People v. Nimo,23 ratiocinated that
in order to sustain a conviction for robbery with homicide, robbery must be proven as conclusively as
the killing itself. On the other hand, the Court of Appeals affirmed with modifications the ruling of the
RTC and found all of the accused guilty of the crime of murder. However, contrary to the findings of the
RTC with regard to the crime of robbery, the Court of Appeals reversed the ruling of the RTC and found
accused Danilo guilty of the separate crime of robbery.

Issue:
Whether the accused appellant Danilo De Leon was placed in double jeopardy when the
appellate court also found him guilty of robbery based on the same information filed where he was
already found guilty of murder.

Ruling:
Yes, We find that the appellate court erred for violating the constitutional right of Danilo against
double jeopardy as enshrined in Section 21, Article III of the 1987 Constitution, to wit:
Section 21. No person shall be twice put in jeopardy of punishment for the same
offense.1âwphi1 If an act is punished by a law and an ordinance, conviction or acquittal under
either shall constitute a bar to another prosecution for the same act.
Double jeopardy attaches if the following elements are present: (1) a valid complaint or information; (2)
a court of competent jurisdiction; (3) the defendant had pleaded to the charge; and (4) the defendant was
acquitted, or convicted or the case against him was dismissed or otherwise terminated without his
express consent.
In case at bar, it is undisputed the presence of all the elements of double jeopardy: (1) a valid
Information for robbery with homicide was filed; (2) the Information was filed in the court of competent
jurisdiction; (3) the accused pleaded not guilty to the charge; and (4) the RTC acquitted Danilo for the
crime of robbery for lack of sufficient evidence, which amounted to an acquittal from which no appeal
can be had. Indeed the conviction for murder was premised on the fact that robbery was not proven. The
RTC Decision which found accused guilty of the crime of murder and not of robbery with homicide on
the ground of insufficiency of evidence is a judgment of acquittal as to the crime of robbery alone.

As the first jeopardy already attached, the appellate court is precluded from ruling on the innocence or
guilt of Danilo of the crime of robbery. To once again rule on the innocence or guilt of the accused of the
same crime transgresses the Constitutional prohibition not to put any person "twice x x x in jeopardy of
punishment for the same offense." As it stands, the acquittal on the crime of robbery based on lack of
sufficient evidence is immediately final and cannot be appealed on the ground of double jeopardy. A
judgment of acquittal is final and unappealable. In fact, the Court cannot, even an appeal based on an
alleged misappreciation of evidence, review the verdict of acquittal of the trial court due to the
constitutional proscription, the purpose of which is to afford the defendant, who has been acquitted, final
repose and safeguard from government oppression through the abuse of criminal processes. The crime
of robbery was not proven during the trial. As we discussed, the acquittal of the accused-appellant,
including Danilo, is not reversible.
Case 61- Custodio

SALVANERA vs. PEOPLE OF THE PHILIPPINES


G.R. No. 143093 May 21, 2007

Doctrine:

In the discharge of an accused in order that he may be a state witness, the following conditions must be
present, namely: 1. Two or more accused are jointly charged with the commission of an offense; 2. The
motion for discharge is filed by the prosecution before it rests its case; 3. The prosecution is required to
present evidence and the sworn statement of each proposed state witness at a hearing in support of the
discharge; 4. The accused gives his consent to be a state witness; and 5. The trial court is satisfied that:
a) There is absolute necessity for the testimony of the accused whose discharge is requested; b) There is
no other direct evidence available for the proper prosecution of the offense committed, except the
testimony of said accused; c) The testimony of said accused can be substantially corroborated in its
material points; d) Said accused does not appear to be the most guilty; and, e) Said accused has not at
any time been convicted of any offense involving moral turpitude.

Facts:

Petitioner Rimberto Salvanera, together with Feliciano Abutin, Edgardo Lungcay and Domingo
Tampelix, is charged with the murder of Ruben Parane.
Salvanera was the alleged mastermind; Lungcay, the hired hitman; Abutin, the driver of the motorcycle
which carried Lungcay to the place of the commission of the crime; while Tampelix delivered the blood
money to the latter.

On March 4, 1997, the prosecution moved for the discharge of accused Feliciano Abutin and
Domingo Tampelix, to serve as state witnesses.
After several motions by the prosecution, the Court of Appeals discharged accused Feliciano Abutin and
Domingo Tampelix from the Information to become state witnesses, and cancelled the bail bond of
petitioner Salvanera.

Issue:

Whether or not there is sufficient grounds to discharge accused Feliciano Abutin and Domingo
Tampelix as state witnesses against Rimberto Salvanera

Ruling:

In the discharge of an accused in order that he may be a state witness, the following conditions
must be present, namely:

(1) Two or more accused are jointly charged with the commission of an offense;
(2) The motion for discharge is filed by the prosecution before it rests its case;
(3) The prosecution is required to present evidence and the sworn statement of each proposed state
witness at a hearing in support of the discharge;
(4) The accused gives his consent to be a state witness; and
(5) The trial court is satisfied that:
a) There is absolute necessity for the testimony of the accused whose discharge is requested;
b) There is no other direct evidence available for the proper prosecution of the offense committed,
except the testimony of said accused;
c) The testimony of said accused can be substantially corroborated in its material points;
d) Said accused does not appear to be the most guilty; and,
e) Said accused has not at any time been convicted of any offense involving moral turpitude.

In the case at bar, we are satisfied from a reading of the records that the testimonies of Abutin
and Tampelix are corroborated on important points by each other’s testimonies and the circumstances
disclosed through the testimonies of the other prosecution witnesses, and "to such extent that their
trustworthiness becomes manifest."

As part of the conspiracy, Abutin and Tampelix can testify on the criminal plan of the
conspirators. Where a crime is contrived in secret, the discharge of one of the conspirators is essential
conspirators. Where a crime is contrived in secret, the discharge of one of the conspirators is essential
because only they have knowledge of the crime. The other prosecution witnesses are not eyewitnesses to
the crime, as, in fact, there is none. No one except the conspirators knew and witnessed the murder. The
testimonies of the accused and proposed state witnesses Abutin and Tampelix can directly link petitioner
to the commission of the crime.

Case 62- De Luna

VDA. DE MANGUERRA vs RAUL RISOS


563 SCRA 471

Doctrine: Deposition As A Mode Of Discovery


Facts:
Respondents were charged with Estafa Through Falsification of Public Document before the
RTC of Cebu City. Concepcion, who was a resident of Cebu City, while on vacation in Manila, was
unexpectedly confined at the Makati Medical Center due to upper gastro-intestinal bleeding; and was
advised to stay in Manila for further treatment. RTC granted the motion and directed that Concepcion’s
deposition be taken before the Clerk of Court of Makati City. The respondent’s motion for
reconsideration was. The court ratiocinated that procedural technicalities should be brushed aside
because of the urgency of the situation, since Concepcion was already of advanced age. After several
motions for change of venue of the deposition-taking, Concepcion’s deposition was finally taken at her
residence.
On appeal, CA declared the conducted deposition void. CA added that the rationale of
the Rules in requiring the taking of deposition before the same court is the constitutional right of the
accused to meet the witnesses face to face. The appellate court likewise concluded that Rule 23 could
not be applied suppletorily because the situation was adequately addressed by a specific provision of the
rules of criminal procedure.
Petitioners contend that Concepcions advanced age and health condition exempt her from the
application of Section 15, Rule 119 of the Rules of Criminal Procedure, and thus, calls for the
application of Rule 23 of the Rules of Civil Procedure.

Issue:
Whether or not rule 23 of the 1997 rules of civil procedure applies to the deposition in a criminal
case.
Ruling:
It is basic that all witnesses shall give their testimonies at the trial of the case in the presence of
the judge. This is especially true in criminal cases in order that the accused may be afforded the
opportunity to cross-examine the witnesses pursuant to his constitutional right to confront the witnesses
face to face. It also gives the parties and their counsel the chance to propound such questions as they
deem material and necessary to support their position or to test the credibility of said witnesses. Lastly,
this rule enables the judge to observe the witnesses demeanor.

Rule 119 categorically states that the conditional examination of a prosecution witness shall be
made before the court where the case is pending. Contrary to petitioner’s contention, there is nothing in
the rule which may remotely be interpreted to mean that such requirement applies only to cases where
the witness is within the jurisdiction of said court and not when he is kilometers away, as in the present
case. The criminal proceedings are primarily governed by the Revised Rules of Criminal
Procedure. Considering that Rule 119 adequately and squarely covers the situation in the instant case,
we find no cogent reason to apply Rule 23 suppletorily or otherwise.

To reiterate, the conditional examination of a prosecution witness for the purpose of taking his
deposition should be made before the court, or at least before the judge, where the case is pending. Such
is the clear mandate of Section 15, Rule 119 of the Rules.

Case 63- Dionisio


MANUEL J. JIMENEZ, JR. vs. PEOPLE OF THE PHILIPPINES
G.R. Nos. 209195 and 209215 September 17, 2014

Doctrine:

In a motion to discharge an accused to become a state witness, it is still the trial court that determines
whether the prosecution’s preliminary assessment of the accused -witness’ qualifications satisfies the
procedural norms. This is a symbiotic relationship as the trial court largely exercises its prerogative
based on the prosecutor’s findings and evaluation.

Facts:
Manuel A. Montero confessed his participation in the killing of Ruby Rose Barrameda naming
Manuel J. Jimenez and several others as co-conspirators. His statements detailed where the alleged steel
casing containing the body of Ruby Rose was dumped, led to the recovery of a cadaver near the place
which he pointed. Montero filed a motion for discharge as a state witness for the prosecution, to which
Jimenez opposed. The motion to discharge was granted by Judge Zaldy B. Docena stating that the
prosecution had presented clear, satisfactory and convincing evidence showing compliance with the
requisites of granting the said motion. Jimenez opposed Judge Docena’s ruling averring that the Judge
committed grave abuse of discretion in granting the motion to discharge because: (1)the requirements
for granting a motion were not properly complied; (2)there is no absolute necessity of the testimony of
Montero; (3)Montero’s testimony do not corroborate with the prosecution’s evidence; (4) and Montero is
favored as a state witness though he appears to be the most guilty.

Issue:
Whether or not Judge Docena gravely abuse his discretion when he granted the motion to
discharge Montero as a state witness?

Ruling:
No. Jurisprudence has defined “grave abuse of discretion” as the capricious and whimsical
exercise of judgment as where the power is exercised in an arbitrary and despotic manner. To resolve a
motion to discharge under Section 17, Rule 119of the Revised Rules of Criminal Procedure, it only
require that that the testimony of the accused sought to be discharged be substantially corroborated in its
material points, not on all points. A trial judge cannot be expected or required, at the start of the trial, to
inform himself with absolute certainty of everything that may develop in the course of the trial with
respect to the guilty participation of the accused. It is still the trial court that determines whether the
prosecution’s preliminary assessment of the accused- witness’ qualifications to be a state witness
satisfies the procedural norms. This relationship is in reality a symbiotic one as the trial court, by the
very nature of its role in the administration of justice, largely exercises its prerogative based on the
prosecutor’s findings and evaluation.
Case 64- Dolor

PEOPLE vs DE GRANO
588 SCRA 550

Doctrine:
Stages of the Trial Where the Presence of the Accused is Required Section 14(2), Article III of the
Constitution, authorizing trials in absentia, allows the accused to be absent at the trial but not at certain
stages of the proceedings, to wit:

(a) at arraignment and plea, whether of innocence or of guilt;


(b) during trial, whenever necessary for identification purposes; and
(c) at the promulgation of sentence, unless it is for a light offense, in which case, the accused may
appear by counsel or representative. At such stages of the proceedings, his presence is required and
cannot be waived.

Facts:
An Information for murder committed against Emmanuel Mendoza was filed with the Regional
Trial Court (RTC), Branch 6, Tanauan, Batangas, against Joven de Grano (Joven), Armando de Grano
(Armando), and Estanislao Lacaba (Estanislao), together with their co-accused Leonides Landicho
(Leonides), Domingo Landicho (Domingo), and Leonardo Genil (Leonardo).
Duly arraigned, Joven, Armando, and Estanislao pleaded "not guilty" to the crime as charged;
while their co-accused Leonides, Leonardo, and Domingo remained at-large. Thereafter, respondents
filed a motion for bail contending that the prosecution’s evidence was not strong.
The prosecution then filed a petition for certiorari with the CA which was denied. They sought
recourse before this Court. the decision of the CA together with the Order of the RTC granting bail to the
respondent was set aside. The RTC was also ordered to immediately issue a warrant of arrest against the
accuse. As a result, Estanislao was re-arrested, but Joven and Armando were not.
However, upon respondents’ motion for reconsideration, this Court resolved to remand the case
to the RTC. The RTC found several accused guilty of the offense as charged. But the case as against
accused Leonides Landicho and Leonardo Genil was sent to the files or archived cases to be revived as
soon as said accused are apprehended. Only Estanislao was present at the promulgation despite due
notice to the other respondents.
In its Opposition, the prosecution pointed out that while the accused jointly moved for the
reconsideration of the decision, all of them, except Estanislao, were at-large. Having opted to become
fugitives and be beyond the judicial ambit, they lost their right to file such motion for reconsideration
and to ask for whatever relief from the court.
Respondents filed a Joint Motion for Reconsideration praying that the Decision be reconsidered
and set aside and a new one be entered acquitting them. The RTC issued an Order modifying its earlier
decision by acquitting Joven and Armando, and downgrading the conviction of Domingo and Estanislao
from murder to homicide.
Estanislao filed a Notice of Appeal. The RTC issued an Order denying the motion and giving due
course to Estanislao’s notice of appeal.
Petitioner, filed a Petition for certiorari before the CA arguing that the private respondents,
having deliberately evaded arrest after being denied bail and deliberately failing to attend the
promulgation of the Decision despite due notice, lost the right to move for reconsideration of their
conviction and that the grounds relied upon by respondent RTC in modifying its Decision are utterly
erroneous.
Respondent De Grano filed a Motion to Dismiss, arguing that the verification and certification
portion of the petition was flawed, since it was signed only by counsel and not by the aggrieved party.
Also, the petition did not contain the conformity of the Solicitor General.
Petitioner filed an Opposition to Motion to Dismiss and explained that, for lack of material time,
it failed to secure the conformity of the OSG when it filed the petition, but it would nevertheless obtain
it. Anent the verification and certification of the petition having been signed by the private prosecutor,
petitioner explained that private complainant Teresita was in fear for her life as a result of the acquittal
of former Mayor Joven de Grano, but she was willing to certify the petition should she be given ample
time to travel to Manila.
However, the petition was dismissed outright by the CA on the grounds that it was not filed by
the OSG and that the assailed Orders were only photocopies and not certified true copies.
Petitioner timely filed a Motion for Reconsideration. Meanwhile, in its 1st Endorsement, DOJ
Secretary Raul M. Gonzalez, endorsed the petition filed by the Assistant City Prosecutor, with the
assistance of the private prosecutor, to the Solicitor General for his conformity.
On April 5, 2005, the CA issued a Resolution denying the motion.

Issue:
Whether or not the accused who failed to appear without justifiable cause shall lose the remedies
available in the Rules against the judgment.

Ruling:
Section 6, Rule 120 of the Revised Rules of Criminal Procedure, the Rules applicable at the time
the Decision was promulgated, provides:
Section 6. Promulgation of judgment. –The judgment is promulgated by reading it in the
presence of the accused and any judge of the court in which it was rendered. However, if the
conviction is for a light offense the judgment may be pronounced in the presence of his counsel
or representative. When the judge is absent or outside the province or city, the judgment may be
promulgated by the clerk of court.
If the accused is confined or detained in another province or city, the judgment may be
promulgated by the executive judge of the Regional Trial Court having jurisdiction over the place of
confinement or detention upon request of the court which rendered the judgment. The court
promulgating the judgment shall have authority to accept the notice of appeal and to approve the bail
bond pending appeal; provided, that if the decision of the trial court convicting the accused changed the
nature of the offense from non-bailable to bailable, the application for bail can only be filed and resolved
by the appellate court.
The proper clerk of court shall give notice to the accused, personally or through his bondsman or
warden and counsel, requiring him to be present at the promulgation of the decision. If the accused was
tried in absentia because he jumped bail or escaped from prison, the notice to him shall be served at his
last known address.
In case the accused fails to appear at the scheduled date of promulgation of judgment despite
notice, the promulgation shall be made by recording the judgment in the criminal docket and serving
him a copy thereof at his last known address or thru his counsel.
If the judgment is for conviction and the failure of the accused to appear was without justifiable
cause, he shall lose the remedies available in these Rules against the judgment and the court shall order
his arrest. Within fifteen (15) days from promulgation of judgment however, the accused may surrender
and file a motion for leave of court to avail of these remedies. He shall state the reasons for his absence
at the scheduled promulgation and if he proves that his absence was for a justifiable cause, he shall be
allowed to avail of said remedies within fifteen (15) days from notice.
Thus, the accused who failed to appear without justifiable cause shall lose the remedies available
in the Rules against the judgment. However, within 15 days from promulgation of judgment, the accused
may surrender and file a motion for leave of court to avail of these remedies. He shall state in his motion
the reasons for his absence at the scheduled promulgation, and if he proves that his absence was for a
justifiable cause, he shall be allowed to avail of said remedies within 15 days from notice.

Case 65- Evangelista


JOCELYN ASISTIO vs. PEOPLE OF THE PHILIPPINES
G.R. No. 200465 April 20, 2015

Doctrine:

The proper remedy of a party aggrieved by a decision of the Court of Appeals is a petition for review
under Rule 45, which is not identical to a petition for certiorari under Rule 65. Under Rule 45,
decisions, final orders or resolutions of the Court of Appeals in any case, i.e., regardless of the nature of
the action or proceedings involved, may be appealed to us by filing a petition for review, which would be
but a continuation of the appellate process over the original case. On the other hand, a special civil
action under Rule 65 is an independent action based on the specific ground therein provided and, as a
general rule, cannot be availed of as a substitute for the lost remedy of an ordinary appeal, including
that to be taken under Rule 45.

Facts:
​Petitioner Jocelyn Asistio was charged with violation of Section 46 of the Cooperative Code of
the Philippines (Republic Act No. [RA] 6938).
The prosecution is to prove that petitioner, then Chairperson of the A. Mabini Elementary School
Teachers Multi-Purpose Cooperative, had entered into an exclusive dealership agreement with Coca-
Cola Bottlers Philippines, Inc., (Coca-Cola) for the sale of softdrink products at the same school. The
school directed the petitioner to submit her financial reports. Instead, petitioner claimed that the
principal had no business and authority to require her to produce financial statements.
​The school principal then created an audit committee to look into the financial reports of the
Cooperative. Based on the documents obtained from Coca-Cola, the audit committee found that
petitioner defrauded the Cooperative and its members for three years. Despite requests for her to return
the amounts she allegedly misappropriated, petitioner failed and refused to do so. Thus, the Cooperative
filed criminal charges against petitioner.
​After the presentation and offer of evidence by the prosecution, petitioner moved to dismiss the
case by way of Demurrer to Evidence with prior leave of court. She argued that the Regional Trial Court
(RTC) of Manila, does not have jurisdiction over the case, as the crime charged does not carry with it a
sanction for which she can be held criminally liable.
​RTC dismissed the case for lack of jurisdiction. RTC also denied for lack of merit the motion for
reconsideration of the order of dismissal.
​The dispute involving the parties is certainly a dispute and issue between and among directors,
officers or members of the A. Mabini Elementary School Teachers Multi-Purpose Cooperative which is
governed by the Guidelines.
​People of the Philippines, represented by the Office of the Solicitor General (OSG), appealed the
order of dismissal to the CA. The CA rendered a Decision reversing and setting aside the RTC Orders
and remanded the case records to the RTC for further proceedings. The CA denied petitioner's motion
for reconsideration of its decision.
​Aggrieved, petitioner filed this petition for certiorari under Rule 65 of the Rules of Court.
Issue:
​Whether or not the proper remedy is petition for review under Rule 45 or petition for certiorari
under Rule 65

Ruling:
​As a rule, the remedy from a judgment or final order of the CA is appeal via petition for review
under Rule 45 of the Rules of Court. In jurisprudence, the Court had again stressed the distinction
between the remedies provided for under Rule 45 and Rule 65, to wit:
The proper remedy of a party aggrieved by a decision of the Court of Appeals is a petition for review
under Rule 45, which is not identical to a petition for certiorari under Rule 65. Under Rule 45,
decisions, final orders or resolutions of the Court of Appeals in any case, regardless of the nature of the
action or proceedings involved, may be appealed to us by filing a petition for review, which would be
but a continuation of the appellate process over the original case. On the other hand, a special civil
action under Rule 65 is an independent action based on the specific ground therein provided and, as a
general rule, cannot be availed of as a substitute for the lost remedy of an ordinary appeal, including that
to be taken under Rule 45.
The Court explained that one of the requisites of certiorari is that there be no available appeal or any
plain, speedy and adequate remedy. Where an appeal is available, certiorari will not prosper, even if the
ground therefore is grave abuse of discretion. It is also well settled that a party cannot file a petition both
under Rules 45 and 65 of the Rules of Court because said procedural rules pertain to different remedies
and have distinct applications. The remedy of appeal under Rule 45 and the original action
for certiorari under Rule 65 are mutually exclusive and not alternative or cumulative. Thus, when
petitioner adopts an improper remedy, petition may be dismissed outright.

Case 66- Gabi

ANTONIO CABADOR vs. PEOPLE OF THE PHILIPPINES


G.R. No. 186001 October 2, 2009

Doctrine:
Demurrer to Evidence; Because some accused have in the past used the demurrer in order to delay the
proceedings in the case, the remedy now carries a caveat – when the accused files a demurrer without
leave of court, he shall be deemed to have waived the right to present evidence and the case shall be
considered submitted for judgment.
Motion to Dismiss; “Demurrer to Evidence” and “Motion to Dismiss,” Distinguished; Pleadings and
Practice; to determine whether the pleading filed is a demurrer to evidence or a motion to dismiss, the
Court must consider (1) the allegations in it made in good faith; (2) the stage of the proceeding at which
it is filed; and (3) the primary objective of the party filing it.
A demurrer to evidence assumes that the prosecution has already rested its case; where the accused filed
his motion to dismiss before he could object to the prosecution’s formal offer before the trial court could
act on the offer, and before the prosecution could rest its case, it could not be said that he had intended
his motion to dismiss to serve as a demurrer to evidence.

Facts:

A petition for review on certiorari, assailing the CA Decision and Resolution that affirmed the
Order of the RTC of Quezon City.

The public prosecutor accused Cabador before the RTC of Quezon City of murdering, in
conspiracy with others, Atty. Jun N. Valerio. After presenting only five witnesses over five years of
intermittent trial, the RTC declared at the end the prosecution’s presentation of evidence and required the
prosecution to make a written or formal offer of its documentary evidence within 15 days from notice.
But the public prosecutor asked for three extensions of time, the last of which was to end on July 28,
2006. Still, the prosecution did not make the required written offer.
Cabador filed a motion to dismiss the case complaining of a turtle-paced proceeding in the case
since his arrest and detention in 2001 and invoking his right to a speedy trial. Further, he claimed that in
the circumstances, the trial court could not consider any evidence against him that had not been formally
offered. He also pointed out that the prosecution witnesses did not have knowledge of his alleged part in
the crime charged.
The RTC issued an Order treating petitioner Cabador’s motion to dismiss as a demurrer to
evidence. And, since he filed his motion without leave of court, the RTC declared him to have waived
his right to present evidence in his defense. The trial court deemed the case submitted for decision
insofar as he was concerned. Cabador filed a motion for reconsideration of this Order but the RTC
denied it. Cabador questioned the RTC’s actions before the CA but the latter denied his petition and
affirmed the lower court’s actions. With the CA’s denial of his motion for reconsideration, the petitioner
came to this Court via a petition for review on certiorari.

Issue:

Whether Cabador’s motion to dismiss before the trial court was in fact a demurrer to evidence
filed without leave of court.

Ruling:

The trial proper in a criminal case usually has two stages: first, the prosecution’s presentation of
evidence against the accused and, second, the accused’s presentation of evidence in his defense. If, after
the prosecution has presented its evidence, the same appears insufficient to support a conviction, the trial
court may at its own initiative or on motion of the accused dispense with the second stage and dismiss
the criminal action. There is no point for the trial court to hear the evidence of the accused in such a case
since the prosecution bears the burden of proving his guilt beyond reasonable doubt. The order of
dismissal amounts to an acquittal.
But because some have in the past used the demurrer in order to delay the proceedings in the
case, the remedy now carries a caveat. When the accused files a demurrer without leave of court, he shall
be deemed to have waived the right to present evidence and the case shall be considered submitted for
judgment. On occasions, this presents a problem such as when, like the situation in this case, the accused
files a motion to dismiss that, to the RTC, had the appearance of a demurrer to evidence. Cabador insists
that it is not one but the CA, like the lower court, ruled that it is.
This Court held in Enojas, Jr. v. Commission on Elections that, to determine whether the
pleading filed is a demurer to evidence or a motion to dismiss, the Court must consider (1) the
allegations in it made in good faith; (2) the stage of the proceeding at which it is filed; and (3) the
primary objective of the party filing it.
In criminal cases, a motion to dismiss may be filed on the ground of denial of the accused’s right
to speedy trial. This denial is characterized by unreasonable, vexatious, and oppressive delays without
fault of the accused, or by unjustified postponements that unreasonably prolonged the trial. This was the
main thrust of Cabador’s motion to dismiss and he had the right to bring this up for a ruling by the trial
court.

The fact is that Cabador did not even bother to do what is so fundamental in any demurrer. He
did not state what evidence the prosecution had presented against him to show in what respects such
evidence failed to meet the elements of the crime charged. His so-called "demurrer" did not touch on any
particular testimony of even one witness. He cited no documentary exhibit. Indeed, he could not
because, he did not know that the prosecution finally made its formal offer of exhibits on the same date
he filed his motion to dismiss.To say that Cabador filed a demurrer to evidence is equivalent to the
proverbial blind man, touching the side of an elephant, and exclaiming that he had touched a wall.
Case 67- Huggins
PEOPLE vs. TAN
GR No. 117321 February 11, 1998

Doctrine:
Constitution abhors an uncounselled confession or admission and whatever information is derived from
such shall be regarded as inadmissible in evidence against the confessant. The waiver of the right to
counsel must be voluntary, knowing, intelligent, and must be made in the presence and assistance of a
counsel.

Facts:
Herson Tan and Lito Amido were charged with the crime of highway robbery with murder. It
was alleged that the two accused conspired in stealing a Honda TMX motorcycle belonging to Freddie
Saavedra, and killed the latter.
​Subsequent to the recovery of the abandoned sidecar which was the subject of robbery, Lt.
Carlos and other police officers invited Tan for questioning. During their conversation, Tan narrated that
he and Amido were responsible for the loss of the motorcycle and the consequent death of Saavedra.
Moreover, he averred that they sold the motorcycle to a certain Danny Teves of Barrio Summit,

Muntinlupa for a sum of P4,000.00.


On arraignment, Tan pleaded not guilty. During the trial, Lt. Carlos testified that when he invited
Tan to their headquarters, he had no warrant of arrest, and in the course of questioning Tan, he admitted
that he did not inform the accused of his constitutional rights to remain silent and the assistance of a
counsel, nor did he reduce the supposed confession into writing.
The trial court convicted Tan for the crime of highway robbery with murder, while Amido was
acquitted due to insufficiency of evidence. Hence, the petition.
Issue:
Whether or not the confession of the accused obtained during custodial investigation is
inadmissible on the ground that it was taken without the presence of a counsel
Ruling:
Yes, the confession of Tan during the custodial investigation is inadmissible as evidence. It is
well-settled that the Constitution abhors an uncounselled confession or admission and whatever
information is derived therefrom shall be regarded as inadmissible in evidence against the confessant as
provided under Article III, Section 12, paragraphs (1) and (3) of the Constitution.
Under RA 7438, custodial investigation shall include the practice of issuing an invitation to a person
who is investigated in connection with an offense he is suspected to have committed, without prejudice
to the liability of the inviting officer for any violation of law. The rules on custodial investigation begin
to operate as soon as the investigation ceases to be a general inquiry into an unsolved crime and begins
to focus a particular suspect, the suspect is taken into custody, and the police carries out a process of
interrogations that tends itself to eliciting incriminating statements.
Under the Constitution and existing law and jurisprudence, a confession to be admissible must
satisfy the following requirements: (1) it must be voluntary; (2) it must be made with the assistance of
competent and independent counsel; (3) it must be express; and (4) it must be in writing.
In the case at hand, it is clear that Tan was not informed of his constitutional rights nor was he
provided with a counsel. None of the requirements for a valid confession was present. Therefore, his
confession is inadmissible. Tan was acquitted in view of the inadequacy of the remaining evidence to
warrant conviction.
Case 68- Marcelo

NELSON IMPERIAL, ET AL., vs. MARICEL M. JOSON


635 SCRA 71

Doctrine: Right to Speedy Trial

Facts:
The case stemmed from the collision of Isuzu ten-wheeler truck collided with a Fuso six-wheeler
truck. Owned by petitioner Nelson Imperial and the Fuso six-wheeler truck was driven by respondent
Santiago Giganto, Jr. in Barangay Concepcion, Sariaya, Quezon. The collision damaged multiple
vehicles with multiple fatalities and serious physical injuries to the victims.
As a consequence of the collisions, a criminal complaint for Reckless Imprudence Resulting to
Multiple Homicide, Multiple Serious Physical Injuries and Damage to Property was filed against Santos
Francisco and Noel Imperial before the Municipal Trial Court (MTC) of Sariaya, Quezon.
Thereafter, a series of complaint for damages was also filed in the, Regional Trial Court (RTC)
of the following cities Naga City, Valenzuela City, and Parañaque City. In the end, the Court of Appeals
ruled that, the case in Naga City and Valenzuela City was dismissed and the jurisdiction of Parañaque
City was upheld on the ground that it was the venue most accessible to majority of the parties.
Meanwhile, the criminal case in Sariaya MTC proceeded, and the petitioner entered a plea of not
guilty at the arraignment scheduled in the case. Thru his counsel, petitioner Francisco proposed the
some facts for stipulation with the prosecution, however the prosecution refused to stipulate on the
foregoing matter on the ground that the prosecution will stipulate and admit to that effect only after the
arraignment. Petitioner filed a motion to compel and disqualify the prosecutor and to correct the pre-trial
order, however, the motion was denied. Instead, the court issued an order that the pre-trial conference be
set anew in view of the reassignment of the case to Prosecutor Francis Sia and the appearance of a new
private prosecutor in the case.
Likewise contending that the nine postponements of the pre-trial conference in Criminal Case
No. 01-99 were capricious, vexatious and oppressive, petitioner Francisco further moved for the
dismissal of the case, on the ground that his constitutional right to a speedy trial had been violated. The
trial court dismissed the said motion.

Issue:

Whether or not, petitioner’s right of speedy trial was violated?

Ruling:
No, petitioner’s right of speedy trial was not violated. According to the Supreme Court, although
the Constitution concededly guarantees that "(a)ll persons shall have the right to a speedy disposition of
their cases before all judicial, quasi-judicial, or administrative bodies", said right is considered violated
only when the proceeding is attended by vexatious, capricious and oppressive delays

In the case, considering that majority of the parties live closer to the Parañaque RTC, and the
witnesses live closer to the Parañaque RTC rather than the Naga RTC. Hence, the former is more
accessible.
Further, in determining whether the accused has been deprived of his right to a speedy
disposition of the case and to a speedy trial, four factors must be considered: (a) length of delay; (b) the
reason for the delay; (c) the defendant's assertion of his right; and (d) prejudice to the defendant." . . .
Petitioner Francisco claims that his right to a speedy trial was violated when the Public
Prosecutors assigned to the case failed to attend the nine hearings, here, the delays entailed by the
postponements of the aforesaid hearings were, to a great extent, attributable to petitioner Francisco's
own pursuit of extraordinary remedies against the interlocutory orders issued by the Sariaya MTC.
Francisco cannot, consequently, complain of violation of his right to speedy trial in view of his pending
petition for certiorari, prohibition and mandamus, which raised, among other matters, issues pertinent to
the conduct of the pre-trial conference by the Sariaya MTC.
Case 69- Martinez

PEOPLE VS SANDIGANBAYAN
645 SCRA 726

Doctrines:
In criminal cases, the grant of a demurrer is tantamount to an acquittal and the dismissal order may not
be appealed because this would place the accused in double jeopardy. Although the dismissal order is
not subject to appeal, it is still reviewable but only through certiorari under Rule 65 of the Rules of
Court. For the writ to issue, the trial court must be shown to have acted with grave abuse of discretion
amounting to lack or excess of jurisdiction such as where the prosecution was denied the opportunity to
present its case or where the trial was a sham thus rendering the assailed judgment void. The burden is
on the petitioner to clearly demonstrate that the trial court blatantly abused its authority to a point so
grave as to deprive it of its very power to dispense justice.
Actual damage to the government arising from the non-liquidation of the cash advance is not an
essential element of the offense punished under the second sentence of Section 89 of Presidential Decree
No. 1445 as implemented by Commission on Audit (COA) Circular No. 90-331; The mere failure to
timely liquidate the cash advance is the gravamen of the offense.
As can be seen, contrary to the findings of the Sandiganbayan, actual damage to the government arising
from the non-liquidation of the cash advance is not an essential element of the offense punished under
the second sentence of Section 89 of P.D. No. 1445 as implemented by COA Circular No. 90-331.
Instead, the mere failure to timely liquidate the cash advance is the gravamen of the offense. Verily, the
law seeks to compel the accountable officer, by penal provision, to promptly render an account of the
funds which he has received by reason of his office.
Facts:
Around December 19, 1995, thereafter, accused MANUEL G. BARCENAS, vice mayor of
Toledo City, had been charged with violation of the Section 89 of PD 1445. The information contained
information that he committed an offense in relation to office and he obtained over 61k which he was
bound to liquidate, which he did not. He was charged before the Sandiganbayan, where upon
presentation of evidence of prosecution had rested, he filed a demurrer to evidence. The Sandiganbayan
granted said demurrer.
Petitioner contends that the prosecution was able to establish all the elements of the offense
defined and penalized under Section 89 of P.D. No. 1445: (1) the private respondent, an accountable
officer, received cash advances in the total amount of P120,000.00 to defray the expenses of the Public
Assistance Committee and Committee on Police Matters covering the period January-March 1993, (2)
the purpose of the cash advance has been served, (3) the private respondent settled his cash advances
only in March 1996, (4) the city auditor sent a demand letter to the private respondent to settle the cash
advance within 72 hours from receipt thereof, and (5) the private respondent received said letter on
December 22, 1995 but failed to liquidate the same within the period. Although it concedes that the
private respondent eventually settled the subject cash advances sometime in March 1996, petitioner
theorizes that damage is not one of the elements of the offense charged. Hence, the settlement of the
cash advance would not exonerate the private respondent but only mitigate his criminal liability.
Otherwise, the purpose of the law would be rendered futile since accountable officers can easily make
cash advances and liquidate the same beyond the period prescribed by law without being penalized for
doing so. Finally, petitioner argues that double jeopardy does not lie in this case because the order of
dismissal was issued with grave abuse of discretion amounting to lack of jurisdiction.
Private respondent counters that the grant of a demurrer to evidence is equivalent to an acquittal
from which the prosecution cannot appeal as it would place the accused in double jeopardy. Further,
assuming that the Sandiganbayan erroneously granted the demurrer, this would, at most, constitute an
error of judgment and not an error of jurisdiction. Thus, certiorari does not lie to correct the grant of the
demurrer to evidence by the Sandiganbayan.
Issue:
Whether the Sandiganbayan acted with grave abuse of discretion amounting to lack or excess of
jurisdiction in giving due course to and eventually granting the demurrer to evidence.

Ruling:
YES. In criminal cases, the grant of a demurrer is tantamount to an acquittal and this would place
the accused in double jeopardy. This may be reviewable through certiorari and the writ may only be
issued if it could be proved that the trial court must be shown to have acted with grave abuse of
discretion. In the case at bar, the Sandiganbayan granted the demurrer to evidence on the ground that the
prosecution failed to prove that the government suffered any damage from Barcenas’ non-liquidation of
the subject cash advances because it was later shown, as admitted by the prosecution’s witness that
Barcenas liquidated the same albeit belatedly.

Case 70- Masungsong

SALLY GO-BANGAYAN vs. BENJAMIN BANGAYAN, JR


G.R. No. 201061 July 3, 2013

Doctrine:
Waiver of right to present evidence; Petition for Review (Appeal). It is well-settled that a
grant of a motion for continuance or postponement is not a matter of right but is addressed to the
discretion of the trial court.

Facts:
Benjamin Bangayan, Jr. married Azucena Alegre in September 1973. In February 1982,
Benjamin and Sally lived together as husband and wife. On 7 March 1982, in order to appease her
father, Sally brought Benjamin to an office in Santolan, Pasig City where they signed a purported
marriage contract.
The relationship of Benjamin and Sally ended in 1994. She then filed criminal actions for
bigamy and falsification of public documents against him. Benjamin, in turn, filed a petition for
declaration of a non-existent marriage and/or declaration of nullity of marriage before the trial court on
the ground that his marriage to Sally was bigamous and that it lacked the formal requisites to a valid
marriage.
After Benjamin presented his evidence, Sally filed a demurrer to evidence and subsequently, a
motion for reconsideration which the trial court denied. The trial court gave Sally several opportunities
to present her evidence on 28 February 2008, 10 July 2008, 4 September 2008, 11 September 2008, 2
October 2008, 23 October 2008, and 28 November 2008. Despite repeated warnings from the trial court,
Sally still refused to present her evidence, prompting the trial court to consider the case submitted for
decision.
ACTIONS of the COURT
RTC and CA: Ruled that Sally Bangayan waived her right to present evidence.
SC: Decision of the CA is affirmed.

Issue:
Whether or Not Sally had waived her right to present evidence.

Ruling:

It is well-settled that a grant of a motion for continuance or postponement is not a matter of right
but is addressed to the discretion of the trial court. In this case, Sally’s presentation of evidence was
scheduled on28 February 2008. Thereafter, there were six resettings of the case.
Sally could not complain that she had been deprived of her right to present her evidence because
all the postponements were at her instance and she was warned by the trial court that it would submit the
case for decision should she still fail to present her evidence on 28 November 2008.
We agree with the trial court that by her continued refusal to present her evidence, she was
deemed to have waived her right to present them. Sally could not accuse the trial court of failing to
protect marriage as an inviolable institution because the trial court also has the duty to ensure that trial
proceeds despite the deliberate delay and refusal to proceed by one of the parties.
Case 71- Morada

PEOPLE OF THE PHILIPPINES vs. JOSE C. GO


G.R. No. 191015 August 6, 2014

Doctrine:
Demurrer to evidence is “an objection by one of the parties in an action, to the effect that the evidence
which is his adversary produced is insufficient in point of law, whether true or not, to make out a case or
sustain the issue. – Demurrer to the evidence is "an objection by one of the parties in an action, to the
effect that the evidence which his adversary produced is insufficient in point of law, whether true or not,
to make out a case or sustain the issue. The party demurring challenges the sufficiency of the whole
evidence to sustain a verdict. The court, in passing upon the sufficiency of the evidence raised in a

demurrer, is merely required to ascertain whether there is competent or sufficient evidence to sustain the
indictment or to support a verdict of guilt. x x x Sufficient evidence for purposes of frustrating a
demurrer thereto is such evidence in character, weight or amount as will legally justify the judicial or
official action demanded according to the circumstances.
To be considered sufficient therefore, the evidence must prove:
(a) the commission of the crime, and
(b) the precise degree of participation therein by the accused."
Thus, when the accused files a demurrer, the court must evaluate whether the prosecution evidence is
sufficient enough to warrant the conviction of the accused beyond reasonable doubt. Same; Same;
Same; The grant of a demurrer to evidence amounts to acquittal and cannot be appealed because it
would place the accused in double jeopardy.— The grant or denial of a demurrer to evidence is left to
the sound discretion of the trial court, and its ruling on the matter shall not be disturbed in the absence
of a grave abuse of such discretion." As to effect, "the grant of a demurrer to evidence amounts to an
acquittal and cannot be appealed because it would place the accused in double jeopardy. The order is
reviewable only by certiorari if it was issued with grave abuse of discretion amounting to lack or excess
of jurisdiction." When grave abuse of discretion is present, an order granting a demurrer becomes null
and void.
Same; Same; Same; The party questioning the acquittal of an accused should be able to clearly establish
that the trial court blatantly abused its discretion such that it was deprived of its authority to dispense
justice.— Grave abuse of discretion is defined as "that capricious or whimsical exercise of judgment
which is tantamount to lack of jurisdiction. ‘The abuse of discretion must be patent and gross as to
amount to an evasion of a positive duty or a virtual refusal to perform a duty enjoined by law, or to act
at all in contemplation of law, as where the power is exercised in an arbitrary and despotic manner by
reason of passion and hostility.’ The party questioning the acquittal of an accused should be able to
clearly establish that the trial court blatantly abused its discretion such that it was deprived of its
authority to dispense justice."

Facts:
Respondent Jose Go executed fictitious loans in favor of two entities – Timmy’s, Inc. and Asia
Textile Mills, Inc. – in which two manager’s checks representing the supposed proceeds of these
fictitious loans were issued but made payable to two different entities – Philippine Recycler’s Inc. and
Zeta International – without any documents issued by the supposed borrowers Timmy’s, Inc. and Asia
Textile Mills, Inc. assigning the supposed loan proceeds to the two payees. Thereafter, these two
manager’s checks – together with several others were encashed, and then deposited in the OCBC
Savings Account of Jose Go. Then, several automatic transfer deposits were made from Jose Go’s
savings account to his OCBC Current Account which were then used to fund Go’s previously
dishonored personal checks. Upon knowledge of the same, PDIC filed a for two (2) counts of Estafa thru
Falsification of Commercial Documents in the Office of the City Prosecutor of the City of Manila
against the private respondents. After finding probable cause, the Office of the City Prosecutor of the
City of Manila filed Information against the private respondents.
Upon arraignment, private respondents pleaded not guilty. A pre-trial was conducted. Thereafter,
trial of the cases ensued and the prosecution presented its evidence. After the presentation of all of the
prosecution’s evidence, the private respondents filed a Motion for Leave to File Demurrer to Evidence to
which was granted by the RTC. Consequently, private respondents filed their Demurrer to Evidence. On
July 2, 2007, the RTC issued an Order granting the motion and dismissed the case. No motion for
reconsideration of the July 2, 2007 was filed by the handling public prosecutor, Only complainant
Philippine Deposit Insurance Corporation (PDIC) filed a Motion for Reconsideration. On January 4,
2008, the prosecution, through the Office of the Solicitor General (OSG), filed an original Petition for
Certiorari with the CA assailing the July 2, 2007 Order of the trial court. The CA denied the said petition
on the ground that the assailed July 2, 2007 Order of the trial court became final since the prosecution
failed to move for the reconsideration thereof, and thus double jeopardy attached. The assailed Order
was received by the Office of the City Prosecutor of Manila on July 3, 2007 and by the Private
Prosecutor on July 5, 2007. While the Private Prosecutor filed a Motion for Reconsideration of the said
Order, the Public Prosecutor did not seek for the reconsideration thereof. It is the Public Prosecutor who
has the authority to file a Motion for Reconsideration of the said order and the Solicitor General who can
file a petition for certiorari with respect to the criminal aspect of the cases. The failure of the Public
Prosecutor to file a Motion for Reconsideration on or before July 18, 2007 and the failure of the
Solicitor General to file a Petition for Certiorari on or before September 1, 2007 made the order of the
trial court final. The OSG moved for reconsideration, but in the assailed January 22, 2010 Resolution,
the CA stood its ground. Hence, the instant Petition was instituted. Petitioner admitted that the public
prosecution indeed filed the motion for reconsideration beyond the period allowed, begs the Court to
excuse its mistake in the name of public interest and substantial justice.

Issue:
Whether or not the Petition for Certiorary filed by the prosecution through the OSG belatedly assailing
the granting by the RTC and CA of private respondents Demurrer to Evidence be granted in the name of
public interest and substantial justice.

Ruling:
The Court granted the Petition. The power of courts to grant demurrer in criminal cases should
be exercised with great caution, because not only the rights of the accused - but those of the offended
party and the public interest as well - are involved. Once granted, the accused is acquitted and the
offended party may be left with no recourse. Thus, in the resolution of demurrers, judges must act with
utmost circumspection and must engage in intelligent deliberation and reflection, drawing on their
experience, the law and jurisprudence, and delicately evaluating the evidence on hand. Demurrer to the
evidence is "an objection by one of the parties in an action, to the effect that the evidence which his
adversary produced is insufficient in point of law, whether true or not, to make out a case or sustain the
issue. The party demurring challenges the sufficiency of the whole evidence to sustain a verdict. The
court, in passing upon the sufficiency of the evidence raised in a demurrer, is merely required to
ascertain whether there is competent or sufficient evidence to sustain the indictment or to support a
verdict of guilt. x x x Sufficient evidence for purposes of frustrating a demurrer thereto is such evidence
in character, weight or amount as will legally justify the judicial or official action demanded according
to the circumstances. To be considered sufficient therefore, the evidence must prove: (a) the commission
of the crime, and (b) the precise degree of participation therein by the accused." Thus, when the accused
files a demurrer, the court must evaluate whether the prosecution evidence is sufficient enough to
warrant the conviction of the accused beyond reasonable doubt. "The grant or denial of a demurrer to
evidence is left to the sound discretion of the trial court, and its ruling on the matter shall not be
disturbed in the absence of a grave abuse of such discretion." As to effect, "the grant of a demurrer to
evidence amounts to an acquittal and cannot be appealed because it would place the accused in double
jeopardy. The order is reviewable only by certiorariif it was issued with grave abuse of discretion
amounting tolack or excess of jurisdiction." When grave abuse of discretion is present, an order granting
a demurrer becomes null and void.
As a general rule, an order granting the accused’s demurrer to evidence amounts to an acquittal.
There are certain exceptions, however, as when the grant thereof would not violate the constitutional
proscription on double jeopardy. For instance, this Court ruled that when there is a finding that there was
grave abuse of discretion on the part of the trial court in dismissing a criminal case by granting the
accused’s demurrer to evidence,its judgment is considered void, as this Court ruled in People v. Laguio,
Jr.: By this time, it is settled that the appellate court may review dismissal orders of trial courts granting
an accused’s demurrer to evidence. This may be done via the special civil action of certiorariunder Rule
65 based on the ground of grave abuse of discretion, amounting to lack or excess of jurisdiction. Such
dismissal order, being considered void judgment, does not result in jeopardy. Thus, when the order of
dismissal is annulled or set aside by an appellate court in an original special civil action via certiorari,
the right of the accused against double jeopardy is not violated. In the instant case, having affirmed the
CA finding grave abuse of discretion on the part of the trial court when it granted the accused’s demurrer
to evidence, we deem its consequent order of acquittal void. The Court declares that the CA grossly
erred in affirming the trial court’s July 2, 2007 Order granting the respondent’s demurrer, which Order
was patently null and void for having been issued with grave abuse of discretion and manifest
irregularity, thus causing substantial injury to the banking industry and public interest. The Court finds
that the prosecution has presented competent evidence to sustain the indictment for the crime of estafa
through falsification of commercial documents, and that respondents appear to be the perpetrators
thereof. In evaluating the evidence, the trial court effectively failed and/or refused to weigh the
prosecution’s evidence against the respondents, which it was duty-bound to do as a trier of facts;
considering that the case involved hundreds of millions of pesos of OCBC depositors’ money – not to
mention that the banking industry is impressed with public interest, the trial court should have conducted
itself with circumspection and engaged in intelligent reflection in resolving the issues. Finally, it must be
borne in mind that "[t]he granting of a demurrer to evidence should x x x be exercised with caution,
taking into consideration not only the rights of the accused, but also the right of the private offended
party to be vindicated of the wrongdoing done against him, for if it is granted, the accused is acquitted
and the private complainant is generally left with no more remedy. In such instances, although the
decision of the court may be wrong, the accused can invoke his right against double jeopardy. Thus,
judges are reminded to be more diligent and circumspect in the performance of their duties as members
of the Bench xx x."

Case 72- Profugo


PEOPLE OF THE PHILIPPINES vs. JERRY PEPINO AND PRECIOSA GOMEZ
779 SCRA 170

Doctrine:
Questioning the validity of arrest: It is settled that any objection to the procedure followed in the matter
of the acquisition by a court of jurisdiction over the person of the accused must be raised before he
enters his plea; otherwise, the objection is deemed waived.

Facts:
Two men and a woman entered the office of Edward at Kilton Motors Corporation in Sucat,
Paranaque City, and pretended to be customers. When Edward was about to receive them one of the
men, eventually identified as Pepino pulled out a gun thinking that it was a holdup, Edward told Pepino
that the money was inside the cahier’s box. Pepino and the other man looted the cashier’s box,
handcuffed Edward, and forced him to go with them. The abductors then confined Edward in an
apartment in Quezon City where they insisted on asking ransom from Edward’s father. The exchange
was performed eventually with the abductors and Edwards Wife wherein after the exchange of money
and Edward being released, the victim reported the kidnapping to Teresita Ang See, a known anti-crime
crusader. After five months the National Bureau of Investigation informed Edward that they had
apprehended some suspects and invited him to identify them from a line-up consisting seven persons.
Pepino and Gomez did not testify for their defense. The defense instead presented Zeny Pepino,
Reynaldo Pepino who alleged that they were arrested without a valid warrant of arrest.

Issue:
Whether or not the illegal arrest of an accused is sufficient cause for selling aside a valid
judgment rendered upon a sufficient complaint after a trial free from error.

Ruling:
No, the Court pointed out t the outset that Gomez did not question before arraignment the
legality of her warrantless arrest or the acquisition of RTC’s jurisdiction over her person. Thus, Gomez
is deemed to have waived any objection to her warrantless arrest.
​It is settled that any objection to the procedure followed in the matter of the acquisition by a
court of jurisdiction over the person of the accused must be raised before he enters his plea; otherwise,
the objection is deemed waived. Appellant is now estopped from questioning any defect in the
information before the trial court. Consequently, any irregularity attendant to his arrest was cured when
he voluntarily submitted himself to the jurisdiction of the trial court by entering a plea of not guilty and
by participating in the trial. At any rate, the illegal arrest of an accused is not sufficient for setting aside
a valid judgement rendered upon a sufficient complaint after a trial free from error.
Case 73- Roxas

LLAMAS vs CA
GR NO. 149588 August 16, 2010

Doctrine:

The remedy of annulment of judgment cannot be availed of in criminal cases, - In People v. Bitanga (525
SCRA 623 [2007]), the Court explained that the remedy of annulment of judgment cannot be availed of
in criminal cases, thus – Section 1, Rule 47 of the Rules of court, limits the scope of the remedy of
annulment of judgment to the following: Section 1. Coverage.- This rule shall govern the annulment by
the Court of Appeals of judgments of final orders and resolutions in civil actions of Regional Trial
Courts for which the ordinary remedies of new trial, appeal, petition for relief or other appropriate
remedies are no longer available through no fault of the petitioner. The remedy cannot be resorted to
when the RTC judgment being questioned was rendered in a criminal case.

Jurisdiction being a matter of substantive law, the established rule is that the statute in force at the time
of the commencement of the action determines the jurisdiction of the court. In this case, at the time of
the filing of the information, the applicable law was Batas Pambansa Bilang 129, approved on August
14, 1981.

Facts:

Petitioners were charged before the Makati RTC with the crime of "other forms of swindling".
The RTC rendered its Decision on June 30, 1994, finding petitioners guilty beyond reasonable doubt of
the crime charged. On appeal, the Court of Appeals affirmed the decision of the trial court. Appellate
court further denied petitioners’ motion for reconsideration, thus, the judgment of conviction became
final and executory.

With the issuance by the trial court of the Warrant of Arrest, the police arrested petitioner
Carmelita C. Llamas. The police, however, failed to arrest petitioner Francisco R. Llamas because he
was nowhere to be found. Petitioner Francisco moved for the lifting or recall of the warrant of arrest,
raising for the first time the issue that the trial court had no jurisdiction over the offense charged.

There being no action taken by the trial court on the said motion, petitioners instituted the instant
proceedings for the annulment of the trial and the appellate courts’ decisions.

Issue:

Whether or not the petitioners can institute the annulment of judgment upon the decision of the
CA and the RTC for their inability to act upon the said motion.

Ruling :

No. The court held that, in People v. Bitanga, the Court explained that the remedy of annulment
of judgment cannot be availed of in criminal cases, thus —
Section 1, Rule 47 of the Rules of Court, limits the scope of the remedy of annulment of judgment to the
following:

Section 1. Coverage. — This Rule shall govern the annulment by the Court of Appeals of
judgments or final orders and resolutions in civil actions of Regional Trial Courts for which the
ordinary remedies of new trial, appeal, petition for relief or other appropriate remedies are no
longer available through no fault of the petitioner.

The remedy cannot be resorted to when the RTC judgment being questioned was rendered in a
criminal case. The 2000 Revised Rules of Criminal Procedure itself does not permit such recourse, for it
excluded Rule 47 from the enumeration of the provisions of the 1997 Revised Rules of Civil Procedure
which have suppletory application to criminal cases. Section 18, Rule 124 thereof, provides:

Sec. 18. Application of certain rules in civil procedure to criminal cases. – The provisions of
Rules 42, 44 to 46 and 48 to 56 relating to procedure in the Court of Appeals and in the Supreme
Court in original and appealed civil cases shall be applied to criminal cases insofar as they are
applicable and not inconsistent with the provisions of this Rule.

There is no basis in law or the rules, therefore, to extend the scope of Rule 47 to criminal cases.
As we explained in Macalalag v. Ombudsman, when there is no law or rule providing for this remedy,
recourse to it cannot be allowed.
Case 74- Casiano

NILO HIPOS vs. JUDGE TEODORO A. BAY


G.R. Nos. 174813-15 March 17, 2009

Doctrine:

Once criminal complaint or an information is filed in court, any disposition or dismissal of the case or
acquittal or conviction of the accused rests within the jurisdiction, competence and discretion of the trial
court.

Mandamus is never available to direct the exercise of judgment or discretion in a particular way or the
retraction or reversal of an action already taken in the exercise of either.

Facts:

Two informations for the crime of rape and one Information for the crime of acts of lasciviousness were
filed against petitioners Darryl Hipos, Jaycee Corsiño, Arthur Villaruel and two others others before
Branch 86 of the Regional Trial Court of Quezon City, acting as a Family Court, presided by respondent
Judge Bay. The private complainants AAA 1 and BBB filed a Motion for Reinvestigation asking Judge
Bay to order the City Prosecutor to study if the proper Informations had been filed. Judge Bay granted
the Motion and ordered a reinvestigation of the cases.

Petitioners on the other hand, filed memorandum to dismiss for lack of probable cause. On 10 August
2004 the City Prosecutor issued resolution on the investigation affirming the Informations signed
Assistant City Prosecutor Raniel S. Cruz and approved by City Prosecutor Claro A. Arellano.

On 3 March 2006, 2nd Assistant City Prosecutor Lamberto C. de Vera, reversed the Resolution dated 10
August 2004, holding that there was lack of probable cause and filed a Motion to Withdraw
Informations before Judge Bay

On 2 October 2006, Judge Bay denied the Motion to Withdraw Informations. The petitioners herein filed
the present Petition for Mandamus

Issue:

Whether or Not can Judge Bay be compelled through a Writ of Mandamus to allow the
withdrawal of the information

Ruling:

No, Mandamus is an extraordinary writ commanding a tribunal, corporation, board, officer or person,
immediately or at some other specified time, to do the act required to be done, when the respondent
unlawfully neglects the performance of an act which the law specifically enjoins as a duty resulting from
an office, trust, or station; or when the respondent excludes another from the use and enjoyment of a
right or office to which the latter is entitled, and there is no other plain, speedy and adequate remedy in
the ordinary course of law

As an extraordinary writ, the remedy of mandamus lies only to compel an officer to perform a
ministerial duty, not a discretionary one; mandamus will not issue to control the exercise of discretion by
a public officer where the law imposes upon him the duty to exercise his judgment in reference to any
manner in which he is required to act, because it is his judgment that is to be exercised and not that of
the court.

Further, mandamus is never available to direct the exercise of judgment or discretion in a


particular way or the retraction or reversal of an action already taken in the exercise of either. In other
words, while a judge refusing to act on a Motion to Withdraw Informations can be compelled by
mandamus to act on the same, he cannot be compelled to act in a certain way, i.e., to grant or deny such
Motion. In the case at bar, Judge Bay did not refuse to act on the Motion to Withdraw Informations; he
had already acted on it by denying the same. Accordingly, mandamus is not available anymore.

If petitioners believed that Judge Bay committed grave abuse of discretion in the issuance of
such Order denying the Motion to Withdraw Informations, the proper remedy of petitioners should have
been to file a Petition for Certiorari against the assailed Order of Judge Bay.

Case 75- Cajurao


PEOPLE VS. LORENZO
619 SCRA 389
Doctrine:

Presumption of innocence of accused is fleshed out by procedural rules which place on the prosecution
the burden of proving that an accused in guilty of offense charged by proof beyond reasonable doubt.
Conviction must rest on the strength of the prosecution’s evidence and not on the weakness of the
defense.

Facts:

Two Informations were filed against Lorenzo charging him violations on Section 5 and 11,
Article 2 of RA 9165 with respect to possessing of 2.04 grams of shabu and selling of 0.20 grams of
shabu. Estanislao charged in a different Information for possessing 0.05 grams of shabu. Both accused
entered ‘not guilty’ pleas. Prosecution presented its lone witness PO1 Noel Pineda. According to
defendants, Lorenzo was in his mountain bike on his way home while Estanislao was riding in his motor
cross style bike. The chain on Estanislao’s bike went loose. When he was repairing his bike, 3
policemen, who were then on board an owner type jeep, arrived and arrested Lorenzo and Estanislao. It
was later that they were informed that the arrest was for illegal drugs. RTC convicted Lorenzo while
acquitted Estanislao. CA affirmed RTC judgment.

Issue/s:

Whether or Not prosecution discharged its burden of proving Lorenzo’s guilt beyond reasonable
doubt for crime charged

Ruling:

No. In both illegal sale and illegal possession prohibited drugs, conviction cannot be sustained if
there is a persistent doubt on the identity of drug. It must be established with moral certainty. Non-
compliance with the stipulated procedure, under justifiable grounds, shall not render void and invalid
such seizures of and custody over said items, for as long as the integrity and evidentiary value of the
seized items are properly preserved by the apprehending officers. There must be a Chain of evidence.
However, in this case, it was not certain when and where the said marking was done nor who had

specifically received and had custody of the specimens thereafter. Prosecution did not present poseur-
buyer who had personal knowledge of the transaction. The lone witness of prosecution was at least four
meters away from accused and poseur-buyer. With that distance, it was impossible for him to hear the
conversation.

Facts and circumstances create a doubt as to whether sachets of shabu seized were the same one
that were released to Camp Crame and submitted for laboratory exam.
​Case 76- Profugo
PEOPLE OF THE PHILIPPINES vs. RENE BARON
G.R No. 185209 June 28, 2010

Doctrine:
Circumstantial evidence is sufficient for conviction if:
1) there is more than one circumstance;
2) the inference is based on prven faccts; and
3) the combination of all circumstances produces a conviction beyond reasonable doubt of the guilt of
the accused.

Facts:
Rene Baron y Tangarocan (Appellant), Rey Villatima, and alias “Dedong” Bargo was charged
with the special complex crime of robbery with homicide committed against Juanito Berallo. Only the
appellant was arrested. While Villatima and Bargo remained at large. Appellant entered a plea of “not
guilty” and denied any participation in the crime. RTC rendered a decision finding the appellant guilty
beyond reasonable doubt of the complex crime of robbery with homicide. Before the appellate court,
appellant alleged that the trial court erred in finding him guilty as charged and in not appreciating in his
favor the exempting circumstances of irresistible force and / or uncontrollable fear of an equal or greater
injury. However, the same was disregarded by the CA holding that all the requisites for said
circumstances were lacking. The appellate court found that the alleged threat, if at all, was not real or
imminent. Appellant had every opportunity to escape but did not take advantage of the same.

Issue:
Whether or not the circumstantial evidence is sufficient to render a judgment of conviction
against for the commission of the special complex crime of robbery with homicide.

Ruling:
Yes, although there is no direct evidence proving that the appellant conspired and participated in
committing the crime. However, his complicity may be proved by circumstantial evidence according to
reason and common experience. Circumstantial evidence is sufficient to sustain conviction if: a) there is
more than one circumstance; b) the facts from which the inferences are derived have been established; c)
the combination of all circumstances is such as to warrant a finding of guilt beyond reasonable doubt. A
judgement of conviction based on circumstantial evidence can be sustained when the circumstances
proved from an unbroken chain that results to a fair and reasonable conclusion pointing to the accused,
to the exclusion of all others, as the perpetrator.
​In this case, circumstantial evidence is sufficient to produce a conviction that the appellant
conspired with his co-accused in committing the crime of robbery with homicide. The concerted manner
in which the appellant and his companions perpetrated the crime showed beyond reasonable doubt the
presence of conspiracy.
Case 77- Fetilo

FELIXBERTO A. ABELLANA vs. PEOPLE OF THE PHILIPPINES and Spouses SAAPIA B.


ALONTO and DIAGA ALONTO
655 SCRA 683

Doctrine:

Judgment. It is an established rule in criminal procedure that a judgment of acquittal shall state
whether the evidence of the prosecution absolutely failed to prove the guilt of the accused or merely
failed to prove his guilt beyond reasonable doubt. In either case, the judgment shall determine if the act
or omission from which the civil liability might arise did not exist. When the exoneration is merely due
to the failure to prove the guilt of the accused beyond reasonable doubt, the court should award the civil
liability in favor of the offended party in the same criminal action. In other words, the “extinction of the
penal action does not carry with it the extinction of civil liability unless the extinction proceeds from a
declaration in a final judgment that the fact from which the civil liability might arise did not exist.”

Facts:

Abellana extended a loan to spouses Alonto secured by a Deed of Real Estate Mortgage over a
lot located in Cebu. Subsequently, Abellana conveyed the said lots and the spouses signed the Deed of
Absolute Sale. However, it was notarized without the spouses appearing before the notary public.
Thereafter Abellana caused the transfer of title to his name and sold the lots to third persons. An
Information was filed charging Abellana with Estafa through Falsification of Public Document and RTC
found him guilty.

The CA set aside the trial court’s Decision because it convicted petitioner of an offense different
from or not included in the crime charged in the Information. However, the RTC found that the spouses
Alonto actually signed the document although they did not personally appear before the notary public
for its notarization. Hence, the RTC instead convicted petitioner of falsification of public document.
On appeal, the CA held that petitioner’s conviction cannot be sustained because it infringed on his right
to be informed of the nature and cause of the accusation against him. The CA, however, found no
reversible error on the civil liability of petitioner as determined by the trial court and thus sustained the
same.

Issue:
Whether the judgement of acquittal of the crime of Falsification of a Public Document should
include the act or omission from which civil liability may arise.

Ruling:

Yes, the judgement of acquittal of the crime of Falsification of a Public Document includes the
act or omission from which civil liability may arise.

In Banal v. Tadeo, Jr., the Court elucidated on the civil liability of the accused despite his
exoneration in this wise: “While an act or omission is felonious because it is punishable by law, it gives
rise to civil liability not so much because it is a crime but because it caused damage to another. Viewing
things pragmatically, we can readily see that what gives rise to the civil liability is really the obligation
and moral duty of everyone to repair or make whole the damage caused to another by reason of his own
act or omission, done intentionally or negligently, whether or not the same be punishable by law. x x x”

Simply stated, civil liability arises when one, by reason of his own act or omission, done
intentionally or negligently, causes damage to another. Hence, for petitioner to be civilly liable to
spouses Alonto, it must be proven that the acts he committed had caused damage to the spouses. Based
on the records of the case, we find that the acts allegedly committed by the petitioner did not cause any
damage to spouses Alonto.
Case 78- Garcia
PEOPLE vs. ASIS
629 SCRA 250

Doctrine:
Circumstantial evidence that merely arouses suspicions or gives room for conjecture is not sufficient to
convict. It must do more than just raise the possibility, or even the probability, of guilt. It must engender
moral certainty. Otherwise, the constitutional presumption of innocence prevails, and the accused
deserves acquittal.

Facts:
George Huang, the nephew of the victim Yu Hing Guan a.k.a. Roy Ching, always passes by the
store of the victim at 1042 Benavidez Street, Binondo, Manila to bring food stuff, ice and other things to
his uncle and mother, Diana Yu, who work[s] in the office of said store.
On February 9, 1998, at around 6:30 oclock in the morning, Huang arrived at the victims store
and discovered that the steel door of the store was locked from the outside. When he opened the steel
door, he found everything to be normal except for the inner door which had always been left open but
which was closed at that time with only a chair blocking it.
When he removed the blocking chair, he discovered the body of his uncle, Yu Hing Guan a.k.a.
Roy Ching (victim), lying prostrate on the ground with a knife embedded on his nape. He closed the
door and proceeded to Luneta, where [his] mother exercises, to inform her of what he saw. After
informing [his] mother, Huang first went to the Chinatown Police Station and reported the incident;
thereafter, he went to another station located in Soler corner Reina Regente to report the incident again.
Diana Yu, the sister of the victim, testified that on February 9, 1998, before 8:30 oclock in the
evening, she was in the office of her brother where she was working at 1042 Benavidez St., Binondo,
Manila. She saw the two appellants, namely: Danilo Asis and Gilbert Formento, and her brother (the
victim), who are all deaf-mutes, talking in sign language. She testified that Danilo Asis frequented the
office of the victim, while Gilbert Formento came only on the night of February 9, 1998. At around 8:30
oclock in the evening, she left the office, leaving both appellants and the victim behind. The following
morning, at around 7:30 oclock in the morning, her son, George Huang, informed her of her brothers
(victims) death. Upon learning of said incident, she went to the office where she saw her brothers body.
She discovered that the sales proceeds of the preceding day were missing and the necklace of her brother
(victim) which he always wore was also missing.
During the course of investigation, SPO1 Cabatbat received a phone call from a relative
informing him that one of the suspects, appellant Danilo Asis, went back to the scene of the crime.
Afterwards, they brought Danilo Asis to the police station for investigation, who expectedly denied
having anything to do with the killing of the victim.
During investigation (February 10, 1998), SPO1 Balatbat noticed that there was a bloodstain in
Asis T-shirt.
During the presentation of prosecution witness Dr. Olga Bausa, they stipulated that the
bloodstains found in the white t-shirt with a lettering of Colorific and in the short pants were human
blood. (Citations omitted)
Issue:
Whether or not the evidence is admissible?
Ruling:
No.
In any event, appellants argument of illegal search and seizure cannot simply be brushed aside,
considering the guarantee so sacredly enshrined in our Constitution.
There is no question that appellants were collared without any arrest warrant. Neither was there
any valid search warrant obtained against them. However, they never questioned the legality of their
arrest through a motion to quash the Information. Instead, they entered a plea of not guilty and
participated in the trial. Settled is the rule that any objection involving the arrest or the trial courts
procedure of acquiring jurisdiction over the person of the accused must be made before the arraignment;
otherwise, the objection is deemed waived.
Indeed, appellants do not now question the legality of their arrest. What they object to is the
introduction of the bloodstained pair of shorts allegedly recovered from the bag of Appellant
Formento. They argue that the search was illegally done, making the obtainment of the pair of shorts
illegal and taints them as inadmissible. The prosecution, on the other hand, contends that it was the wife
of appellant who voluntarily surrendered the bag that contained the bloodstained trousers of the victim.
Her act, it claims, constituted a valid consent to the search without a warrant.
To constitute a valid waiver, it must be shown that first, the right exists; second, the person
involved had knowledge, actual or constructive, of the existence of such a right; and third, the person
had an actual intention to relinquish the right. How could Appellant Formento have consented to a
warrantless search when, in the first place, he did not understand what was happening at that moment?
The prosecution witnesses themselves testified that there was no interpreter to assist him -- a deaf-mute -
- during the arrest, search and seizure. Naturally, it would seem that he indeed consented to the
warrantless search, as the prosecution would want this Court to believe.
As early as 1938, Justice Jose P. Laurel pointed out in Pasion vda. de Garcia v. Locsin:
As the constitutional guaranty is not dependent upon any affirmative act of the citizen, the courts
do not place the citizen in the position of either contesting an officers authority by force, or waiving his
constitutional rights; but instead they hold that a peaceful submission to a search or seizure is not a
consent or an invitation thereto, but is merely a demonstration of regard for the supremacy of the law.
This point becomes even more pronounced in the present case, in which appellant is a deaf-mute,
and there was no interpreter to explain to him what was happening. His seeming acquiescence to the
search without a warrant may be attributed to plain and simple confusion and ignorance.
Verily, courts indulge every reasonable presumption against waiver of fundamental constitutional rights
and x x x we do not presume acquiescence [to] the loss of fundamental rights.
Neither can the OSG claim that appellants wife voluntarily surrendered the bag that contained
the bloodstained trousers of the victim. As admitted by Prosecution Witness PO2 Pablo Ileto, the victims
sister and appellants wife cannot understand each other. Eventually, appellants wife gave the belongings
of Gilbert Formen to where the bloodstained shorts of the victim were recovered. How can the surrender
of appellants belongings in this case be voluntary, when the person surrendering them did not even
understand the person she was communicating with?
All told, the bloodstained pair of shorts was a piece of evidence seized on the occasion of an
unlawful search and seizure. Thus, it is tainted and should thus be excluded for being the proverbial fruit
of the poisonous tree. In the language of the fundamental law, it shall be inadmissible in evidence for
any purpose in any proceeding
Case 79- Lasam

RODOLFO BASILONIA vs. HON. DELANO F. VILLARUZ


G.R. Nos. 191370-71 August 10, 2015

Doctrines:
Aside from the civil indemnity arising from the crime, costs and incidental expenses of the suit are part
of the judgment and it is incumbent upon the prevailing party in whose favor they are awarded to submit
forthwith the itemized bill to the clerk of court.
Once a judgment of conviction becomes final and executory, the trial court has the ministerial duty to
immediately execute the penalty of imprisonment and/or pecuniary penalty (fine). A motion to execute
judgment of conviction is not necessary.
In cases where the accused is a detention prisoner, i.e., those convicted of capital offenses or convicted
of noncapital offenses where bail is denied, or refused to post bail, a ed to post bail, a mittimus or
commitment order should be immediately issued after the promulgation of judgment by the trial court as
long as the penalty imposed requires the service of sentence in the National Penitentiary.

Facts:
In this case, a decision was promulgated against petitioners for capital offenses. Thus, petitioners
filed a Notice of Appeal on July 30, 1987, which the trial court granted on August 3, 1987. On January
23, 1989, failure of petitioners to file brief despite extensions of time given, the CA dismissed the appeal

and the resolution was entered in the Book of Entries of Judgment on September 18, 1989. Thereafter,
the entire case records were remanded to the trial court on October 4, 1989. Almost two decades passed
from the entry of judgment, on May 11, 2009, private respondent Dixon C. Roblete, claiming to be the
son of the deceased victim, Atty. Roblete, filed a Motion for Execution of Judgment. He alleged, among
others, that despite his request to the City Prosecutor to file a motion for execution, the judgment has not
been enforced because said prosecutor has not acted upon his request.
Issue:
Whether respondent trial court committed grave abuse of discretion amounting to lack or excess
of jurisdiction in granting a motion for execution which was filed almost twenty (20) years after a
judgment in a criminal case became final and executory.

Ruling:
No, the trial court did not commit grave abuse of discretion. Upon the judgment, the period for
prescription of actions to demand the fulfillment of obligations declared by a judgment commences from
the time the judgment became final. These two modes of execution are available depending on the
timing when the judgment creditor invoked its right to enforce the court's judgment. Execution by
motion is only available if the enforcement of the judgment was sought within five (5) years from the
date of its entry.
An action for revival of judgment is not intended to reopen any issue affecting the merits of the
case or the propriety or correctness of the first judgment.The purpose is not to re-examine and re-try
issues already decided but to revive the judgment; its cause of action is the judgment itself and not the
merits of the original action. However, being a mere right of action, the judgment sought to be revived is
subject to defenses and counterclaims like matters of jurisdiction and those arising after the finality of
the first judgment or which may have arisen subsequent to the date it became effective such as
prescription, payment, or counterclaims arising out of transactions not connected with the former
controversy.
In the instant case, it is obvious that the heirs of Atty. Roblete did not file a motion for execution
within the five-year period or an action to revive the judgment within the ten-year period. Worse, other
than the bare allegation that the judgment hasnot been enforced because the public prosecutor has not
acted on the request to file a motion for execution, no persuasive and compelling reason was presented
to warrant the exercise of Our equity jurisdiction. Unfortunately for private respondent Roblete, the
instant case does not fall within the exceptions afore-stated. It cannot be claimed that the delay in
execution was entirely beyond their control or that petitioners have any hand in causing the same. As
regards the civil aspect of a criminal case is concerned, it is apt to point that — Litigants represented by
counsel should not expect that all they need to do is sit back and relax, and await the outcome of their
case.

Case 80- Marcelo


ARMILYN MORILLO vs. PEOPLE OF THE PHILIPPINES AND RICHARD NATIVIDAD
777 SCRA 207

Doctrine: Dismissal of the case amounts to acquittal of the accused

Facts:

Sometime in July 2003, respondent Richard Natividad, Milo Malong and Bing Nanquil,
introducing themselves as contractors doing business in Pampanga City under the name and style of RB
Custodio Construction, purchased construction materials for their project inside the Subic Freeport Zone
from petitioner Armilyn Morillo, owner of Amasea General Merchandize and Construction Supplies.
The parties agreed that twenty percent (20%) of the purchases shall be paid within seven (7) days after
the first delivery and the remaining eighty percent (80%) to be paid within thirty-five (35) days after the
last delivery, all of which shall be via postdated checks.

Pursuant to the agreement, petitioner delivered construction materials amounting to a total of


P500,054.00 at the construction site where respondent and his partners were undertaking their project.
After the last delivery, respondent paid P20,000.00 in cash and issued two (2) post-dated checks, drawn
from Metrobank, Pampanga branch, in the amounts of P393,000.00 and P87,054.00. Upon maturity,
petitioner attempted to deposit the checks in her savings account at Equitable PCI Bank, San Lorenzo,
Makati City. They were, however, dishonored by the drawee bank. Immediately thereafter, petitioner
communicated the dishonor to respondent and his partners and demanded for payment. Again,
respondent issued two (2) post-dated Metrobank checks and assured petitioner that they will be honored
upon maturity. Upon deposit in her savings account at Equitable PCI Bank, Makati Branch, the checks
were once again dishonored for the reason that the account from which they were drawn was already a
closed account.

Consequently, petitioner made several demands from respondent and his partners, but to no avail,
prompting her to file a complaint with the City Prosecution Office, Makati City. Thus, on August 12,
2004, two (2) Informations were filed against respondent and Milo Malong.

ISSUE:

Whether or not the dismissal of the case tantamount to the acquittal of the case?

RULING:

Yes. It is well settled that violation of BP 22 cases is categorized as transitory or continuing


crimes, which means that the acts material and essential thereto occur in one municipality or territory,
while some occur in another. Thus, a person charged with a continuing or transitory crime may be
validly tried in any municipality or territory where the offense was in part committed.

Guided by the foregoing pronouncements, there is no denying, therefore, that the check subject of the
instant case was drawn, issued, and delivered in Pampanga does not strip off the Makati MeTC of its
jurisdiction over the instant case for it is undisputed that the subject check was deposited and presented
for encashment at the Makati Branch of Equitable PC IBank. The MeTC of Makati, therefore, correctly
took cognizance of the instant case and rendered its decision in the proper exercise of its jurisdiction.
First of all, the Court stresses that the appellate court’s dismissal of the case is not an acquittal of
respondent. Basic is the rule that a dismissal of a case is different from an acquittal of the accused
therein. Except in a dismissal based on a Demurrer to Evidence filed by the accused, or for violation of
the right of the accused to a speedy trial, the dismissal of a criminal case against the accused will not
result in his acquittal.

Thus, when the appellate court herein dismissed the instant case on the ground that the MeTC
lacked jurisdiction over the offense charged, it did not decide the same on the merits, let alone resolve
the issue of respondent’s guilt or innocence based on the evidence proffered by the prosecution. The
appellate court merely dismissed the case on the erroneous reasoning that none of the elements of BP 22
was committed within the lower court’s jurisdiction, and not because of any finding that the evidence
failed to show respondent’s guilt beyond reasonable doubt. Clearly, therefore, such dismissal did not
operate as an acquittal, which, as previously discussed, may be repudiated only by a petition for
certiorari under Rule 65 of the Rules of Court, showing a grave abuse of discretion.

Case 81- Lukban

MUNIB S. ESTINO and ERNESTO G. PESCADERA vs . PEOPLE OF THE PHILIPPINES


G.R. Nos. 163957-58. April 7, 2009
Doctrine:
Rule 121 of the Rules of Court allows the conduct of a new trial before a judgment of conviction
becomes final when new and material evidence has been discovered which the accused could not with
reasonable diligence have discovered and produced at the trial and which if introduced and admitted
would probably change the judgment. Although the documents offered by petitioners are strictly not
newly discovered, it appears to us that petitioners were mistaken in their belief that its production
during trial was unnecessary.
Facts:
For review before the Court under Rule 45 are the April 16, 2004 Decision and June 14, 2004
Resolution of the Sandiganbayan in the consolidated Criminal Case Nos. 26192 and 26193 entitled
People of the Philippines v. Munib S. Estino and Ernesto G. Pescadera. In G.R. Nos. 163957-58,
petitioners Munib S. Estino and Ernesto G. Pescadera appeal their conviction of violation of Section
3(e), Republic Act No. (RA) 3019 or the Anti-Graft and Corrupt Practices Act for failure to pay the
Representation and Transportation Allowance (RATA) of the provincial government employees of Sulu.
In G.R. Nos. 164009-11, petitioner Pescadera alone appeals his conviction of malversation of public
funds under Article 217 of the Revised Penal Code for failure to remit the Government Service
Insurance System (GSIS) contributions of the provincial government employees amounting to PhP
4,820,365.30.
In these consolidated appeals, petitioners pray for their acquittal.
Estino was elected Vice-Governor of Sulu in the May 1998 elections along with Gov. Abdusakur
Tan. On June 23, 1998, this Court issued a status quo order in G.R. No. 133676, suspending the effects
of the proclamation of Gov. Tan and ordering Vice-Gov. Estino to assume the position of Governor until
further orders. Thus, Estino acted as Governor of Sulu from July 27, 1998 up to May 23, 1999 when this
Court lifted the suspension order against Gov. Tan. Ernesto G. Pescadera, on the other hand, was
Provincial Treasurer of Sulu during Estino's stint as Acting Governor. Pursuant to Commission on Audit
(COA) ARMM Office Order No. 99-165 dated August 26, 1999, a special audit team was created upon
the request of the Provincial Government of Sulu. An audit of the disbursement vouchers and payrolls
for the period starting July 27, 1998 up to May 23, 1999 was then conducted by COA State Auditor II
Mona U. Balabaran and her team. The COA Special Audit Report stated that there were anomalies in the
payment of salary differentials, allowances, and benefits, among others.
The Sandiganbayan found the accused both guilty beyond reasonable doubt for violation of Sec.
3(e) of R.A. 3019, and pursuant to Section 9 thereof. Hence, Petitioners filed a Motion for
Reconsideration and a Supplemental Motion for Reconsideration and New Trial which were denied in
the June 14, 2004 Sandiganbayan Resolution.
Issue:
Whether or not a new trial is proper in the determination the guilt of the petitioners in non-
payment of RATA in violation of Sec 3(e) of RA 3019.
Ruling:
YES. Petitioners’ defense is anchored on their payment of RATA, and for this purpose, they
submitted documents which allegedly show that they paid the RATA under the 1998 reenacted budget.
They also claim that the COA Report did not sufficiently prove that they did not pay the RATA because
the alleged disbursement vouchers, which were supposed to be annexed to the COA Report as proof of
nonpayment of RATA, were not submitted with said report.
Rule 121 of the Rules of Court allows the conduct of a new trial before a judgment of conviction
becomes final when new and material evidence has been discovered which the accused could not with
reasonable diligence have discovered and produced at the trial and which if introduced and admitted
would probably change the judgment. Although the documents offered by petitioners are strictly not
newly discovered, it appears to us that petitioners were mistaken in their belief that its production during
trial was unnecessary.
Case 82- Marcos

BRIONES vs. PEOPLE OF THE PHILIPPINES


GR NO. 156009 June 5, 2009

Doctrine:
1. An error or mistake committed by a counsel in the course of judicial proceedings is not a ground for
new trial.
2. Conditions for new trial to be granted on the ground of newly discovered evidence includes: that the
evidence was discovered after trial; that such evidence could not have been discovered and produced at
the trial even with the exercise of reasonable diligence; that it is material of such weight that it would
probably change the judgement if admitted.

Facts:
A criminal information was filed against Briones for crime of robbery. Briones allegedly took the
service firearm of S/G Gual while the latter approached the group where the former is involved in a
mauling. S/G Gual positively identified Briones. RTC found Briones guilty of the crime of simple theft
(Art. 309 Par. 3 of RPC) after giving weight to prosecutions positive testimony as against the defenses of
denial and alibi.

On his appeal, he raised the issue of self-defense. The Court of Appeals found Briones guilty of
robbery under Article 293 in relation to par. 5 of Article 294 of RPC and not of theft.

Issue:
Whether or not a new trial may be granted on the ground of newly discovered evidence.

Ruling:
No. The for new trial to be granted on the ground of newly discovered evidence, the concurrence
of the following conditions must obtain:

(a) the evidence must have been discovered after trial;


(b) the evidence could not have been discovered at the trial even with the exercise of reasonable
diligence;
(c) the evidence is material, not merely cumulative, corroborative, or impeaching; and
(d) the evidence must affect the merits of the case and produce a different result if admitted.

In this case, although the firearm surfaced after the trial, the other conditions were not
established.
Evidence to be newly discovered, must be one that could not, by exercise of due diligence, have been
discovered by the court below. Briones failed to show he had exerted reasonable diligence to locate
the firearm.

The allegation the he told his brothers and sisters to search for the firearm, which yielded in
negative result is purely self-serving. He now admits having taking the firearm and immediately
disposed of it at a nearby house, adjacent to the place of the incident.

Hence, even before the case went to court, he already knew the location of the subject firearm,
but did not do anything he did not even declare this knowledge at the trial below.

In petitions for new trial in a criminal proceeding where certain evidence was not presented, the
defendant, in order to secure a new trial, must satisfy the court that; he has a good defense, and that the
acquittal would in all probability follow the introduction of the omitted evidence. Briones change of
defense from denial to alibi to self-defense or in defense of a relative will not change the outcome.
Case 83- Velasco

SALUDAGA VS SANDIGANBAYAN
GR NO. 184537 APRIL 23, 2010

Doctrine: Rule 121 New Trial or Reconsideration, Ground; effect

Facts:
This is a petition for certiorari, prohibition and mandamus under Rule 65 of the 1997 Rules on
Civil Procedure with a prayer for the issuance of a writ of preliminary injunction and temporary
restraining order assailing the July 14, 2008 Resolution of the Sandiganbayan in Criminal, denying the
Motion for Preliminary Investigation filed by the petitioners who were charged with a violation of
Section 3(e) of Republic Act No. 3019. At first, the Information was quashed for lack of amount of
actual damaged cause in the alleged crime which is essential. The information was re-filed thus,
petitioners filed a Motion for Preliminary Investigation which was strongly opposed by the prosecution
in its Opposition. Petitioners contend that there was a substitution of the first Information which should
have preceded by a preliminary investigation. Further, they claim that newly discovered evidence
mandates re-examination of the finding of a prima facie cause to file the case. The prosecutors on the
other hand argues that the re-filed information did not change the nature of the offense charged, but
merely modified the mode by which accused committed the offense. The substance of such modification
is not such as to necessitate the conduct of another preliminary investigation. Moreover, no new
allegations were made, nor was the criminal liability of the accused upgraded in the re-filed
information. Thus, new preliminary investigation is not in order. The motions filed by the petitioners
were denied by the court hence, this petition to the highest court.

Issues:

Whether or Not the Honorable Sandiganbayan acted with grave abuse of discretion amounting to
lack or excess of jurisdiction when it refused to order the preliminary Investigation and if so, is such a
sufficient ground for new trial.

Ruling:
The court found no merit on the petition. Petitioners insist that the offenses charged in the first
and second Information are not the same, and what transpired was a substitution of Information that
required prior conduct of preliminary investigation. Even assuming there was no substitution, substantial
amendments were made in the second Information, and that its submission should have been preceded
by a new preliminary investigation.

The court ruled that the use of the disjunctive term “or” connotes that either act qualifies as a violation
of Section 3 paragraph (e), as 2 different modes of committing the offense.

This does not however indicate that each mode constitutes a distinct offense, but rather, that an accused
may be charged under either mode or under both which implies that there’s no substituted information.
Case 84- Mortel
LUMANOG vs. PEOPLE OF THE PHILIPPINES
G.R. NO. 182555 ​SEPTEMBER 7, 2010
Doctrine: To justify new trial, it must be shown that the evidence was newly discovered.

Facts: ​
Appellants were the accused perpetrators of the ambush-slay of former Chief of the Metropolitan
Command Intelligence and Security Group of the Philippine Constabulary (now Philippine National
Police), Colonel Rolando N. Abadilla.
These cases were consolidated. The Supreme Court affirmed the decision of the Court of
Appeals convicting Lumanog, Santos, Fortuna, and De Jesus of the crime murder for the death of Col.
Abadilla. Lumanog and Santos separately filed their motions for reconsideration seeking the reversal of
their conviction. They assailed the weight and credence accorded to the identification of the accused by
the lone eyewitness presented by the prosecution, security guard Freddie Alejo. Fortuna submitted an
affidavit executed by a certain Orencio Jurado, Jr., who claimed to be one of the police officers initially
assigned to investigate the case. Fortuna contended that said belated statement would certainly case
doubt on the procedures undertaken by the police authorities in the apprehension of the likely
perpetrators. ​
Issue: ​ ​
Whether or not introduction of additional evidence after the trial is valid to justify new trial.
Ruling: ​
No. Introduction of additional evidence after trial is not valid to justify new trial. Evidently,
Fortuna seeks the introduction of additional evidence to support the defense argument that there was no
positive identification of Abadilla’s killers. To justify a new trial or setting aside of judgment of
conviction on the basis of such evidence, it must be shown that the evidence was “newly discovered”
pursuant to Sec. 2, Rule 121 of the Revised Rules of Criminal Procedure, as amended. Evidence, to be
considered newly discovered, must be one that could not, by the exercise of due diligence, have been
discovered before the trial in the court below.
Movant failed to show that the defense exerted efforts during the trial to secure testimonies from
police officers like Jurado, or other persons involved in the investigation, who questioned or objected to
the apprehension of the accused in this case. Hence, the belatedly executed affidavit of Jurado does not
qualify as newly discovered evidence that will justify re-opening of the trial and/or vacating the
judgment. In any case, we have ruled that whatever flaw that may have initially attended the out-of-court
identification of the accused, the same was cured when all the accused-appellant were positively
identified by the prosecution witness during the trial.
Case 85- Velasco
Payumo v. Sandiganbayan
654 SCRA 277, July 25, 2011

Doctrine: Rule 121 grounds for new trial when there is newly discovered evidence
Facts:
The petitions stem from the facts of Criminal Case No. 4219 involving a shooting incident that
occurred on February 26, 1980 at around 5:30 o'clock in the afternoon in SitioAluag, Barangay Sta.
Barbara, Iba, Zambales. A composite team of Philippine Constabulary (PC) and Integrated National
Police (INP) units allegedly fired at a group of civilians instantly killing one and wounding several
others. The accused were indicted for Murder with Multiple Frustrated and Attempted Murder before the
Sandiganbayan.
After four years of trial, the Second Division of the Sandiganbayan rendered its Decision dated
October 5, 1984, convicting the accused as co-principals in the crime of Murder with Multiple
Frustrated and Attempted Murder. On January 11, 1985, the accused filed their Motion for New Trial,
which was denied. The accused elevated the case to the Supreme Court, which set aside the October 5,
1984 Decision of the Sandiganbayan and remanding the case for a new trial. Thus, Criminal Case No.
4219 was remanded to the Sandiganbayan and was raffled to the First Division. Accordingly, the First
Division received anew all the evidence of the parties, both testimonial and documentary. Later, with the
creation of the Fourth and Fifth divisions, Criminal Case No. 4219 was transferred to the Fifth Division.
On February 23, 1999, the Fifth Division promulgated judgment, convicting the accused of the
crime of Murder with Multiple Attempted Murder. The accused filed their Omnibus Motion to Set Aside
Judgment and for New Trial. Since the Fifth Division could not reach unanimity in resolving the
aforesaid omnibus motion, a Special Fifth Division was constituted pursuant to Section 1 (b) of Rule
XVIII of the 1984 Revised Rules of the Sandiganbayan. On September 27, 2001, Special Fifth Division,
voting 3-2, issued the subject Resolution promulgated on October 24, 2001, setting aside the November
27, 1998 Decision and granting a second new trial of the case. The Special Fifth Division pronounced
among others that a second new trial would enable it to allow the accused to adduce pertinent evidence
including the records of the Judge Advocate General Office (JAGO), Armed Forces of the Philippines,
to shed light on the "serious allegations"

Issue:
Whether the Sandiganbayan acted in excess of its jurisdiction when it granted a new trial of
Criminal Case

Ruling:
Yes. The Court finds and so rules that the Sandiganbayan Special Fifth Division acted in excess
of its jurisdiction. The Court finds the issue to be devoid of any legal and factual basis. Rule 121,
Section 2(b) of the 2000 Rules on Criminal Procedure provides that: new and material evidence has been
discovered which the accused could not with reasonable diligence have discovered and produced at the
trial and which if introduced and admitted would probably change the judgment, as one of the grounds
to grant a new trial. The records of the JAGO relative to the February 26, 1980 incident do not meet the
criteria for newly discovered evidence that would merit a new trial. A motion for new trial based on
newly-discovered evidence may be granted only if the following requisites are met: (a) that the evidence
was discovered after trial; (b) that said evidence could not have been discovered and produced at the
trial even with the exercise of reasonable diligence; (c) that it is material, not merely cumulative,
corroborative or impeaching; and (d) that the evidence is of such weight that, if admitted, would
probably change the judgment. It is essential that the offering party exercised reasonable diligence in
seeking to locate the evidence before or during trial but nonetheless failed to secure it. In this case,
however, such records could have been easily obtained by the accused and could have been presented
during the trial with the exercise of reasonable diligence. Hence, the JAGO records cannot be considered
as newly discovered evidence. There was nothing that prevented the accused from using these records
during the trial to substantiate their position that the shooting incident was a result of a military
operation. Accordingly, the assailed Resolution dated October 24, 2001 must be set aside.

Case 86- Oineza


MACAPAGAL vs. PEOPLE OF THE PHILIPPINES
717 SCRA 425
Doctrine:
The disallowance of the notice of appeal signifies the disallowance of the appeal itself. A petition for
review under Rule 45 of the Rules of Court is a mode of appeal of a lower court’s decision or final order
direct to the Supreme Court. However, the questioned Order denying her notice of appeal is not a
decision or final order from which an appeal may be taken. The Rules of Court specifically provides that
no appeal shall be taken from an order disallowing or dismissing an appeal. Rather, the aggrieved party
can elevate the matter through a special civil action under Rule 65. Thus, in availing of the wrong mode
of appeal in this petition under Rule 45 instead of the appropriate remedy of Rule 65, the petition merits
an outright dismissal.
Facts:
On November 25, 2008, the RTC rendered a decision finding petitioner guilty of the crime of
Estafa for misappropriating, for her own benefit, the total amount of P800,000.00, which is the value of
the unreturned and unsold pieces of jewelry. Petitioner received the decision on January 13, 2009 then
she timely moved for reconsideration, but was likewise denied in an Order dated May 20, 2009 which
the petitioner allegedly received on July 31, 2009. She supposedly filed a Notice of Appeal. On August
3, 2009, but the same was denied on June 29, 2010 for having been filed out of time.
Issue:
Whether or not the regional trial court of manila, Branch 9 gravely erred in denying the notice of
appeal filed by the herein petitioner?
Held:
The Court notes that the instant case suffers from various procedural infirmities which this Court
cannot ignore and are fatal to petitioner’s cause. It appears that petitioner assails not only the denial by
the RTC of her notice of appeal but likewise seeks the reversal of her conviction for estafa. For reasons
that will be discussed below, the petition is bound to fail, because of petitioner’s complete disregard of
the procedural rules and the orders of the Court.First, petitioner availed of the wrong mode of assailing
the trial court’s denial of her notice of appeal. Sections 2 and 3, Rule 122 of the Revised Rules of
Criminal Procedure lay down the rules on where, how and when appeal is taken.
Second, even if we treat this petition as one for certiorari under Rule 65, it is still dismissible for
violation of the hierarchy of courts. Although the Supreme Court has concurrent jurisdiction with the
RTC and the CA to issue writs of certiorari, this should not be taken as granting parties the absolute and
unrestrained freedom of choice of the court to which an application will be directed. Direct resort to this
Court is allowed only if there are special, important and compelling reasons clearly and specifically
spelled out in the petition, which are not present in this case.
Third, even if we ignore the above non-compliance and consider the petition as an appeal of the
trial court’s decision convicting her of estafa, again, we cannot do so for yet another fatal procedural
shortcoming committed by petitioner. As stated earlier, petitioner elevated to this Court not only the
Order denying her notice of appeal but also the Decision convicting her of estafa and the Order denying
her motion for reconsideration. In utter disregard of the rules of procedure, petitioner attached to the
petition only the June 29, 2010 RTC Order denying her notice of appeal but she failed to attach a clearly
legible duplicate original or a certified true copy of the assailed decision convicting her of estafa and the
order denying her motion for reconsideration. A petition for review on certiorari under Rule 45 of the
Rules of Court must contain a certified true copy or duplicate original of the assailed decision, final
order or judgment. Failure to comply with such requirement shall be sufficient ground for the dismissal
of the petition.

Case 87- Olaivar


PEOPLE vs MORALES
616 SCRA 233
Doctrine: Appeals; A unique nature of an appeal in a criminal case is that the appeal throws the whole
case open for review and it is the duty of the appellate court to correct, cite and appreciate errors in the
appealed judgment whether they are assigned or unassigned. We draw attention to the unique nature of
an appeal in criminal case: the appeal throws the whole case open for review and it is the duty of the
appellate court to correct, cite and appreciate errors in the appealed judgment whether they are
assigned or unassigned. On the basis of such review, we find the present appeal meritorious.
Facts:
PO1 Eduardo Roy and PO3 Armando Rivera of Quezon City Police District made a buy-bust
operation and caught Morales having two sachets of shabu together with marked money. Morales, on his
part averred that two male persons in civilian clothing who identified themselves as police officers
handcuffed and frisk him without finding anything. On their way to the police station, Roy produced a
sachet of shabu from his pocket and upon reaching the station presented it as evidence against Morales.
Quezon City RTC found Morales guilty. The Court of Appeals affirmed the trial court’s decision. Hence,
this appeal.
Issue:
Whether or not the concurring judgment of RTC and CA be disturbed on appeal to SC.
Ruling:
Supreme Court draw attention to the unique nature of an appeal in a criminal case: the appeal
throws the whole case open for review and it is the duty of the appellate court to correct, cite and
appreciate errors in the appealed judgment whether they are assigned or unassigned. Prevailing
jurisprudence uniformly hold that the trial courts findings of fact, especially when affirmed by the CA,
are, as a general rule, entitled to great weight and will not be disturbed on appeal. However, this rule
admits of exceptions and does not apply where facts of weight and substance with direct and material
bearing on the final outcome of the case have been overlooked, misapprehended or misapplied.
Evidence presented and relevant law and jurisprudence, we hold that this case falls under the exception.
Furthermore, the evidence presented by the prosecution failed to reveal the identity of the person
who had custody and safekeeping of the drugs after its examination and pending presentation in court.
Thus, the prosecution likewise failed to establish the chain of custody which is fatal to its cause. The
identity of the corpus delicti in this case was not proven beyond reasonable doubt. There was likewise a
break in the chain of custody which proves fatal to the prosecutions’ case. Since the prosecution has
failed to establish the element of corpus delicti with the prescribed degree of proof required for
successful prosecution of both possession and sale of prohibited drugs, we resolve to ACQUIT Roldan
Morales.
Case 88- Remigio
QUIDET vs PEOPLE
618 SCRA 1
Doctrine:
Even if co-accused did not appeal, they are still entitled to the favorable terms of judgement of the
appellant accused.

Facts:
Rosie Quidet, Feliciano Taban Jr., and Aurelio Tubo were charged with homicide for the death of
Jimmy Tagarda and frustrated homicide with Andrew Tagarda as the victim. Upon arraignment, all the
accused entered a plea of not guilty on the frustrated homicide case but Taban entered a voluntary plea
of guilt while petitioner and Tubo maintained their innocence in the homicide case. All were sentenced
with reclusion temporal as to the homicide case and prision mayor as to the frustrated homicide. It was
proven that there was conspiracy on part of the three on planning the executing the act. The Court of
appeals ruled in favor of the RTC.
Issue:
Whether or not Taban and Tubo are included in the relief provided in the appeal.
Ruling:
Yes, Although Taban and Tubo did not appeal their conviction, this part of the appellate courts
judgment is favorable to them, thus, they are entitled to a reduction of their prison terms. In People vs.
Pacaa, the rule is that an appeal taken by one or more of several accused shall not affect those who did
not appeal except insofar as the judgment of the appellate court is favorable and applicable to the latter.

Case 89- Cajurao


BALABA VS. PEOPLE
593 SCRA 210

Doctrine: Appeals Rule 122- 125

Facts:

Office of the Special Prosecutor charged Balaba with the crime of Malversation of Public Funds,
to which the Regional Trial Court convicted him. Balaba filed his Notice of Appeal, where he indicated
that he would file his appeal before the Court of Appeals and filed his Appellant’s Brief Office of the
Solicitor General, instead of filing an Appellee’s Brief, filed a Manifestation and Motion praying for the
dismissal of the appeal for being improper since the Sandiganbayan has exclusive jurisdiction over the
appeal. CA dismissed the appeal on the ground of lack of jurisdiction and denied ensuing Motion for
Reconsideration. Balaba filed a petition for Certiorari under Rule 45 to the Supreme Court. He claims
that it was due to inadvertence that the notice of appeal was filed before the Court of Appeals instead of
the Sandiganbayan. Balaba adds that his appeal was dismissed on purely technical grounds. Balaba asks
the Court to relax the rules to afford him an opportunity to correct the error and fully ventilate his appeal
on the merits.

Issue/s:

Whether or not Balaba’s appeal should be given due course.

Ruling:

NO. Upon Balaba’s conviction by the trial court, his remedy should have been an appeal to the
Sandiganbayan. Paragraph 3, Section 4(c) of Republic Act No. 8249 (RA 8249), which further defined
the jurisdiction of the Sandiganbayan, reads:

The Sandiganbayan shall exercise exclusive appellate jurisdiction over final judgments,
resolutions or orders of
the regional trial courts whether in the exercise of their own original jurisdiction or of their appellate
jurisdiction as herein provided. (Emphasis ours)

There is nothing in said paragraph which can conceivably justify the filing of Balaba’s appeal
before the Court of Appeals instead of the Sandiganbayan. Clearly, the Court of Appeals is bereft of any
jurisdiction to review the judgment Balaba seeks to appeal. In Melencion v. Sandiganbayan, we ruled:
An error in designating the appellate court is not fatal to the appeal. However, the correction in
designating the proper appellate court should be made within the 15-day period to appeal. Once made
within the said period, the designation of the correct appellate court may be allowed even if the records
of the case are forwarded to the Court of Appeals. Otherwise, the second paragraph of Section 2, Rule
50 of the Rules of court would apply. The second paragraph of Section 2, Rule 50 of the Rules of Court
reads: "An appeal erroneously taken to the Court of Appeals shall not be transferred to the appropriate
court but shall be dismissed outright." (Emphasis ours)

In this case, Balaba sought the correction of the error in filing the appeal only after the expiration
of the period to appeal. The trial court promulgated its Decision on 9 December 2002. Balaba filed his
notice of appeal on 14 January 2003. The Court of Appeals issued the Decision declaring its lack of
jurisdiction on 15 December 2004. Balaba tried to correct the error only on 27 January 2005, clearly
beyond the 15-day period to appeal from the decision of the trial court. Therefore, the Court of Appeals
did not commit any error when it dismissed Balaba’s appeal because of lack of jurisdiction.

Case 90- Sevilla


PEOPLE vs. OLIVO
GR No. 130355 January 18, 2001

Doctrine:

In case the judgment of acquittal, it shall state whether the evidence of the prosecution absolutely failed
to prove the guilt of the accused or merely failed to prove his guilt beyond reasonable doubt.

Facts:

This is an appeal from the decision, dated February 27, 1997, of the Regional Trial Court,
Branch 6, Baguio City finding accused-appellant Jessie Olivo, Jr. guilty of murder and sentencing him to
suffer the penalty of reclusion perpetua and to pay the heirs of the victim Jane Lorielinda Tacyo y
Balongay the sum of P50,000.00 as indemnity, P63,800.00 as actual damages, P200,000.00 as moral
damages, and costs.

That on or about the 13th day of June, 1996 or thereabouts, in the City of Baguio, Philippines,
and within the jurisdiction of this Honorable Court, the above-named accused, with intent to kill and
taking advantage of superior strength, did then and there willfully, unlawfully and feloniously stab
JANE LORIELINDA TACYO y BALONGAY with the use of a screw driver and smash her head with
the use of two big rocks, thereby inflicting upon the latter neurogenic shock due to massive head
crushing injuries and multiple stab wounds in the body, which injuries directly caused her death.
That in the commission of the offense there was present the qualifying circumstances of taking
advantage of superior strength in that the accused intentionally employed excessive force out of
proportion to the means of defense available to the offended party, and scoffing in that the victim was
left to lie completely undressed and naked. Upon arraignment, accused-appellant pleaded not guilty to
the crime charged and so the trial of the case followed.

Issue:
Whether or not the accused Jessie Olivo, Jr. guilty beyond reasonable doubt of MURDER as
defined and penalized under Article 248 of the Revised Penal Code, as amended by Section 6 of
Republic Act 7659?

Ruling:

While no eyewitness testified as to the killing of Lorie, there is sufficient circumstantial evidence
to warrant the conclusion that accused-appellant committed the crime. To begin with, circumstantial
evidence is sufficient to prove the elements of the crime if the following requisites are present: (1) there
is more than one circumstance; (2) the facts from which the inferences are derived are proven; and (3)
the combination of all the circumstances is such as to produce a conviction beyond a reasonable doubt.
It is settled rule that the trial courts assessment of the credibility of witnesses is generally accorded great
respect because it had the opportunity to hear them and to observe their demeanor and manner of
testifying. The exception is when it overlooked or misapplied some facts which could have affected the
result of the case. Indeed, the proximate contact of the trial court with those who take the witness stand
places it in a more competent position to discriminate between a true and false testimony.
Art. 248. Murder. - Any person who, not falling within the provisions of Article 248 shall kill another
shall be guilty of murder and shall be punished by reclusion perpetua to death if committed with any of
the following attendant circumstances:
1. ... taking advantage of superior strength...
2. ... or outraging or scoffing at his person or corpse.
The trial court correctly held that the qualifying circumstance of abuse of superior strength was
present since accused-appellant was taller and stronger than the victim. An attack made by an armed
man upon a woman, who died as result thereof, is murder, because his sex and weapon gave him
superiority of strength.
The prosecution having proven the qualifying circumstance of taking advantage of superior strength, the
trial court correctly ruled that accused-appellant committed murder. Murder is punished by reclusion
perpetua to death. In the absence of any other aggravating or mitigating circumstance, accused-appellant
should be punished with the lesser penalty of reclusion perpetua.

Case 91- Sim


GUASCH vs DELA CRUZ
589 SCRA 297

Doctrine:
No motion for reconsideration is filed within the reglementary period, the decision attains finality and
becomes executory in due course must be strictly enforced as they are considered indispensable
interdictions against needless delays and for orderly discharge of judicial business.

Facts:
Respondent Arnaldo dela Cruz filed a Complaint-Affidavit against petitioner Mercedita T.
Guasch with the City Prosecutor of Manila. On several occasions, petitioner transacted business with
him by exchanging cash for checks of small amount without interest. Guasch went to his de la Cruz
residence requesting him to exchange her check with cash of P3,300,000.00. Initially, he refused.
However, petitioner returned the next day and was able to convince him to give her P3,300,000.00 in
cash in exchange for her Insular Savings Bank Check No. 0032082 dated January 31, 2000 upon her
assurance that she will have the funds and bank deposit to cover the said check by January 2000. On the
date of maturity and upon presentment, however, the check was dishonored for the reason that the
account against which it was drawn was already closed.
An information for estafa was filed against the petitioner. After petitioner entered her plea of
not guilty and after the prosecution rested its case, petitioner filed a Motion with Leave to Admit
Demurrer to Evidence with attached Demurrer to Evidence. The trial court issued an Order dated June
16, 2005 granting the demurrer to evidence and dismissing the case.
Respondent then filed a Manifestation with attached Motion to Amend Order dated June 16,
2005 (Motion to Amend) to include a finding of civil liability of petitioner. Respondents counsel
justified his failure to file the motion within the reglementary period of 15 days because all postal offices
in Metro Manila were allegedly ordered closed in the afternoon due to the rally staged on Ayala Avenue.
Meantime, respondent filed a Petition for Certiorari with the Court of Appeals praying that the trial
courts Order dated June 16, 2005 granting the demurrer to evidence be set aside. The trial court denied
respondents Motion to Amend in its Order, finding that counsel for respondent was inexcusably
negligent; Respondent filed a Motion for Reconsideration but the same was denied by the trial court.
The CA also dismissed the appeal. Thus, this case.

Issue:
Whether the Court of Appeals erred in holding that the trial court committed grave abuse of
discretion when it denied respondents Motion to Amend.
Ruling:
NO, the CA did not erred. As a general rule, the statutory requirement that when no motion for
reconsideration is filed within the reglementary period, the decision attains finality and becomes
executory in due course must be strictly enforced as they are considered indispensable interdictions
against needless delays and for orderly discharge of judicial business. The purposes for such statutory
requirement are twofold: first, to avoid delay in the administration of justice and thus, procedurally, to
make orderly the discharge of judicial business, and, second, to put an end to judicial controversies, at
the risk of occasional errors, which are precisely why courts exist. Controversies cannot drag on
indefinitely. The rights and obligations of every litigant must not hang in suspense for an indefinite
period of time.
However, certain elements are considered for the appeal to be given due course, such as: (1) the
existence of special or compelling circumstances, (2) the merits of the case, (3) a cause not entirely
attributable to the fault or negligence of the party favored by the suspension of the rules, (4) lack of any
showing that the review sought is merely frivolous and dilatory, and (5) the other party will not be
unduly prejudiced thereby.
Several of these elements obtain in the case at bar.First, there is ostensible merit to respondents
cause.Second, respondent concededly has an available remedy even if his Motion to Amend was denied,
which is to institute a separate civil action to recover petitioners civil liability. However, to require him
to pursue this remedy at this stage will only prolong the litigation between the parties which negates the
avowed purpose of the strict enforcement of reglementary periods to appeal, that is, to put an end to
judicial controversies.
It is a cherished rule of procedure that a court should always strive to settle the entire controversy
in a single proceeding leaving no root or branch to bear the seeds of future litigation. Given the
circumstances in this case, we find that the trial court committed grave abuse of discretion when it
denied respondents Motion to Amend.

Case 92- Velasco


PEOPLE VS. TARUC
GR NO. 185202 FEBRUARY 18, 2009

Doctrine:
Once an accused escapes from prison or confinement or jumps bail or flees to a foreign country, he
loses his standing in court and unless he surrenders or submits to the jurisdiction of the court he is
deemed to have waived any right to seek relief from the court

Facts:

Francisco Taruc (“Taruc”) was charged before the RTC of Bataan with the crime of murder in
connection with the death of Emelito Sualog. Taruc pleaded not guilty. After trial on the merits, the RTC
of Bataan found Taruc guilty for murder and sentenced him to suffer the death penalty. The case was
brought to the Court of Appeals for automatic review in accordance with A.M. No. 00-5-03-SC.
Thereafter, the CA was informed that Taruc escaped from prison. Then, the CA affirmed the Decision of
the RTC. Taruc appealed the Decision of the CA to the Supreme Court.

Issue:

Whether or not Taruc has lost his right to appeal his conviction considering that he escaped from
jail and eluded arrest.

Held:

Once an accused escapes from prison or confinement or jumps bail or flees to a foreign country,
he loses his standing in court and unless he surrenders or submits to the jurisdiction of the court he is
deemed to have waived any right to seek relief from the court.

Although Rule 124, Section 8 particularly applies to the Court of Appeals, it has been extended
to the Supreme Court by Rule 125, Section 1 of the Revised Rules of Criminal Procedure.
Notwithstanding, accused-appellant did not preclude the Court of Appeals from exercising its review
jurisdiction, considering that what was involved was capital punishment. Automatic review being
mandatory, it is not only a power of the court but a duty to review all death penalty cases.

Case 93- Botor


PEOPLE vs. TIU
GR No. 142885 October 22, 2003

Doctrine:
The Court cannot give credence to the testimony of a person who has a history of giving false
testimonies.

Facts:

Appellants were charged with violating the Dangerous Drugs Act of 1972, and both were found
guilty of the offense charged. However, the Prosecution and Accused presented conflicting narration of
facts.

[5]
The testimonies of prosecution witnesses PO3 Tupil, P/Insp. Culili, P/Insp. Mana and P/Insp.
[6]
Cario, are as follows: That police officers Tupil, Culili and Mana were on alert duty at the Philippine
National Police Narcotics Group, Special Operations Division (SOD) situated in Camp Crame, Quezon
City when a confidential informant (CI) walked in and reported that he had knowledge of two persons,
going by the names William and Edgar, who were engaged in the selling and/or delivering of large
volumes of shabu in Metro Manila. The CI disclosed that Edgar promised him a commission if he can
find a good buyer for their available stocks of shabu.

After interrogating the CI and evaluating the information given, a buy-bust operation was
planned. The exchange would take place between the next day. A team was immediately formed
comprising of Tupil, to act as the poseur-buyer. Thus the exchange took place, Tiu delivered the Shabu
and thereafter, Tupil identified himself as a police officer and grabbed Tiu. De Paz managed to run away
at first, but was immediately caught by Dimalanta and Culili.

On the other hand, Appellants claim that the buy-bust operation never took place. Instead, they
depicted an elaborate frame-up perpetrated by the police. That what happened was that the common-law
wife of Tiu, was in route to Manila on board Tius Mitsubishi Lancer GRS. Somewhere in Sta. Cruz,
Pampanga, their vehicle was stopped by armed men who introduced themselves as narcotics policemen
stationed in Olongapo City. The policemen accused them of riding in a hot car and told them that they
must be brought to Camp Crame.

Upon arrival thereat, Perez and Cercado were interrogated by said officers regarding the
whereabouts of Tiu. Both replied that Tiu resides in Marikina and that he may already be home. The
police officers then proceeded to Tius house with Perez and Cercado and decided to wait for him.

Tiu arrived the following day, accompanied by De Paz. The police officers immediately drew
their weapons and ordered them to lie on the ground. One police officer told Tiu that their superior,
Director Acop, is very mad at him because of false rumors Tiu was allegedly spreading against Director
Acop in Olongapo City. Tiu, De Paz, Perez and Cercado were thereafter brought to Camp Crame. Tiu
and De Paz were brought before the Marikina Prosecutors Office for inquest proceedings.

Issue: Whether or Not should the testimonies given by the police officers be given credence in deciding
the case

Ruling:
No. The Court understands the importance of buy-bust operations as an effective method,
[15]
sanctioned by law, of apprehending drug peddlers. At the same time, it is aware that it is susceptible to
mistake, harassment, extortion and abuse.

The elements necessary successfully to prosecute an accused for the illegal sale of shabu are: 1)
Identities of the buyer and seller, the object, and the consideration; and 2) the delivery of the thing sold
[19]
and the payment therefor. To prove that these elements were present in the case at bar, the prosecution
relies on the testimony of the poseur-buyer, Tupil. The four other witnesses who testified for the
prosecution were incompetent to testify on these facts. The prosecutions case crucially depends on the
strength of the testimony of Tupil.

While the Court concedes that a single, trustworthy and credible witness could be sufficient to
[20]
convict an accused, it does not hold true in this case. Tupils credibility has been seriously eroded by
the fact that, in another drug-related criminal case, he was found to have framed-up the accused therein
for illegal sale of shabu. It having been indubitably shown that Tupil has had a history of framing-up
suspects, not to mention his having given false testimony, the Court cannot give credence to his
testimony in this case.

While the defense of frame-up can be easily fabricated, this claim assumes importance when it is
shown that the apprehending officers, who were also the prosecutions witnesses, have a penchant for
[23]
framing innocent people or, in our colorful vernacular, committing HULIDAP. Under the
circumstances, a cloud of doubt, arising from the lack of credibility of the prosecution witnesses, attends
the finding of guilt against appellants.

Moreover, to corroborate their claim that no buy-bust operation occurred, appellants were able to
present disinterested witnesses who had no personal or business relationship with them: Tessie
Lesiguiez is a balut and cigarette vendor who sells her wares just beside the 7-11 convenient store,
where the alleged buy-bust operation occurred. She testified that she did not witness any buy-bust
operation, Carmelita Villanueva is another vendor who also testified that she does not recall any
unusual incident that happened at that time, and Erna Boadilla is a next-door neighbor of Tiu. She said
that on the date, she saw 5 or 6 armed men accosting Tiu who was, thereafter, boarded inside a car.

In criminal cases, it is incumbent upon the prosecution to establish the guilt of the accused
[24]
beyond reasonable doubt. The Court finds that the guilt of appellants herein has not been proven
beyond reasonable doubt, as measured by the required moral certainty for conviction. While there has
been no strong proof that appellants were indeed framed-up, in criminal cases the overriding
consideration is not whether the Court doubts the innocence of the accused but whether it entertains a
[25]
reasonable doubt as to his guilt. Considering the shady credibility of the prosecution witnesses, and
the evidence presented by appellants, the constitutional presumption of innocence has not been
overcome.
Case 94- Bugtas
COLINARES vs. PEOPLE
662 SCRA 266
Doctrine:
The Probation Law, said the Court in Francisco, requires that an accused must not have appealed his
conviction before he can avail himself of probation. This requirement “outlaws the element of
speculation on the part of the accused—to wager on the result of his appeal—that when his conviction is
finally affirmed on appeal, the moment of truth well-nigh at hand, and the service of his sentence
inevitable, he now applies for probation as an ‘escape hatch’ thus rendering nugatory the appellate
court’s affirmance of his conviction.” In a real sense, the Court’s finding that Arnel was guilty, not of
frustrated homicide, but only of attempted homicide, is an original conviction that for the first time
imposes on him a probationable penalty. Had the RTC done him right from the start, it would have found
him guilty of the correct offense and imposed on him the right penalty of two years and four months
maximum. This would have afforded Arnel the right to apply for probation.
Facts:
Arnel Colinares was charged and found guilty beyond reasonable doubt of frustrated homicide
by the RTC of Camarines Sur. He was sentenced to suffer imprisonment from two years and four months
of prison correccional, as minimum, to six years and one day of prison mayor, as maximum. Since the
maximum probationable imprisonment under the law was only up to six years, Arnel did not qualify for
probation. On appeal by Colinares, the Court of Appeals sustained the RTC’s decision. Unsatisfied with
the Court of Appeal’s decision, petitioner then appealed to the Supreme Court and took the position that
he should be entitled to apply for probation in case the Court metes out a new penalty on him that makes
his offense probationable, which was strongly opposed by the Solicitor General reiterating that under the
Probation Law, no application for probation can be entertained once the accused has perfected his appeal
from the judgment of conviction. The Supreme Court, however, found that Colinares is guilty of
attempted homicide and not of frustrated homicide.
Issue:
Whether or not Arnel Colinares may still apply for probation on remand of the case to the trial court
Ruling:
Yes, The Supreme Court ruled that Colinares may apply for probation upon remand of his case to
the RTC. Ordinarily, an accused would no longer be entitled to apply for probation, he having appealed
from the judgment of the RTC convicting him for frustrated homicide. But in this case the Supreme
Court ruled to set aside the judgment of the RTC and found him only liable for attempted homicide, if
the Supreme Court follows the established rule that no accused can apply for probation on appeal, the
accused would suffer from the erroneous judgment of the RTC with no fault of his own, therefore
defying fairness and equity.
Case 95- Cabalonga
VILLAREAL vs PEOPLE
G.R. No. 151258. December 1, 2014

Doctrine:

An acquittal rendered by a court of competent jurisdiction after trial on the merits is immediately final
and and cannot be appealed on the ground of double jeopardy. Despite the rule, the court condes that
the rule on double jeopardy is not absolute. This rule is inapplicable to cases in which the state assails
the very jurisdiction of the court that issued the criminal judgment

Facts:

Seven Freshmen Law students of Ateneo de Manila University School of Law have been
initiated by the Aquila Legis Juris Fraternity on February 1991. The initiation rites started when the
neophytes were met by some members of the mentioned fraternity at the lobby of the Ateneo Law
School. They were consequently brought to a house and briefed on what will be happening during the
days when they will be initiated. They were informed that there will be physical beatings and that the
neophytes can quit anytime they want. They were brought to another house to commence their initiation.

The neophytes were insulted and threatened even before they got off the van. Members of the
fraternity delivered blows to the neophytes as they alighted from the van. Several initiation rites were
experienced by the neophytes like the Indian run, Bicol express and rounds. They were asked to recite
provisions and principles of the fraternity and were hit everytime they made a mistake.

Accused fraternity members, Dizon and Villareal, asked the head of the initiation rites
(Victorino) to reopen the initiation. Fraternity members subjected neophytes to paddling and additional
hours of physical pain. After the last session of beatings, Lenny Villa could not walk. Later that night, he
was feeling cold and his condition worsened. He was brought to the hospital but was declared dead on
arrival.

Criminal case was filed against 26 fraternity members and was subsequently found guilty beyond
reasonable doubt of the crime of homicide and penalized with reclusion perpetua.
On January 10 2002, CA modified the criminal liability of each of the accused according to
individual participation. 19 of the the accused were acquitted, 4 of the appellants were found guilty of
slight physical injuries, and 2 of the accused-appellants (Dizon and Villareal) were found guilty beyond
reasonable doubt of the crime of homicide.

Accused Villareal petitioned for review on Certriori under Rue 45 on the grounds that the CA
made 2 reversible errors: first, denial of due process and second, conviction absent proof beyond
reasonable doubt. Consequently, petitioner Villareal died on 13 March 2011 and filed a Notice of Death
of Party on 10 August 2011.

Issue:

Whether the CA committed grave abuse of discretion, amounting to lack or excess of


jurisdiction, when it set aside the finding of conspiracy by the trial court and adjudicated the liability of
each accused according to individual participation
Ruling:

NO. Grave abuse of discretion cannot be attributed to a court simply because it allegedly
misappreciated the facts and the evidence. Mere errors of judgment are correctible by an appeal or a
petition for review under Rule 45 of the Rules of Court, and not by an application for a writ of certiorari.
Pursuant to the rule on double jeopardy, the Court is constrained to deny the Petition contra Victorino et
al. – the 19 acquitted fraternity members.

A verdict of acquittal is immediately final and a re-examination of the merits of such acquittal,
even in the appellate courts, will put the accused in jeopardy for the same offense.

Case 96- Casiano


DANDY L. DUNGO and GREGORIO A. SIBAL, JR vs. PEOPLE OF THE PHILIPPINES
G.R. No. 209464. July 1, 2015

Doctrine:
Sec 13 (c) Rule 124. Appeal is a proceeding undertaken to have a decision reconsidered by bringing it to
a higher court authority. The right to appeal is neither a natural right nor is it a component of due
process. It is a mere statutory privilege and may be exercised only in the manner and in accordance with
the provisions of law.
In cases where the Court of Appeals imposes reclusion Perpetua, life imprisonment or a lesser penalty, it
shall render and enter judgment imposing such penalty. The judgment may be appealed to the Supreme
Court by notice of appeal filed with the court of Appeals.

Facts:
Marlon Villanueva is a UP Los Banos student and a neophyte in Alpha Phi Omega fraternity as
condition for his admission to fraternity he must undergo final rite of initiation. He was subjected to
physical harm which resulted to his death. Based on the multiple injuries and contusion all over his
body it was opined by Dr Camarillo who conducted an autopsy that Villanueva is a hazing victim.
Prosecutor presented 20 witnesses to account what happened on the early morning of January 14,

2006 at Villa Novaliches resort in Pansol Laguna. All the witnesses testimony were given great weight
particularly the testimony of Susan Ignacio, Sari-sari store owner in front of the resort and Donato
Magat tricycle driver plying the route of Pansol Calamba.
Ignacio testified that on January 13, 2006 about 20 persons arrived on the resort on board a
jeepney including the victim. Ignacio identified Dandy Dungo as the guy seated beside the driver. Later,
that evening 15 men gathered on the terrace looked like they were praying and then lights went off.
Magat testified that around 3 o' clock AM while he was waiting for passenger at corner of the resort.
Two men in their 20's carrying body of a man approached him and he was instructed to drive at the
nearest hospital. Magat sensed that the man being carried is cold and asked his companion what
happened to which they replied, he had too much drink. On the emergency room the victim was
motionless, not breathing and had no heartbeat. Dr Masilungan tried to revive Villanueva for about 30
minutes however, did not respond to resuscitation. He was pronounce dead. An autopsy of Dr Camarillo
from PNP of Calamba confirmed that his injuries where hazing-related. During the autopsy 2
matchsticks were retrieved from the cadaver with markings Alpha Phi Omega (APO) Fraternity.

Dandy Dungo and Gregorio Sibal Jr were both alumni fraternity members of APO-Theta
Chapter. Sibal and Dungo on their defense has different alibis but RTC- Calamba found Dungo and Sibal
guilty of the crime of violating Sec 4 of the Anti- Hazing Law and was sentence to suffer reclusion
perpetua. The evidence of prosecution evidence undeniably proved that Villanueva's injuries which
resulted to his death from of the initiation rites done in darkness and secrecy.
CA affirmed that decision of RTC in toto. Dungo and Sibal filed notice of appeal that the
prosecution failed to establish conspiracy act or circumstance that will prove joint purpose and design
among the accused.
Issue:
​Whether or Not the accused can raise issue of conspiracy on appeal under petition for certiorari
under Rule 45
Ruling:
No. In this case, petitioners opted to appeal the CA decision via a petition for certiorari under
Rule 45. Consequently, they could only raise questions of law. but the petitioners began to assail the
existence of conspiracy in their reply, which is a question of fact that would require an examination of
the evidence presented. In the interest of justice, however, and due to the novelty of the issue presented,
the Court deems it proper to open the whole case for review.
An appeal is a proceeding undertaken to have a decision reconsidered by bringing it to a higher
court authority. The right to appeal is neither a natural right nor is it a component of due process. It is a
mere statutory privilege, and may be exercised only in the manner and in accordance with the provisions
of law.
When the CA imposed a penalty of reclusion perpetua or life imprisonment, an accused may: (1)
file a notice of appeal under Section 13 (c), Rule 124 to avail of an appeal as a matter of right before the
Court and open the entire case for review on any question; or (2) file a petition for review on certiorari
under Rule 45 to resort to an appeal as a matter of discretion and raise only questions of law.

Case 97- Cuevas


MANANSALA vs PEOPLE

777 SCRA 563


Doctrine:

At the outset, it must be stressed that in criminal cases, an appeal throws the entire case wide open for
review and the reviewing tribunal can correct errors, though unassigned in the appealed judgment, or
even reverse the trial court's decision based on grounds other than those that the parties raised as
errors. The appeal confers the appellate court full jurisdiction over the case and renders such court
competent to examine records, revise the judgment appealed from, increase the penalty, and cite the
proper provision of the penal law.

Facts:
On May 31, 1999, private complainant Kathleen L. Siy (Siy), instructed her secretary, Marissa
Bautista (Bautista), to withdraw via Automated Teller Machine (ATM) the amount of P38,000.00 from
her Metrobank and Bank of the Philippine Islands bank accounts. However, Bautista was not able to
make such withdrawal as the ATM was offline so she took it upon herself to get such amount from the
petty cash custodian of UMC instead, but she forgot to inform Siy where she got the money. On June 9,
1999, UMC Finance Manager Violeta Q. Dizon-Lacanilao (Lacanilao) informed Siy that as per the Petty
Cash Replenishment Report (subject report) of the same date prepared by UMC Petty Cash Custodian
Manansala, she allegedly made a cash advance in the amount of P38,000.00 which remained
unliquidated. It was only then that Siy found out what Bautista had done, and she immediately rectified
the situation by issuing two (2) checks to reimburse UMC's petty cash account. As the checks were
eventually encashed resulting in the replenishment of UMC's petty cash account, Lacanilao instructed
Manansala to revise the subject report by deleting the entry relating to Siy's alleged cash advance, to
which Manansala acceded.
Sometime in March 2000, Lacanilao instructed Manansala to retrieve the subject report, re-insert the
entry relating to Siy's alleged cash advance therein, reprint the same on a scratch paper, and repeatedly
fold the paper to make it look old. On the basis of the reprinted subject document, Siy was
administratively charged for using office funds for personal use. On April 18, 2000, Siy was terminated
from her job and Lacanilao succeeded the former in the position she left vacant. The foregoing prompted
Siy to pursue criminal charges against Marty, Lacanilao, and Manansala for Falsification of Private
Documents. Eventually, the charge against Marty was withdrawn, and an Amended Information dated
July 19, 2001 for the aforesaid crime was filed against Lacanilao and Manansala before the Metropolitan
Trial Court of Makati City, Branch 65 (MeTC). In her defense, Manansala maintained that she was just
following Lacanilao's orders as the latter is her superior who approves her work. She added that when
Lacanilao instructed her to reprint the subject report, she was apprehensive to follow because she
suspected something, but nevertheless acquiesced to such instruction.

In a Decision dated October 27, 2010, the MeTC both found Lacanilao and Manansala guilty
beyond reasonable doubt of committing the crime of Falsification of Private Documents. Aggrieved,
Manansala appealed her conviction to the Regional Trial Court of Makati. the RTC affirmed the MeTC
ruling in toto. Manansala moved for reconsideration but was denied in an Order dated January 30, 2012.
Undaunted, she elevated the matter to the CA via a petition for review. The CA affirmed the RTC ruling.

Issue:
Whether or not the CA correctly affirmed Manansala's conviction for Falsification of Private
Documents.

Ruling:
The petition is without merit.
At the outset, it must be stressed that in criminal cases, an appeal throws the entire case wide
open for review and the reviewing tribunal can correct errors, though unassigned in the appealed
judgment, or even reverse the trial court's decision based on grounds other than those that the parties
raised as errors. The appeal confers the appellate court full jurisdiction over the case and renders such
court competent to examine records, revise the judgment appealed from, increase the penalty, and cite
the proper provision of the penal law.

Proceeding from the foregoing, the Court agrees with the ruling of the courts a quo convicting
Manansala of the crime of Falsification of Private Documents, but disagrees in the appreciation of the
"mitigating circumstance" of acting under an impulse of uncontrollable fear in her favor.

In the instant case, while the records show that Manansala was apprehensive in committing a falsity in
31
the preparation of the subject report as she did not know the repercussions of her actions, nothing
would show that Lacanilao, or any of her superiors at UMC for that matter, threatened her with loss of
employment should she fail to do so. As there was an absence of any real and imminent threat,
intimidation, or coercion that would have compelled Manansala to do what she did, such a circumstance
cannot be appreciated in her favor. Wherefore, the petition is DENIED.

Case 98- Custodio


ABRAHAM MICLAT, JR. y CERBO vs. PEOPLE OF THE PHILIPPINES
G.R. No. 176077 August 31, 2011

Doctrine:

A peace office of a private person may, among others, without a warrant, arrest a person:
(a) When, in his presence, the person to be arrested has committed, is actually committing, or is
attempting to commit an offense (Sec. 5 a)

In this case, two elements must be present:


(1) the person to be arrested must execute an overt act indicating that he has just committed, is actually
committing, or is attempting to commit a crime; and
(2) such overt act is done in the presence or within the view of the arresting officer.

The right against warrantless searches and seizure, however, is subject to legal and judicial exceptions,
namely:

1. Warrantless search incidental to a lawful arrest;


2. Search of evidence in "plain view";
3. Search of a moving vehicle;
4. Consented warrantless search;
5. Customs search;
6. Stop and Frisk; and
7. Exigent and emergency circumstances.

What constitutes a reasonable or unreasonable warrantless search or seizure is purely a judicial


question, determinable from the uniqueness of the circumstances involved, including the purpose of the
search or seizure, the presence or absence of probable cause, the manner in which the search and
seizure was made, the place or thing searched, and the character of the articles procured.

Facts:

On November 8, 2002, P/Insp. Jose Valencia of the Caloocan City Police Station-SDEU called
upon his subordinates after the receiving an INFOREP Memo from Camp Crame relative to the illicit
and down-right drug-trading activities being undertaken along Palmera Spring II, Bagumbong, Caloocan
City involving among others, one Abraham Miclat.
Immediately, P/Insp. Valencia formed a surveillance team together with operatives from the Drug
Enforcement Unit. The team then proceeded to the target area to verify the said informant and/or
memorandum.
Thru a small opening in the curtain-covered window, PO3 Antonio peeped inside and there at a
distance of 1½ meters, he saw "Abe" arranging several pieces of small plastic sachets which he believed
to be containing shabu. Slowly, said operative inched his way in by gently pushing the door as well as
the plywood covering the same.

Upon gaining entrance, PO3 Antonio forthwith introduced himself as a police officer while
"Abe," on the other hand, after being informed of such authority, voluntarily handed over to the former
the four (4) pieces of small plastic sachets the latter was earlier sorting out.
PO3 Antonio immediately placed the suspect under arrest and brought him and the four (4) pieces of
plastic sachets containing white crystalline substance to their headquarters and turned them over to PO3
Fernando Moran for proper disposition.
Issue:
Whether or Not the arrest and seizure conducted against Abraham Miclat is valid

Ruling:

In the instant case, Miclat was caught in flagrante delicto and the police authorities effectively
made a valid warrantless arrest.

The established facts reveal that on the date of the arrest, agents of the Station Drug Enforcement
Unit (SDEU) of the Caloocan City Police Station were conducting a surveillance operation in the area of
Palmera Spring II to verify the reported drug-related activities of several individuals. During the
operation, PO3 Antonio, through petitioner’s window, saw petitioner arranging several plastic sachets
containing what appears to be shabu in the living room of their home. The plastic sachets and its
suspicious contents were plainly exposed to the view of PO3 Antonio, who was only about one and one-
half meters from where petitioner was seated. PO3 Antonio then inched his way in the house by gently
pushing the door.

Upon gaining entrance, the operative introduced himself as a police officer. After which,
petitioner voluntarily handed over to PO3 Antonio the small plastic sachets. PO3 Antonio then placed
petitioner under arrest and informed him of his constitutional rights.PO3 Antonio then took the
petitioner and the four (4) pieces of plastic sachets to their headquarters and turned them. Considering
the circumstances immediately prior to and surrounding the arrest of the petitioner, petitioner was
clearly arrested in flagrante delicto as he was then committing a crime, violation of the Dangerous Drugs
Act, within the view of the arresting officer. As to the admissibility of the seized drugs in evidence, it
too falls within the established exceptions.
Case 99- De Luna
PEOPLE OF THE PHILIPPINES vs. BELEN MARIACOS
621 SCRA 327
Doctrine:

Requisites for a valid warrantless arrest, search and seizure


Facts:
Belen Mariacos was charged with violation of the Comprehensive Dangerous Drugs Act of 2002.
She was convicted to the same with a penalty of life imprisonment. On appeal at the CA, she averred
that there was no probable cause for her arrest and that her against an unreasonable search was flagrantly
violated by Police Officer when the latter searched the bag, assuming it was hers, without a search
warrant and with no permission from her.
Further, appellant claimed that the prosecution failed to prove the corpus delicti of the crime. She
alleged that the apprehending police officers violated Dangerous Drugs Board Regulation which
prescribes the procedure in the custody of seized prohibited and regulated drugs, instruments,
apparatuses, and articles.

​The OSG, however, argued that appellant was now estopped from questioning the illegality of
her arrest since she voluntarily entered a plea of "not guilty" upon arraignment and participated in the
trial and presented her evidence. The OSG brushed aside appellant’s argument that the bricks of
marijuana were not photographed and inventoried in her presence or that of her counsel immediately
after confiscation, positing that physical inventory may be done at the nearest police station or at the
nearest office of the apprehending team, whichever was practicable.
The CA ruled that appellant was caught in flagrante delicto of "carrying and conveying" the bag
that contained the illegal drugs, and thus held that appellant’s warrantless arrest was valid.
Issue:
Whether or Not the accused was illegally search and arrested.
Ruling:
The search in this case was valid. The vehicle that carried the contraband or prohibited drugs was
about to leave. PO2 Pallayoc had to make a quick decision and act fast. It would be unreasonable to
require him to procure a warrant before conducting the search under the circumstances. Time was of the
essence in this case. The searching officer had no time to obtain a warrant. Indeed, he only had enough
time to board the vehicle before the same left for its destination. The police officer has probable cause to
conduct a search of the packages in this case because of a previous tip from a secret agent of the police.
For this rule to apply, it is imperative that there be a prior valid arrest. Although, generally, a
warrant is necessary for a valid arrest, the Rules of Court provides the exceptions. It is when, in his
presence, the person to be arrested has committed, is actually committing, or is attempting to commit an
offense. Be that as it may, we have held that a search substantially contemporaneous with an arrest can
precede the arrest if the police has probable cause to make the arrest at the outset of the search. Given
that the search was valid, appellant’s arrest based on that search is also valid.
Contrary to appellant’s claim, the prosecution’s evidence establishes the chain of custody from
the time of appellant’s arrest until the prohibited drugs were tested at the police crime laboratory.

Case 100 – Dionisio


PEOPLE vs. TUAN
G.R. NO. 176066, 11 AUGUST 2010

Doctrine:
Search Warrant ---A description of the place to be searched is sufficient if the officer serving the warrant
can, with reasonable effort, ascertain and identify the place intended and distinguish it from other
places in the community. A designation or description that points out the place to be searched to the
exclusion of all others, and on inquiry unerringly leads the peace officers to it, satisfies the
constitutional requirement of definiteness. In the case at bar, the address and description of the place to
be searched in the Search Warrant was specific enough. There was only one house located at the stated
address, which was accused-appellant’s residence, consisting of a structure with two floors and
composed of several rooms.

Facts:
On January 2000, two informants namely, Tudlong and Lad-ing arrived at the office of CIDG
(Criminal Investigation and Detention Group) in Baguio City, and reported to SPO2 Fernandez, Chief of
the Station Drug Enforcement Unit (SDEU), that a certain "Estela Tuan" had been selling marijuana at
Barangay Gabriela Silang, Baguio City. SPO2 Fernandez set out to verify the report of Tudlong and
Lad-ing. On the afternoon of the same day, he gave Tudlong and Lad-ing P300.00 to buy marijuana, and
accompanied the two informants to the accused Tuan’s house. Tudlong and Lad-ing entered the house,
while SPO2 Fernandez waited at the adjacent house. Later, Tudlong and Lad-ing came out and showed
SPO2 Fernandez the marijuana they bought. Upon returning to the CIDG office, SPO2 Fernandez
requested a laboratory examination on the specimen and yielded positive results for marijuana. SPO2
Fernandez, together with the informants, filed the Application for a Search Warrant before Judge
Iluminada Cabato-Cortes (Judge Cortes) of the Municipal Trial Court in Cities (MTCC), Baguio City on
January 25, 2000. Two hours later, at around three o’clock, Judge Cortes personally examined SPO2
Fernandez, Tudlong, and Lad-ing, after which, she issued a Search Warrant, which stated Tuan’s
residence as “the house of the accused Estela Tuan at Brgy. Gabriela Silang, Baguio City” Even though
accused Tuan was not around, the CIDG team was allowed entry into the house by Magno Baludda
(Magno), accused’s father, after he was shown a copy of the Search Warrant. SPO2 Fernandez guarded
the surroundings of the house, while SPO1 Carrera and PO2 Chavez searched inside. They saw, in the
presence of Magno, a movable cabinet in Tuan’s room, below of which they found a brick of marijuana
and a firearm. Later Tuan arrived and thereafter, the police officers asked Tuan to open a cabinet, in
which they saw more bricks of marijuana.
The defense, on the other hand, disclaimed ownership of the bricks and alleged that a Search
Warrant was issued for her house because of a quarrel with her neighbor named Lourdes Estillore
(Estillore). The RTC found accused guilty as charged.
On appeal, the CA modified by acquitting Tuan of the charge for illegal possession of firearm but
affirming her conviction for illegal possession of marijuana. Tuan raised the matter to the Supreme
Court contending, among others, that the warrant failed to particularly describe the place because the
house was a two-storey building composed of several rooms.
Issue:
Whether the search warrant particularly described the place to be searched.
Ruling:
Yes. A description of the place to be searched is sufficient if the officer serving the warrant can,
with reasonable effort, ascertain and identify the place intended and distinguish it from other places in
the community. A designation or description that points out the place to be searched to the exclusion of
all others, and on inquiry unerringly leads the peace officers to it, satisfies the constitutional requirement
of definiteness. In the case at bar, the address and description of the place to be searched in the Search
Warrant was specific enough. There was only one house located at the stated address, which was
accused-appellant’s residence, consisting of a structure with two floors and composed of several rooms.
Case 101 – Dolor
ESQUILLO vs PEOPLE
629 SCRA 370

Doctrine:
Warrantless searches are valid in these situations:
a) consented searches,
b) searches incident to a lawful arrest,
c) searches of vessels and aircraft for violation of immigration, custom and drug laws,
d) searches of moving vehicles,
e) searches of automobiles at borders,
f) the prohibited articles are in plain view,
g) searches of buildings to enforce fire, sanitary and other regulations,
h) stop and frisk situations

Facts:
​Susan Esquillo was convicted of the violation of the Dangerous Drugs Acts. Police officers were
assigned in Malibay, Pasay to conduct surveillance on a notorious snatcher named Ryan. It was during
that time the police officers noticed the petitioner. Two police officers came to Esquillo and another
person while they were transacting. They saw the latter place inside a yellow cigarette case a plastic
sachet containing a white substance. They approached the petitioner and introduced themselves as police
officer. Subsequently, they inquired regarding the sachet the petitioner placed inside the case. The
petitioner acted suspiciously and even tried to flee. The police officers prevented her from doing so.
They apprised the petitioner of her constitutional rights and then they confiscated the sachet. They
marked the sachet with initials SRE and took the petitioner to the police station.
The petitioner contends against the police officer’s statement. The petitioner said that she was
resting at home when policemen barged inside and asked her whether or not she knew a certain Ryan.
She replied in the negative. Afterwards, she was forcibly taken to the police station and was detained
there. During her detention, the police officers were claiming that there was shabu inside the wallet they
seized from her Esquillo argues that the arrest was invalid and that the officers planted evidence against
her.
The lower cause said that the officers had probable cause to search Esquillo under the stop-and-
frisk doctrine.
Issue:
Whether or Not the warrantless arrest was valid

Ruling:
The issue whether the arrest was valid was waived by the petitioner when she did not quash it
before arraignment. The issue was only raised the first time during appeal on the appellate court.
The circumstances before the eventual arrest gave the police officers a reasonable belief that a search on
her was warranted. The police officer saw IN PLAIN VIEW that the petitioner was placing a plastic
sachet containing a white substance inside her cigarette case. Given the training of police officers, they
would likely be drawn to curiosity and approach her to inquire regarding such matter. The petitioner’s
reaction of attempting to flee after the police officer introduced his self gave more reason for the officer
to check the petitioner.
Warrantless searches are valid in these situations:
a) consented searches,
b) searches incident to a lawful arrest,
c) searches of vessels and aircraft for violation of immigration, custom and drug laws,
d) searches of moving vehicles,
e) searches of automobiles at borders,
f) the prohibited articles are in plain view,
g.) searches of buildings to enforce fire, sanitary and other regulations, h.) stop and frisk situations

On regards her arrest, when the officer saw the white substance from a distance, the plain view doctrine
was imposed. When searched the officers followed the definition and requirements of a valid stop-and-
frisk as stated in People v. Chua - that he should properly introduce himself and make initial inquiries,
approach and restrain a person who manifests unusual and suspicious conduct, in order to check the
latter’s outer clothing for possibly concealed weapons.

Case 102 – Evangelista

SPOUSES JOEL AND MARIETTA MARIMLA vs. PEOPLE OF THE PHILIPPINES


G.R. NO. 158467 OCTOBER 16, 2009

Doctrine:

The general rule is that a party is mandated to follow the hierarchy of courts. However, in exceptional
cases, the Court, for compelling reasons or if warranted by the nature of the issues raised, may take
cognizance of petitions filed directly before it. In this case, the Court opts to take cognizance of the
petition, as it involves the application of the rules promulgated by this Court in the exercise of its rule-
making power under the Constitution

Facts:
​A Special Investigator (SI) Ray C. Lagasca of the NBI Anti-Organized Crime Division filed two
(2) applications for search warrant with the Regional Trial Court (RTC) of Manila seeking permission to
search the petitioner’s house located on RD Reyes St., Brgy. Sta. Trinidad, Angeles City, for Violation of
Section 16, Article III of Republic Act (R.A.) No. 6425, as amended. The applications alleged that SI
Lagasca’s request for the issuance of the search warrants was founded on his personal knowledge as well
as a witness named Roland D. Fernandez (Fernandez), obtained after a series of surveillance operations
and a test buy made at the petitioner’s house. The purpose of application for search warrants was to seize
an undetermined amount of Methamphetamine Hydrochloride, popularly known as SHABU,
MARIJUANA and other drug paraphernalia.

​Executive Judge Mario Guaria III (Judge Guaria III) issued a warrant after personally examining
the case presented before him.

​The members of the NBI Anti-Organized Crime Division, in coordination with the Philippine
National Police of Angeles City, searched house and they were able to seize cash and other drug
paraphernalia.
​An Information was filed against petitioners for violation of Section 8, Article II of R.A. No.
6425, as amended by R.A. No. 7659.
​Petitioners filed a Motion to Quash Search Warrant and to Suppress Evidence Illegally Seized on
the ground that it was void ab initio for having it issued outside the jurisdiction and judicial region of the
court and that the evidence illegally seized by virtue of the questioned search warrant is therefore
inadmissible in evidence.
​The Office of the City Prosecutor claims that the questioned search warrant does not fall within
the coverage of Sec. 2 of Rule 126 of the Revised Rules on Criminal Procedure, but under A.M. No. 99-
10-09-SC, which authorizes the Executive Judges and Vice Executive Judges of the RTCs of Manila and
Quezon City to act on all applications for search warrants involving dangerous drugs, among others,
filed by the NBI, and provides that said warrants may be served in places outside the territorial
jurisdiction of the RTCs of Manila and Quezon City.

​Judge Omar T. Viola denied petitioners Motion to Quash Search Warrant and to Suppress
Evidence Illegally Seized for lack of merit. Petitioners filed a Motion for Reconsideration but it was
denied by the court.

Issue:
​Whether or not the respondent court acted with grave abuse of discretion amounting to lack or in
excess of jurisdiction in issuing the assailed Orders, denying petitioners Motion to Quash Search
Warrant and to Suppress Evidence Illegally Seized and their Motion for Reconsideration

Ruling:

The general rule is that a party is mandated to follow the hierarchy of courts. However, in
exceptional cases, the Court, for compelling reasons or if warranted by the nature of the issues raised,
may take cognizance of petitions filed directly before it. In this case, the Court opts to take cognizance
of the petition, as it involves the application of the rules promulgated by this Court in the exercise of its
rule-making power under the Constitution

​It may be seen that A.M. No. 99-10-09-SC authorizes the Executive Judge and Vice Executive
Judges of the RTCs of Manila and Quezon City to act on all applications for search warrants involving
heinous crimes, illegal gambling, dangerous drugs and illegal possession of firearms on application filed
by the PNP, NBI, PAOC-TF, and REACT-TF. On the other hand, Rule 126 of the Revised Rules on
Criminal Procedure provides that the application for search warrant shall be filed with: (a) any court
within whose territorial jurisdiction a crime was committed, and (b) for compelling reasons, any court
within the judicial region where the crime was committed if the place of the commission of the crime is
known, or any court within the judicial region where the warrant shall be enforced.

​Nothing in A.M. No. 99-10-09-SC prohibits the heads of the PNP, NBI, PAOC-TF and REACT-
TF from delegating their ministerial duty of endorsing the application for search warrant to their
assistant heads. Under Section 31, Chapter 6, Book IV of the Administrative Code of 1987, an assistant
head or other subordinate in every bureau may perform such duties as may be specified by their superior
or head, as long as it is not inconsistent with law

​We cannot find any irregularity or abuse of discretion on the part of Judge Omar T. Viola for
denying petitioners Motion to Quash Search Warrant and to Suppress Evidence Illegally Seized. On the
contrary, Judge Guaria III had complied with the procedural and substantive requirements for issuing the
questioned search warrant.

Case 103 – Gabi

PEOPLE OF THE PHILIPPINES vs. JERRY PUNZALAN AND PATRICIA PUNZALAN


G.R. No. 199087 November 11, 2015

Doctrine:
Section 12, Chapter V of A.M. No. 03-8-02-SC authorizes the Executive Judges and Vice Executive
Judges of the Regional Trial Court (RTC) of Manila and Quezon City to issue warrants to be served in
places outside their territorial jurisdiction in special criminal cases such as those involving heinous
crimes, illegal gambling, illegal possession of firearms and ammunitions as well as violations of the
Comprehensive Dangerous Drugs Act of 2002.
In the issuance of a search warrant, probable cause requires such facts and circumstances that would
lead a reasonably prudent man to believe that an offense has been committed and the objects sought in
connection with the offense are in the place to be searched.

Facts:
Intelligence Agent 1 Sandaan (IA1 Sandaan) and her team implemented a search warrant issued
by then Manila RTC Judge Peralta, Jr. to (i) make an immediate search of the premises of Jerry and

Patricia Punzalan, et al. who are all residents of 704 Apelo Cruz Compound, Barangay 175, Malibay,
Pasay City; and (ii) to seize and take possession of an undetermined quantity of assorted dangerous
drugs, including the proceeds or fruits and bring said property to the court.
Since there are three houses or structures inside the compound believed to be occupied by
Punzalan, a sketch of the compound describing the house to be searched was prepared and attached to
the search warrant.
The Philippine Drug Enforcement Agency (PDEA) Team tasked to conduct the search, before
proceeding to the target area, passed by the barangay hall to coordinate with Barangay Chairman and
Kagawads. The team likewise brought with them a media representative affiliated with "Sunshine
Radio" to cover the operation.
When they were already outside the house of the Punzalans, IA1 Sandaan knocked on the door.
A woman, later identified as Patricia Punzalan, slightly opened the door. When they introduced
themselves as PDEA agents and informed the occupant that they have a search warrant, Patricia
immediately tried to close the door but was not successful since the PDEA agents pushed the door open.
Inside the house, the team immediately saw plastic sachets placed on top of the table. I01
Pagaragan was able to seize nine (9) heat-sealed plastic sachets, two (2) square-shaped transparent
plastic containers and a small round plastic container. All three (3) plastic containers contained smaller
heat-sealed plastic sachets of white crystalline substance of suspected shabu. There were also other
paraphernalia, guns, money and a digital weighing scale. Accordingly, SI2 Esteban and IO2 Alvarado
effected the arrest of accused-appellants Jerry and Patricia Punzalan after informing them of their
constitutional rights. I01 Pagaragan immediately marked the seized items by placing the marking
"ADP". After searching and marking the evidence found on the first floor, the team, together with the
barangay officials and accused-appellants, proceeded to, and conducted the search on the second and
third floors but found nothing. They went downstairs where they conducted the inventory of recovered
items. I01 Pagaragan prepared the Receipt/Inventory of Property Seized and a Certification of Orderly
Search which were later signed by the barangay officials.
After their arrest, the Punzalans were brought to the PDEA Office in Quezon City for
investigation. IO1 Pagaragan presented the seized evidence to Atty. Benjamin Gaspe, who prepared the
Booking Sheet and Arrest Report, Request for Drug Test/Physical and Medical Examination. They
likewise caused the preparation of their respective affidavits. Photographs were also taken during the
actual search and inventory. Laboratory examination of the seized pieces of drug evidence gave positive
results for the presence of methamphetamine hydrochloride, otherwise known as shabu, a dangerous
drug.
Thereafter, the Punzalans were charged with violation of Section 11, Article II of R.A. No. 9165
for illegal possession of 40.78 grams of methamphetamine hydrochloride otherwise known as shabu, a
dangerous drug.
The trial court convicted the Punzalans. The trial court held that the issuance of a search warrant
against the premises of different persons named therein is valid as there is no requirement that only one
search warrant for one premise to be searched is necessary for its validity. Also, the address of the
Punzalans was clearly and adequately described. A sketch that specifically identifies the places to be
searched was attached to the records and such description of the place was unquestionably accurate that
the PDEA agents were led to and were able to successfully conduct their operation in the premises
described in the search warrant.
The trial court also ruled that the implementation of the search warrant sufficiently complied
with the requirements of the law. Despite the Punzalans' assertion that they were arrested outside their
house and were made to board a van parked along the street beside the river and were not allowed by the
PDEA agents to witness the search conducted inside the house, the trial court was convinced that the
Punzalans were in fact inside their house and were physically present during the conduct of the search.
In its findings, the trial court observed that there were actually two phases of the search done in
the Punzalan house. The first or initial search was done at the ground floor of the house, immediately
after the PDEA agents gained entry and was beyond doubt made in the presence of both accused. This is
where the bulk of illegal drugs were found, confiscated and consequently marked. The trial court further
stated that it is of no moment that the barangay officials were not able to witness the said initial search
and their failure to arrive on time to witness the first or initial search at the ground floor of the Punzalan
house, or even their total absence thereat, will not render the subject search invalid and unlawful
inasmuch as their presence is not required. The trial court held that the prosecution successfully and
sufficiently established that the two accused were present during the initial search, thus, satisfying the
requirement of a lawful and valid search.
The second phase of the search was conducted at the upper floors of the house after the markings
on the 293 sachets of confiscated specimens were completed by I01 Pagaragan. This was witnessed and
participated in by the barangay officials. Finally, after the search of the entire house was concluded, it is
not disputed that an inventory of all the items seized was conducted by I01 Pagaragan in compliance
with the provisions of Section 21, Article II of R.A. No. 9165. In fact, it was admitted by the barangay
officials that they were requested to wait for the DOJ representative, to which they willingly acceded.
The Punzalans filed a motion for reconsideration but it was denied.
On appeal, the CA affirmed the conviction of the Punzalans. The CA held that there was a valid search
and seizure conducted and the seized items are admissible in evidence. The prosecution was able to
prove all the elements of illegal possession of dangerous drugs: (1) the accused is in possession of an
item or object which is identified to be a prohibited drug; (2) such possession is not authorized by law;
and (3) the accused freely and consciously possessed the said drug. Hence, this appeal.
Issue:
Whether or Not the search warrant and the subsequent search were valid
Ruling:
Yes, the search warrant and the subsequent search were valid.
Valid search warrant
A.M. No. 03-8-02-SC, entitled "Guidelines on the Selection and Appointment of Executive Judges and
Defining their Powers, Prerogatives and Duties" as approved by the Court in its Resolution of January
27, 2004, as amended, provides:
SEC. 12. Issuance of search warrants in special criminal cases by the RTC of Manila and Quezon
City. - The Executive Judges and, whenever they are on official leave of absence or are not
physically present in the station, the Vice-Executive Judges of the RTCs of Manila and Quezon
City shall have authority to act on applications filed by the National Bureau of Investigation
(NBI), the Philippine National Police (PNP) and the Anti-Crime Task Force (ACTAF), for search
warrants involving heinous crimes, illegal gambling, illegal possession of firearms and
ammunitions as well as violations of the Comprehensive Dangerous Drugs Act of 2002, the
Intellectual Property Code, the Anti-Money Laundering Act of 2001, the Tariff and Customs
Code, as amended, and other relevant laws that may hereafter be enacted by Congress, and
included herein by the Supreme Court.
The search warrant issued by the RTC of Manila, Branch 17 satisfactorily complies with the
requirements for the issuance thereof as determined by the issuing court, thus:
In the issuance of a search warrant, probable cause requires such facts and circumstances that
would lead a reasonably prudent man to believe that an offense has been committed and the objects
sought in connection with that offense are in the place to be searched. There is no exact test for the
determination of probable cause in the issuance of search warrants. It is a matter wholly dependent on
the finding of trial judges in the process of exercising their judicial function. When a finding of probable
cause for the issuance of a search warrant is made by a trial judge, the finding is accorded respect by
reviewing courts.

Valid search
Section 8, Rule 126 of the Revised Rules of Criminal Procedure provides:
SEC. 8. Search of house, room, or premises to be made in presence of two witnesses. - No search
of a house, room, or any other premises shall be made except in the presence of the lawful
occupant thereof or any member of his family or in the absence of the latter, two witnesses of
sufficient age and discretion residing in the same locality.
As correctly ruled by the CA, even if the barangay officials were not present during the initial search,
the search was witnessed by the Punzalans themselves, hence, the search was valid since the rule that
"two witnesses of sufficient age and discretion residing in the same locality" must be present applies
only in the absence of either the lawful occupant of the premises or any member of his family.

You might also like